Infectious diseases of the skin Flashcards

1
Q

1- Your mother calls you because she has a friend who came back from a wedding 3 days ago and has a red area on her leg. She says she may have had a bug bite but isn’t sure. It started as a red bump and it started to spread and is now about 6 cm, and almost target-like with redness on the outside and inside and a paler pink between. You recommend that she go to her doctor immediately. She has no medication allergies. What is the best course of action for the doctor?

A. Prescribe oral clarithromycin
B. Obtain IgG and IgM studies
C. Obtain urine antigen studies
D. Prescribe oral doxycycline
E. Admit the patient to the hospital for IV ceftriaxone

A

Correct choice: D. Prescribe oral doxycycline

Explanation: The patient most likely has erythema migrans, a cutaneous and early form of Lyme disease. The best course of action for classic cutaneous symptoms is treatment with oral doxycycline. Alternatives are only used in pregnancy or allergy. The other options highlight variable other testing for Lyme disease. The CDC recomends a 2-step testing procedure with an enzyme immunoassay or an indirect immunofluorescence assay followed, if positive, by an immunoblot test. IgG and IgM as well as urine antigen studies are not recommended. Oral clarithromycin and IV ceftriaxone are not appropriate in this case as the patient has no medication allergies and has early disease. Doxycycline is the perferred treatment.

How well did you know this?
1
Not at all
2
3
4
5
Perfectly
2
Q

2- Which of the following is the most common treatment for sporotrichosis?

A. Supersaturated potassium iodide
B. Amphotericin B
C. Surgical excision
D. Cryotherapy
E. Itraconazole

A

Correct choice: E. Itraconazole

Explanation: Sporotrichosis (also known as “rose gardener’s disease”) is an infection caused by the fungus Sporothrix schenckii. Cutaneous (skin) infection is the most common form of the infection. It occurs when the fungus enters the skin through a small cut or scrape, usually after someone touches contaminated plant matter. Skin on the hands or arms is most commonly affected.
The first symptom of cutaneous sporotrichosis is usually a small, painless papule that can develop any time from 1 to 12 weeks after exposure to the fungus. The papule can be red, pink, or purple, and it usually appears on the finger, hand, or arm where the fungus has entered through a break in the skin. The papule will eventually grow larger and may look like an erosion or ulcer that is very slow to heal. Additional papules or ulcers may appear later near the original one.
The most common treatment for this type of sporotrichosis is itraconazole, taken by mouth for 3 to
6 months. Supersaturated potassium iodide (SSKI) is another treatment option for cutaneous sporotrichosis. For those with severe sporotrichosis that affects the lungs, bones, joints, or central nervous system, intravenous amphotericin B is the treatment of choice. After the first treatment with amphotericin B, patients may then receive itraconazole by mouth, for a total of at least 1 year of antifungal treatment. People with sporotrichosis in the lungs may also need surgery to cut away the infected tissue.

How well did you know this?
1
Not at all
2
3
4
5
Perfectly
3
Q

3- A 30-year-old female presents complaining of thickened, yellowish, dystrophic toenails. On examination, you also notice maceration and scaling of the toe-web spaces and soles of the feet. You suspect she may have chronic tinea pedis and a dermatophyte infection of the nail unit. Which of the following is not a common causative pathogen of tinea unguium?

A. Trichophyton tonsurans
B. Trichophyton rubrum
C. Trichophyton interdigitale
D. Epidermophyton floccosum
E. Microsporum canis

A

Correct choice: E. Microsporum canis

Explanation: Tinea unguium refers specifically to dermatophyte infection of the nail unit. It occurs worldwide, affects men more often than women, and is frequently associated with chronic tinea pedis. Trauma and other nail disorders represent predisposing factors. Although all dermatophytes can cause tinea unguium, Microsporum spp. do so very rarely. The most common causative pathogens are T. rubrum, T. interdigitale, T. tonsurans (in children), and E. floccosum.

How well did you know this?
1
Not at all
2
3
4
5
Perfectly
4
Q

4- What is the most likely diagnosis?

A. Syphilis
B. Gingival hypertrophy
C. Necrotizing periodontitis
D. Pyogenic stomatitis
E. Scurvy

A

Correct choice: C. Necrotizing periodontitis

Explanation: This is necrotizing periodontitis which is a marker of severe immunosuppression. Scurvy has bleeding gums, pyostomatitis vegetans is associated with IBD, gingival hypertrophy is a medication side effect commonly from cyclosporine.

How well did you know this?
1
Not at all
2
3
4
5
Perfectly
5
Q

5- A 20-year-old man who recently returned from the Caribbean presents to your office with a painful penile ulcer and painful inguinal adenitis. A lesional swab for Gram stain shows a “school of fish” pattern. What is the most likely causative organism?

A. Herpes simplex virus
B. Treponema pallidum
C. Chlamydia trachomatis L1-3
D. Haemophilus ducreyi
E. Klebsiella granulomatis

A

Correct choice: D. Haemophilus ducreyi
Explanation: The stem describes the classic presentation of chancroid, a sexually-transmitted infection caused by the bacterium H. ducreyi. Worldwide, chancroid prevalence has declined, although infection might still occur in some regions of Africa and the Caribbean. Like genital herpes and syphilis, chancroid is a risk factor in the transmission and acquisition of HIV infection. A lesional swab sent for Gram stain classically shows the Gram-negative coccobacilli arranged in a “school of fish pattern.” Several effective treatment regimens exist, the most common being Azithromycin 1g orally in a single dose. The remaining listed organisms do not cause chancroid.

How well did you know this?
1
Not at all
2
3
4
5
Perfectly
6
Q

6- A patient with AIDS developed purulent nodules and draining sinuses of the neck. Gram stain of pustular contents supports the diagnosis of which infectious organism?

A. Actinomyces israelii
B. Staphylococcal aureus
C. Pseudomonas aeruginosa
D. Mucor spp.
E. Blastomyces dermatitidis

A

Correct choice: A. Actinomyces israelii
Explanation: Actinomyces israelii are filamentous, gram-positive, and non-acid-fast bacteria that typically present as fluctuant nodules classically at the angle of the jaw (“lumpy jaw”) with fistulas and draining sinuses containing “sulfur granules”. IV Penicillin G is the initial treatment and should be used for 4-6 weeks. Oral Penicillin may be required afterwards for 2-12 months depending on severity of infection.

How well did you know this?
1
Not at all
2
3
4
5
Perfectly
7
Q

7- A patient presents to your office with an ulcerated nodule with regional lymphadenopathy on his forearm. A culture of the ulcerated nodule grows Burkholderia mallei. The patient improves with doxycycline. When the patient asks why he developed this condition, you tell her this disease is caused by contact with which of the following?

A. Horses
B. Pigs
C. Sheep
D. Sphagnum moss
E. Dogs

A

Correct choice: A. Horses
Explanation: The patient in this clinical vignette has glanders caused by Burkholderia mallei. It is a disease caused by contact with infected horses. The characteristic features consist of an ulcerated nodule with regional lymphadenopathy. Treatment options include imipenem or doxycycline or sulfadiazine.

How well did you know this?
1
Not at all
2
3
4
5
Perfectly
8
Q

8- What is the most likely diagnosis?

A. Acute paronychia
B. Onychomycosis
C. Herpetic whitlow
D. Orf
E. Disseminated gonococcal infection

A

Correct choice: C. Herpetic whitlow
Explanation: This is an image of herpetic whitlow, which is typified by grouped vesicles on an erythematous base occurring on the finger(s). Herpetic whitlow is caused by herpes simplex virus (type 1 or 2) during primary infection or as result of autoinoculation. Commonly, it is caused by HSV-2 in adults with positive history for genital infection.
The other answer choices do not present with grouped vesicles on an erythematous base.

How well did you know this?
1
Not at all
2
3
4
5
Perfectly
9
Q

9- These nail findings are most likely due to infection with which organism?

A. Trichophyton tonsurans
B. Trichophyton mentagrophytes
C. Trichophyton rubrum
D. Trichophyton verrucosum
E. Trichophyton schoenleinii

A

Correct choice: C. Trichophyton rubrum
Explanation: The image displays the clinical features of onychomycosis. T. rubrum is the most common cause of onychomycosis in the US. The remaining listed dermatophytes do not cause onychomycosis as often as T. rubrum.

How well did you know this?
1
Not at all
2
3
4
5
Perfectly
10
Q

10- A infectious pustular dermatitis develops in a goat farmer. The causative organism is known to be very sturdy and survives many months in the cold winter. There is localized lymphadenopathy and crusting. The most likely etiologic organism is:

A. Sporothrix schenckii
B. Francisella tularensis
C. Parapox virus
D. Erysipelothrix rhusiopathiae
E. Orthopox virus

A

Correct choice: C. Parapox virus
Explanation: This describes Orf. It is also known as ecthyma contagiosum or contagious pustular dermatosis. It is transmitted to humans by a parapoxvirus in sheep, goats, and reindeer. It is self- limited with an excellent prognosis, thus treatment is supportive. Sporothrix schenckii causes sporotrichosis, a fungal infection associated with inoculation via rose thorns. Tularemia, which is associated with rabbit contact, is caused by the bacterium Francisella tularensis. Erysipelothrix rhusiopathiae, the causative agent of erysipeloid, is a bacterium that is transmitted via pigs or fish. Orthopox viruses can cause vaccinia, smallpox, or cowpox.

How well did you know this?
1
Not at all
2
3
4
5
Perfectly
11
Q

11- Which of the following is FALSE regarding actinomycosis?

A. Poor dental hygiene is a risk factor
B. It most often develops on the jawline
C. It is caused by an anaerobic filamentous Gram-negative bacterium
D. Drainage of yellow sulfur-like granules is characteristic
E. The treatment of choice is penicillin

A

Correct choice: C. It is caused by an anaerobic filamentous Gram-negative bacterium
Explanation: Actinomyces israelii, the causative agent of actinomycosis, is an anaerobic filamentous Gram-positive bacterium that is part of the normal oral flora. The other remaining answer choices are true.

How well did you know this?
1
Not at all
2
3
4
5
Perfectly
12
Q

12- A 4-year-old female presents with three warts on the right hand for two months. Her mother tried apple cider vinegar and Mr. Freeze, neither of which helped. What is the next best approach?

A. Cryotherapy x 10 seconds a 2 cycles to all three lesions, use topical salicylic acid at home as directed

B. Inject 0.3 cc Candida antigen into two of the warts after cleaning them with rubbing alcohol pad
C. Excision
D. Electrofulguration
E. Use imiquimod cream Monday, Wednesday, and Friday to each wart under occlusion

A

Correct choice: A. Cryotherapy x 10 seconds a 2 cycles to all three lesions, use topical salicylic acid at home as directed.
Explanation: Cryotherapy is the next most reasonable step for this patient with warts. While several of the alternative modalities might be tried in the future, the most reasonable next step is to perform cryotherapy to each wart after obtaining consent from the parent.

How well did you know this?
1
Not at all
2
3
4
5
Perfectly
13
Q

13- Which of the following is a reasonable first-line treatment for the disease caused by Bartonella bacilliformis and spread by the Lutzomyia sandfly?

A. Ciprofloxacin
B. Vancomycin
C. Clindamycin
D. Imipenem
E. Streptomycin

A

Correct choice: A. Ciprofloxacin
Explanation: Oroya Fever (Carrion’s Disease) is caused by Bartonella bacilliformis. The Lutzomyia sandly is the vector. Multiple antimicrobial agents are active against B. bacilliformis in vitro, including fluoroquinolones, chloramphenicol, doxycycline, and rifampin. Ciprofloxacin is a reasonable first-line treatment, although resistance is increasing. Therefore, many favor a combination of ciprofloxacin and ceftriaxone. Previously, chloramphenicol was the preferred agent, as it has activity against Salmonella as well, a common secondary infection. However, treatment failure has been described with chloramphenicol. In addition treatment of Oroya fever with chloramphenicol does not eliminate risk for development of verruga peruana. The other choices are not considered good treatment options against this organism since relatively high dosease are required to inhibit bacterial growth.

How well did you know this?
1
Not at all
2
3
4
5
Perfectly
14
Q

14- A patient presents with yellowish brown concretions on axillary hair shafts bilaterally. Which of the following non-medical management advice can the patient follow to improve this condition?

A. Avoidance of shaving
B. Shaving
C. Switching to unscented, fragrance-free deodorant
D. Avoiding deodorant
E. Avoid participation in athletics

A

Correct choice: B. Shaving
Explanation: Trichomycosis axillaris is caused by Corynebacterium tenuis. It is characterized by yellowish brown concretions on axillary hair shafts. Treatment options include shaving, benzoyl peroxide gel, topical erythromycin.

How well did you know this?
1
Not at all
2
3
4
5
Perfectly
15
Q

15- A 7-year-old boy presents to your clinic with his mother, who tells you the boy has been having multiple days of flu-like symptoms and a new rash that is asymptomatic. On exam you notice a 5cm erythematous annular patch on his thigh. Both the boy and his mother deny a history of a tick or other insect bite. What is the best treatment?

A. None; this condition is self-resolving
B. Doxycycline
C. IV ceftriaxone
D. Trimethoprim-sulfamethoxazole
E. Amoxicillin

A

Correct choice: E. Amoxicillin
Explanation: The stem describes a classic presentation of early localized Lyme disease, which commonly does not include a positive history of a tick bite. This diagnosis can be made clinically when erythema chronicum migrans (expanding annular erythematous patch at the site of tick bite) is noted on exam as this is pathognomonic. As such, starting antibiotic treatment is warranted to prevent progression of the disease into the early disseminated or chronic stages. In pregnant women and children <8 years old, amoxiciliin for 14-21 days is used due to concern for potential bone and tooth abnormalities induced by tetracycline-class antibiotics. Lyme disease is not self-resolving (although erythema chronicum migrans usually disappears within 4 weeks without treatment) and requires antibiotic treatment to prevent disease progression and its associated morbidity. For adults

and children > 8 years old, doxycycline for 14-21 days is the treatment of choice. The other answer choices are distractors.

How well did you know this?
1
Not at all
2
3
4
5
Perfectly
16
Q

16- A 3-year-old girl is brought into clinic by her father, who reports that she has an itchy rash on the scalp. On exam, you notice a large oval erythematous scaly thin plaque. Wood’s light examination of the lesion demonstrates fluorescence. What is the most likely causative organism?

A. Trichophyton tonsurans
B. Microsporum nanum
C. Trichophyton rubrum
D. Candida albicans
E. Microsporum canis

A

Correct choice: E. Microsporum canis
Explanation: The question stems describes a case of fluorescent tinea capitis, which is most often due to infection with M. canis. T. tonsurans is the most common cause of tinea capitis in North America, however this is an endothrix infection and thus does not demonstrate fluorescence with Wood’s light examination. T. rubrum and M. nanum rarely produce non-fluorescent ectothrix tinea capitis. Candida albicans does not cause tinea capitis.

How well did you know this?
1
Not at all
2
3
4
5
Perfectly
17
Q

17- A 19-year-old military recruit presents to the dermatology office with intensely pruritic papules on the trunk and genital region. A papule is scraped with a #15 blade and immersed in mineral oil. The following image is seen on microscopy. What is the most likely diagnosis?

A. Tinea corporis
B. Tinea versicolor
C. Pityrosporum folliculitis
D. Scabies
E. Cheyletiellosis

A

Correct choice: D. Scabies

Explanation: The above image shows scabetic scybala (fecal pellets). The diagnosis can be confirmed through the detection of scabies mites, eggs or feces with a mineral oil prep.
Tinea corporis on KOH prep will show segmented hyphae. Tinea versicolor and pityrosporum folliculitis will show both hyphae and yeast cells on KOH. Cheyletiellosis, also referred to as “walking dandruff”, are nonburrowing mites commonly found on rabbits, dogs, and cats. The mites have been known to cause disease in humans, ranging from mild dermatitis to more severe illness with systemic symptoms. Because these mites do not complete any part of their life cycle in humans, diagnosis can be challenging and one is unlikely to find these findings on a skin scraping.

How well did you know this?
1
Not at all
2
3
4
5
Perfectly
18
Q

18- You are called for an emergent consult in a patient with exquisite tenderness, erythema, warmth and swelling that does not respond to antibiotics. The skin appears shiny and tense. The patient complains of pain out of proportion to skin findings. Within 36 hours the skin changes from reddish in color to a grayish-blue color in ill-defined patches. You notice a thin, watery, malodorous fluid. An x-ray of her forearm (pictured) shows the presence of gas in the soft tissues. Which of the following risk factors is NOT associated with higher mortality in this life-threatening condition?

A. Diabetes mellitus
B. Elevated lactic acid level
C. Elevated serum creatinine
D. Delay to first debridement
E. Disease due to Staphylococcus aureus

A

Correct choice: E. Disease due to Staphylococcus aureus
Explanation: Necrotizing fasciitis is characterized by rapidly progressive necrosis of subcutaneous fat and fascia, which can be life-threatening without prompt recognition, aggressive surgical intervention and immediate antibiotic therapy. Higher mortality is associated with female sex, older age, malnutrition, greater extent of infection, delay to first debridement, an elevated serum creatinine or lactic acid levels, disease due to group A streptococci, and a greater degree of organ dysfunction at the time of admission to hospital. Diabetes mellitus can also result in higher mortality, particularly if renal dysfunction or peripheral arterial disease is also present.
1 –Diabetes can also result in a higher mortality in necrotizing fasciitis, particularly if renal dysfunction or peripheral arterial disease is also present. 2 – Higher mortality in necrotizing fasciitis is associated with female sex, older age, malnutrition, greater extent of infection, delay to first debridement, an elevated serum creatinine or lactic acid level, disease due to group A streptococci, and greater degree of organ dysfunction at the time of admission to hospital. 3 – Higher mortality in necrotizing fasciitis is associated with female sex, older age, malnutrition, greater extent of infection, delay to first debridement, an elevated serum creatinine or lactic acid level, disease due to group A streptococci, and greater degree of organ dysfunction at the time of admission to hospital. 4
- Higher mortality in necrotizing fasciitis is associated with female sex, older age, malnutrition, greater extent of infection, delay to first debridement, an elevated serum creatinine or lactic acid level, disease due to group A streptococci, and greater degree of organ dysfunction at the time of admission to hospital.

How well did you know this?
1
Not at all
2
3
4
5
Perfectly
19
Q

19- This patient has a cat. What is the etiology of her findings?

A. Borrellia recurrentis
B. Bartonella quintana
C. Bartonella bacilliformis
D. Borrellia burgdoferi
E. Bartonella henselae

A

Correct choice: E. Bartonella henselae
Explanation: This is submental lymphadenopathy secondary to cat scratch disease due to bartonella henselae. It is one of the most common causes of chronic lymphadenopathy in children. B. quintana causes bacillary angiomatosis and trench fever. B. Bacilliformis causes oroya fever/verruga peruana. B. Burgdorferi causes lyme disease and B. recurrentis causes relapsing fever.

How well did you know this?
1
Not at all
2
3
4
5
Perfectly
20
Q

20- Which of the following is the most appropriate treatment of this infant’s painful finger?

A. Incision and drainage
B. Acyclovir
C. Foscarnet
D. Dicloxacillin
E. Clindamycin

A

Correct choice: B. Acyclovir
Explanation: The most appropriate treatment of herpetic whitlow is acyclovir.
This question requires the examinee to clinically identify herpetic whitlow by the presence of clustered vesicles with surrounding erythema and edema located on a distal digit. Herpetic whitlow, caused by direct inoculation of the digital pulp by herpes simplex virus (HSV) 1 or 2, should be treated with acyclovir (choice 2). Incision and drainage (choice 1) may be required to treat a digital pulp abscess (felon), which would manifest with an erythematous fluctuant nodule lacking clustered vesicles. Foscarnet (choice 3) is used to treat acyclovir-resistant HSV infections, which typically occur in immunosuppressed patients. Dicloxacillin (choice 4) treats streptococcal and staphylococcal infections. On the differential diagnosis for herpetic whitlow is blistering distal dactylitis, usually caused by group A Streptococcus and appropriately treated with dicloxacillin. However, blistering distal dactylitis presents with a single vesicle or bulla, rather than clustered vesicles, at the palmar fingertip. Clindamycin (choice 5) is used to treat methicillin-resistant Staphylococcus aureus infections as well as bacterial toxin-mediated diseases (staphylococcal scalded skin syndrome, toxic shock syndrome), which would not present with clustered vesicles on a digit.

How well did you know this?
1
Not at all
2
3
4
5
Perfectly
21
Q

21 -Clinically, actinomycotic mycetoma and eumycotic mycetoma appear identical. The importance of identifying the etiologic agent is

A. Academic
B. Selecting the appropriate therapy
C. Ordering appropriate stains
D. Determine if the infection is contagious
E. Determine if amputation is necessary

A

Correct choice: B. Selecting the appropriate therapy
Explanation: The etiologic agents of actinomycotic mycetoma are filamentous bacteria that require antibiotics, whereas fungal agents causing eumycotic mycetoma require antifungal therapy. The other answer choices are not as important as selecting the correct treatment option for patients with mycetoma.

How well did you know this?
1
Not at all
2
3
4
5
Perfectly
22
Q

22- Which of the following are treatment options for a patient who presents with malodor, hyperhidrosis, shallow pits on the plantar surfaces of both feet?

A. Topical erythromycin
B. Topical clindamycin
C. Benzoyl peroxide
D. Doxycycline
E. Choice A, B and C are correct

A

Correct choice: E. Choice A, B and C are correct
Explanation: Pitted keratolysis is caused by Kytococcus sedentarius (formerly Micrococcus sedentarius). It is characterized by 1-3 mm pits on plantar surfaces of both feet, malodor and hyperhidrosis. Treatments include topical erythromycin, clindamycin, or benzoyl peroxide. Oral treatment is not indicated for this condition.

How well did you know this?
1
Not at all
2
3
4
5
Perfectly
23
Q

23- Which bacteria is responsible for these findings?

A. Staphylococcus aureus
B. Kytococcus sedentarius
C. Actinomyces israelii
D. Pseudomonas aeruginosa
E. Corynebacterium tenuis

A

Correct choice: E. Corynebacterium tenuis
Explanation: Trichomycosis axillaris is caused by Corynebacterium (mostly Corynebacterium tenuis). Erythrasma is caused by Corynebacterium minutissimum, and its coral-red fluorescence is caused by production of coproporphyrin III by the organisms.

How well did you know this?
1
Not at all
2
3
4
5
Perfectly
24
Q

24- Sensation is intact in this lesion, but a Fite stain is positive. This lesions is associated with which of the following:

A. IL-2
B. IL-4
C. IL-12
D. IL-15
E. IL-18

A

Correct choice: B. IL-4
Explanation: The lesion is low immune or lepromatous leprosy, which is associated with TH2 cytokines including IL-4, IL-5, IL-10, and IL-13. Tuberculoid leprosy is associated with Th1 cytokines including IFN-gamma, IL2, IL12, IL15, IL18 and IL23.

How well did you know this?
1
Not at all
2
3
4
5
Perfectly
25
Q

25- A goat farmer presents with fever and flu-like symptoms, as well as violaceous papulonodules on the trunk and lower extremities. You consider Malta fever in your differential diagnosis. Which of the following statements regarding this diagnosis is correct?

A. Treatment is with doxycycline
B. Treatment is with ciprofloxacin
C. Skin lesions are frequently seen
D. Caused by a gram positive rod
E. Occurs frequently in the US

A

Correct choice: A. Treatment is with doxycycline

Explanation: Treatment of malta fever is with doxycycline or rifampin. Malta fever is another name for Brucellosis. This infection is caused by ingesting raw goat milk and unpasteurized goat cheese infected with Brucellae (gram-negative rods). It is infrequent in the US, occurring mainly in veterinarians and farmers. Acute brucellosis has very non-specific features and presents as a flu-like illness. Skin lesions are infrequent (20% of cases) but are usually violaceous papulonodules on the trunk and lower extremities.

How well did you know this?
1
Not at all
2
3
4
5
Perfectly
26
Q

26- Brucellosis is typically associated with which of the following routes of transmission?

A. Contact with rabbits
B. Ingestion of raw seafood
C. Flea bite from infected animal
D. Inhalation of aerosolized bird excretions
E. Ingestion of unpasteurized dairy products

A

Correct choice: E. Ingestion of unpasteurized dairy products
Explanation: Brucellosis (aka Undulant fever; Malta fever) is caused by the gram-negative bacterium Brucella spp. It is typically contracted via direct contact with an infected animal (undulant) or ingestion of infected meat or unpastuerized dairy products. Butchers, farmers, veterinarians, and unwise tourists are at highest risk. Clinical findings include: cyclic fevers, arthralgias, hepatosplenomegaly, rare skin involvement with a violaceous papulonodular eruption, and/or endocarditis (most common cause of death). The treatment of choice is doxycycline combined with rifampin. The remaining answer choices have the following disease associations: Contact with rabbits-Tularemia, Ingestion of raw seafood- Vibrio infection, Flea bite from infected animal- Plague ,Inhalation of aerosolized bird excretions- Psittacosis.

How well did you know this?
1
Not at all
2
3
4
5
Perfectly
27
Q

27- What is the treatment for secondary syphilis in a penicillin allergic patient?

A. Penicillin
B. Doxcycyline
C. Rifampiin
D. Clindamycin
E. Ceftriaxone

A

Correct choice: B. Doxcycyline
Explanation: Secondary syphillis is caused by the spirochete, T. pallidum. Penicillin is the treatment of choice for secondary syphillis. In penicillin allergic individuals, doxycyline may be used. Treatment for neurosyphilis, congenital syphilis and syphilis in pregnant women is desensitization to penicillin and then treatment with pcn.

How well did you know this?
1
Not at all
2
3
4
5
Perfectly
28
Q

28- A 20-year-old man developed edematous, pink to red follicular papules and pustules primarily on the trunk while on vacation. He remembers being in the jacuzzi about 12 hours prior to the onset of his rash. The lesions are itchy. After 3 weeks, the lesions resolve spontaneously. Which of the following organisms is most likely responsible for this patient’s initial skin condition?

A. Staphylococcus aureus
B. Streptococcus pyogenes
C. Proteus mirabilis
D. Pseudomonas aeruginosa
E. Escherichia coli

A

Correct choice: D. Pseudomonas aeruginosa
Explanation: Hot tube folliculitis is due to Pseudomonas aeruginosa in the setting of use of a hot tub/whirlpool 12-48 hours prior to onset. It is characterized by edematous, pik to red follicular papules and pustules, primarily on the trunk. It is often pruritic and resolve sspontaneously in immunocompetent hosts. It my be a serious illness in immunocompromised hosts. The condition is generally self-limited - patients can use antibacterial soap. If severe or immunocompromised, patient can take ciprofloxacin for 7-14 days. The water in the hot tub, whirpool or swimming pool must be maintained with chlorine and changed every 6-8 weeks to lower organic carbon level.

How well did you know this?
1
Not at all
2
3
4
5
Perfectly
29
Q

29- What is the etiologic agent for this lesion?

A. Pox virus
B. Herpes virus
C. Human papilloma virus
D. Togavirus
E. Coxsackie virus

A

Correct choice: A. Pox virus
Explanation: This is molluscum contagiosum which is due to a pox virus.

How well did you know this?
1
Not at all
2
3
4
5
Perfectly
30
Q

30- This organism causes a resistant tinea pedis that is clinically indistinguishable from dermatophytosis:

A. Aspergillus flavus
B. Scytalidium dimidiatum
C. Curvularia sp.
D. Scopulariopsis
E. Fonsecaea sp

A

Correct choice: B. Scytalidium dimidiatum
Explanation: Scytalidium dimidiatum is a non-dermatophyte cause of tinea pedis that is very difficult to treat due to its resistance to most antifungals. It is sensitive to cylcoheximide so it should be cultured on media free of this antifungal.

How well did you know this?
1
Not at all
2
3
4
5
Perfectly
31
Q

31- A woman who handles fish tanks develops a slow-growing cluster of papules on the hand. Which antibiotic is a reasonable single-drug approach to treatment?

A. Isoniazid
B. Clarithromycin
C. Rifampin
D. Penicillin
E. Dapsone

A

Correct choice: B. Clarithromycin
Explanation: This patient most likely has contract Mycobacterium marinum, otherwise known as “fish tank granuloma.” A reasonable approach to treatment of M. marinum consists of treatment with two antibiotics for one to two months after resolution of symptoms. One approach consists of clarithromycin combined with ethambutol OR rifampin. For minimal disease, some favor single- drug therapy with clarithromycin, minocycline or doxycline. The other antibiotic choices are not considered first-line for single-drug therapy.

How well did you know this?
1
Not at all
2
3
4
5
Perfectly
32
Q

32- What characteristic color is seen on diascopy of this infectious lesion?

A. Apple jelly
B. Blue
C. Orange
D. Red
E. Coral red

A

Correct choice: A. Apple jelly
Explanation: Leishmaniasis recidivans is a type of Old World leishmaniasis. Clinically, it appears as a red papule covered with white scale. On diascopy, it has a characteristic apple jelly color. The other listed colors are not seen on diascopy of leishmaniasis.

How well did you know this?
1
Not at all
2
3
4
5
Perfectly
33
Q

33- The most common cause of tinea capitis in the United States today is:

A. Trichophyton schoenleinii
B. T. tonsurans
C. T. mentagrophytes
D. Microsporum audouinii
E. M. canis

A

Correct choice: B. T. tonsurans

Explanation: T. tonsurans is currently the primary cause of tinea capitis in the US.
None of the remaining listed answers is the most common current cause of tinea capitis (M. audouinii was previously the primary cause of tinea capitis in the US).

How well did you know this?
1
Not at all
2
3
4
5
Perfectly
34
Q

34- This lesion is hypoesthetic and is associated with which of the following:

A. Interleukin 2
B. Interleukin 4
C. Interleukin 5
D. Interleukin 10
E. Interleukin 13

A

Correct choice: A. Interleukin 2

Explanation: The lesion is high immune or tuberculoid leprosy, which is associated with Th1 cytokines including IFN-gamma, IL2, IL12, IL15, IL18 and IL23. Lepromatous leprosy is associated with Th2 cytokines including IL4, 5, 10 and 13.

How well did you know this?
1
Not at all
2
3
4
5
Perfectly
35
Q

35- What organism caused this disease?

A. Kytococcus sedentarius
B. Candida albicans
C. Staphylococcus aureus
D. Corynebacterium tenuis
E. Corynebacterium diptheria

A

Correct choice: A. Kytococcus sedentarius
Explanation: Pitted keratolysis is a skin disorder characterized by crateriform pitting that primarily affects the pressure-bearing aspects of the plantar surface of the feet. It is caused by a cutaneous infection with micrococcus sedentarius which is included in the Corynebacteria genus. These are gram-positive, catalase-positive, aerobic or facultatively anaerobic, generally nonmotile rods. Corynebacterium diphtheriae infection is classically characterized by a local inflammation, usually in the upper respiratory tract, associated with toxin-mediated cardiac and neural disease. Corynebacterium tenuis causes trichomycosis Patients typically present with yellow, black, or red pinpoint nodules on the hair shafts in the inguinal region. Staphylococcus aureus is a common bacterium that can result in formation of pustules, furuncles, cellulitis and abscesses. Candida species are responsible for superficial infections such as oropharyngeal candidiasis (thrush) and vulvovaginal candidiasis (vaginal Candidiasis) and is also occurs as an opportunistic infection The other associated organisms do not cause pitted keratolysis.

How well did you know this?
1
Not at all
2
3
4
5
Perfectly
36
Q

36- Trichomycosis axillaris is caused by:

A. Burkholderia mallei
B. Proteus species
C. Corynebacterium tenuis
D. Micrococcus sedentarius
E. Corynebacterium minitissimum

A

Correct choice: C. Corynebacterium tenuis

Explanation: Corynebacterium tenuis is the causative organism associated with most cases. Although up to 33% of adults have colonization by this bacterium in the inguinal or axillary regions, factors such as hyperhidrosis predispose to more extensive growth and resultant clinical manifestations. The other answer choices listed are incorrect.

How well did you know this?
1
Not at all
2
3
4
5
Perfectly
37
Q

37- This Nigerian patient tells you that he is a farmer. Which of the following is the most likely causative organism?

A. M. marinum
B. M. tuberculosis
C. M. kansasii
D. M. ulcerans
E. M. fortuitum

A

Correct choice: D. M. ulcerans
Explanation: The image depicts a Buruli ulcer, which is due to infection with Mycobacterium ulcerans. Buruli ulcer is a neglected tropical disease of skin and subcutaneous tissue that can result in long-term cosmetic and functional disability. It is a geographically restricted infection but transmission has been reported in endemic areas in more than 30 countries worldwide. The heaviest burden of disease lies in West and Sub-Saharan Africa where it affects children and adults in subsistence agricultural communities. Mycobacterium ulcerans infection is probably acquired via inoculation of the skin either directly from the environment or indirectly via insect bites. The environmental reservoir and exact route of transmission are not completely understood. Treatments include excision, local heating (as mycobacteria prefer cooler temperatures); drug therapy is often disappointing. The other mycobacteria are not the most common cause of a Buruli ulcer.

How well did you know this?
1
Not at all
2
3
4
5
Perfectly
38
Q

38- Which of the following is true about this condition?

A. Histology will show amastagotes in plasma cells
B. Cuture is performed on MacConkey agar
C. Diagnosis may be made by sending a sample for real-time PCR assay to the Centers for Disease Control and Prevention
D. This condition does not typically resolve without treatment E.Organisms are evenly spaced and surrounded by a pseudocapsule

A

Correct choice: C. Diagnosis may be made by sending a sample for real-time PCR assay to the Centers for Disease Control and Prevention.
Explanation: This is cutaneous leishmaniasis. Amastagotes are seen in dermal macrophages. Leishmaniais grows on Nicolle-Novy-MacNeal media or chick embryo media. The CDC does complimentary diagnoses for leishmaniasis with PCR. Old World cutaneous leishmaniasis and New World leishmaniasis typically resolves without treatment, but is treated to minimize scarring and prevent dissemination. The treatment of choice is parenteral pentavalent antimonials for cutaneous and mucocutaneous forms, while liposomal amphotericine B is used for visceral leishmaniasis. Leishmaniasis organisms often line up at periphery of vacuole without forming any surrounding pseudocapsules in contrast to histoplasma infection with evenly spaced organisms, that are surrounded by a pseudocapsule.

How well did you know this?
1
Not at all
2
3
4
5
Perfectly
39
Q

39- The treatment of choice for Oroya Fever is:

A. Penicillin
B. Doxycycline
C. Erythromycin
D. Minocycline
E. Ciprofloxacin

A

Correct choice: E. Ciprofloxacin
Explanation: Oroya Fever (Carrion’s Disease) is caused by Bartonella bacilliformis. The Lutzomyia sandly is the vector. Multiple antimicrobial agents are active against B. bacilliformis in vitro, including fluoroquinolones, chloramphenicol, doxycycline, and rifampin. Ciprofloxacin is a reasonable first-line treatment, although resistance is increasing. Therefore, many favor a combination of ciprofloxacin and ceftriaxone. Previously, chloramphenicol was the preferred agent, as it has activity against Salmonella as well, a common secondary infection. However, treatment failure has been described with chloramphenicol. In addition treatment of Oroya fever with chloramphenicol does not eliminate risk for development of verruga peruana. The other answer choices are not the current choice of antibiotics for Oroya Fever.

How well did you know this?
1
Not at all
2
3
4
5
Perfectly
40
Q

40- An immunosuppressed patient with a history of a renal transplant develops folliculocentric facial spicules (see photo). A virus was detected. What is the most likely causative organism?

A. Merkel cell polyomavirus (MCPyV)
B. HSV
C. HPV
D. Trichodysplasia spinulosa-associated polyoma virus (TSPyV)
E. VZV

A

Correct choice: D. Trichodysplasia spinulosa-associated polyoma virus (TSPyV)
Explanation: She has trichodysplasia spinulosa which is seen in settings of immunosuppression. Recently, TSV has been identified as the causative agent.

How well did you know this?
1
Not at all
2
3
4
5
Perfectly
41
Q

41- Which of the following is most closely associated with Staphylococcus aureus?

A. Chronic paronychia
B. Bullous impetigo
C. Bullous cellulitis
D. Blistering distal dactylitis
E. Ecthyma

A

Correct choice: B. Bullous impetigo

Explanation: Bullous impetigo is caused by phage group II Staphylococcus aureus. This question tests the examinee’s knowledge of bacterial infections and in particular those caused by S. aureus.

Of the answer choices, bullous impetigo (choice 2) is most closely associated with S. aureus infection. In bullous impetigo, phage group II S. aureus produces exfoliatoxins A/B which cleave desmoglein 1, leading to bulla formation. In contrast to acute paronychia, which is commonly caused by S. aureus, chronic paronychia (choice 1) is related more to chronic irritant/allergic contact dermatitis than infection. Most cases of cellulitis (choice 3), bullous or otherwise, are not cultured, but it is generally considered to be caused more often by streptococci than staphylococci. Blistering distal dactylitis (choice 4) and ecthyma (choice 5) are most commonly caused by Streptococcus pyogenes.

How well did you know this?
1
Not at all
2
3
4
5
Perfectly
42
Q

42- A 34-year-old male presents to clinic for evaluation of a scaly eruption of the bilateral groins for the past 6 months. His primary care physician has prescribed topical terbinafine, topical hydrocortisone, and topical triamcinolone, each of which demonstrated no benefit after a one month trial. A Wood’s lamp exam demonstrated coral red fluorescence within the eruption. What organism is responsible for this condition?

A. Corynebacterium minutissimum
B. Trichophyton Rubrum
C. Malassezia furfur
D. Kytococcus sedentarius
E. Trueperella pyogenes

A

Correct choice: A. Corynebacterium minutissimum

Explanation: Choice 1, corynebacterium minutissimum, is the causative agent in erythrasma, which is the described condition. Coral-red fluorescence with Wood’s light is attributable to coproporphyrin III, produced by the bacterium. Trichophyton rubrum is a dermatophyte, which would respond well to topical terbinafine. Malassezia furfur is the yeast implicated in Pityriasis Versicolor. Kytococcus sedentarius is a bacterium which is often associated with pitted keratolysis.

How well did you know this?
1
Not at all
2
3
4
5
Perfectly
43
Q

43- A 54 year old male with recent weight loss develops disseminated flesh-colored large papules with central umbilication. A diagnosis of molluscum contagiosum is made. Which of the following conditions should be considered testing for?

A. HIV status

B. Hepatitis C status
C. Diabetes mellitus
D. Familial hypercholesterolemia
E. Porphyria cutanea tarda

A

Correct choice: A. HIV status
Explanation: Molluscum contagiosum can occur in the setting of immunodeficiency, including in patients with HIV or following treatment with immunosuppressive drugs. In immunocompromised patients, lesions can be large and widespread. Since the introduction of potent antiretroviral therapy, the number of molluscum cases in HIV patients has decreased substantially.
Patient with extensive lesions should be tested for HIV infection and the possibility of other immune system disorders also can be considered. The other conditions are not as strongly associated with large, disseminated molluscum lesions.

How well did you know this?
1
Not at all
2
3
4
5
Perfectly
44
Q

44- A 28-year-old male presents with follicular pustules on the upper trunk and mid-chest. Many of the lesions are arising on an erythematous base. On exam, you notice the eruption favors areas with terminal hairs and is also present on the scalp and beard, buttocks and lower extremities. You suspect he may have superficial folliculitis. What is the most common cause of this condition which would be revealed on bacterial culture of pustular contents?

A. Normal flora
B. Staphylococcus aureus
C. Streptococcus
D. Pseudomonas aeruginosa
E. Group B streptococcus

A

Correct choice: A. Normal flora
Explanation:Superficial folliculitis is characterized by follicular pustules, often arising on an erythematous base. It favors areas with terminal hairs, such as the scalp and beard, as well as the upper trunk, buttocks, and lower extremities. Bacterial culture of pustular contents most commonly reveals normal flora, followed by Staphylococcus aureus.

How well did you know this?
1
Not at all
2
3
4
5
Perfectly
45
Q

45- This disorder is linked to an abnormality in which of the following intercellular proteins?

A. Desmoplakin
B. Desmocollin 1
C. Desmoglein 1
D. Desmoglein 3
E. Desmoglein 4

A

Correct choice: C. Desmoglein 1
Explanation: In staphylococcal scalded skin syndrome (SSSS), phage group II Staphyloccocus aureus produces toxins that cleave desmoglein 1.
This question presents the examinee with an image depicting the highly characteristic periorificial radial crusting and fissuring of SSSS, and assesses knowledge of disease pathomechanism. In SSSS, phage group II Staphyloccocus aureus produces exfoliatoxins (epidermolytic toxins) A and B, which disseminate hematogenously to cleave desmoglein 1 (choice 3), leading to desmosomal splitting, granular layer disruption, and formation of superficial bullae. Desmoplakin (choice 1) is mutated in Carvajal syndrome, which consists of striate palmoplantar keratoderma, woolly hair, and left-sided cardiomyopathy. Desmocollin 1 (choice 2) is the target antigen in the subcorneal pustular dermatosis type of IgA pemphigus. Autoantibodies to desmoglein 3 (choice 4) are formed in pemphigus vulgaris. Mutations in desmoglein 4 (choice 5) occur in autosomal recessive monilethrix and localized autosomal recessive hypotrichosis.

How well did you know this?
1
Not at all
2
3
4
5
Perfectly
46
Q

46- A new patient complains of persistent itchy hand and foot rashes not responding to changing soaps, minimizing hand washing, and using an over-the-counter topical steroid. Which of the following is the most appropriate next step in management?

A. High-potency topical steroid under occlusion

B. Phototherapy
C. Skin biopsy
D. Patch testing
E. Potassium hydroxide preparation

A

Correct choice: E. Potassium hydroxide preparation

Explanation: A potassium hydroxide (KOH) preparation should be performed to diagnose tinea manuum/pedis presenting as the “two feet-one hand syndrome.”

This question requires the examinee to identify the “two feet-one hand syndrome” of tinea manuum/ pedis. In the photo only the left hand shows powdery white scale favoring the palmar creases, and there is moccasin scale of the visible sole. Thus, the next best step in management is to perform a KOH preparation (choice 5) to make the diagnosis of dermatophytosis. Although hand dermatitis is included in the differential, in this scenario it would be inappropriate to start a high-potency topical steroid under occlusion (choice 1) or phototherapy (choice 2) prior to ruling out dermatophytosis. A skin biopsy (choice 3) is not required when KOH, a simple in-office procedure, will suffice to make the correct diagnosis. Patch testing (choice 4) for persistent hand dermatitis helps to rule out allergic contact dermatitis, but in this clinical scenario a KOH preparation should be performed first.

How well did you know this?
1
Not at all
2
3
4
5
Perfectly
47
Q

47- A 33-year-old African-American HIV-infected male presents with marked pruritic follicular papules, urticarial and pustular that favor the face, scalp, upper trunk, and upper arms which has been present for months. A biopsy showed lesions characterized by spongiosis and exocytosis of eosinophils and lymphocytes into the follicular epithelium. You tell him treatment of the underlying viral infection with a resultant rise in CD4 cell count may lead to a resolution of lesions. At what CD4 count does this dermatology condition occur in HIV-infected patients?

A. 50
B. 100
C. 200
D. 400
E. 1000

A

Correct choice: C. 200

Explanation: Eosinophilic folliculitis is characterized by markedly pruritic follicular papules, urticarial and/or pustular, that favor the face, scalp, and upper trunk. It occurs in the setting of immunosuppression, including from HIV infection and following allogeneic hematopoietic stem cell transplantation. In HIV-infected patients, the CD4 count is usually <200 cells/mm3; it may also be a manifestation of IRIS (immune reconstitution inflammatory syndrome). Histologically, lesions are characterized by spongiosis and exocytosis of eosinophils and lymphocytes into the follicular epithelium. In HIV-infected patients, the CD4 count in patients with eosinophilic folliculitis is usually <200 cells/mm3; it may also be a manifestation of IRIS (immune reconstitution inflammatory syndrome).

How well did you know this?
1
Not at all
2
3
4
5
Perfectly
48
Q

48- A Brazilian man presents to your office with a chronic ulcer of his right nasal ala. Biopsy reveals histiocytes containing small oval organisms with bar shaped paranuclear kinetoplasts. What is the best treatment for this condition?

A. Amphotericin B
B. Fluconazole
C. Surgical excision
D. Ivermectin
E. Sodium stibogluconate

A

Correct choice: E. Sodium stibogluconate
Explanation: The question stem describes a classic clinio-pathologic presentation of New World mucocutaneous leishmaniasis, which is most often due to L. braziliensis. First line treatment is with a pentavalent antimonial such as sodium stibogluconate or meglumine antimonite. Amphotericin B is the first line treatment for visceral leishmaniasis. The remaining answer choices are not useful in the treatment of New World mucocutaneous leishmaniasis.

How well did you know this?
1
Not at all
2
3
4
5
Perfectly
49
Q

49- A female patient develops herpes simplex viral infections recurrently, at least 7 times per year. The infections are located on the lips. She has tried lysine daily as a treatment, what is the appropriate treatment for this patient without kidney disease?

A. Valacyclovir 2 g BID PO at first sign of outbreak then stop
B. Valacyclovir 1 g TID PO x 7 days at first sign of outbreak then stop
C. Valacyclovir 500 mg daily PO

D. Abreva topical BID at first sign of outbreak then stop
E. Lysine supplement TID x 7 days at first sign of outbreak then stop

A

Correct choice: C. Valacyclovir 500 mg daily PO
Explanation: The dose of valacyclovir for suppressive therapy for herpes simplex virus infections is 500 mg by mouth daily. Patients without significant renal impairment with 4-6 outbreaks of HSV per year (or more) are generally candidates. Valacyclovir 2 g by mouth and repeat dosage in 12 hours is the amount used for intermittent treatment of HSV. Valacyclovir 1 g by mouth TID x 7 days is the dosing schedule for herpes zoster. Abreva and lysine may possibly reduce the duration and severity of HSV outbreaks.

How well did you know this?
1
Not at all
2
3
4
5
Perfectly
50
Q

50- A 3-year-old boy with a history of atopic dermatitis develops numerous monomorphic, punched-out erosions with hemorrhagic crusting and vesicles. It became widespread and affected the head, neck and trunk. Which of the following is not a complication that is commonly associated with this condition?

A. Superinfection with Staphylococcus aureus
B. Superinfection with Streptococcus pyogenes
C. Herpetic keratoconjunctivitis
D. Meningoencephalitis
E. Pneumonia due to Streptococcus pneumoniae

A

Correct choice: E. Pneumonia due to Streptococcus pneumoniae
Explanation: Eczema herpeticum represents rapid dissemination of a herpes simplex viral infection over the eczematous skin of AD patients. It initially develops as an eruption of vesicles, but affected individuals more often present with numerous monomorphic, punched-out erosions with hemorrhagic crusting. Eczema herpeticum is frequently widespread and may occur at any site, with a predilection for the head, neck, and trunk. It is often associated with fever, malaise, and lymphadenopathy, and complications may include superinfection with S. aureus or S. pyogenes as well as herpetic keratoconjunctivitis and meningoencephalitis. Patients with mutations in the filaggrin gene and those who have both severe AD and asthma have an increased risk for eczema herpeticum, and decreased production of antimicrobial peptides may have a pathogenic role. Patients with AD are also predisposed to the development of widespread molluscum contagiosum.

Superinfection with Staphylococcus aureus: Eczema herpeticum is often associated with fever, malaise, and lymphadenopathy, and complications may include superinfection with S. aureus or S. pyogenes as well as herpetic keratoconjunctivitis and meningoencephalitis.

How well did you know this?
1
Not at all
2
3
4
5
Perfectly
51
Q

51- This condition is transmitted by what vector?

A. Black fly
B. Sandfly
C. Anopheles mosquito
D. Deer tick
E. Dog tick

A

Correct choice: B. Sandfly

Explanation: This is Leishmaniasis and is transmitted by the sandfly. The sandfly is the vector for Oroya fever/Carrion’s disease. Onchocerciasis is transmitted by the blackfly.

How well did you know this?
1
Not at all
2
3
4
5
Perfectly
52
Q

52- In this patient, this test will be helpful in making the diagnosis:

A. Fluorescent antibody test for herpes
B. Patch test
C. Tissue culture
D. Indirect immunofluorescence
E. KOH exam

A

Correct choice: A. Fluorescent antibody test for herpes
Explanation: This patient has herpes zoster. A direct fluorescent antibody test for the varicella zoster virus can be performed to confirm the diagnosis

How well did you know this?
1
Not at all
2
3
4
5
Perfectly
53
Q

53- What is the underlying infection in this neonate?

A. Syphilis
B. Leishmania
C. Chlamydia
D. Gonorrhea
E. Tuberculosis

A

Correct choice: A. Syphilis

Explanation: This is pemphigus syphiliticus, the only blistering manifestation of syphilis. It occurs in newborns. They have severe bullous lesions.

How well did you know this?
1
Not at all
2
3
4
5
Perfectly
54
Q

54- Which of the following statements is false regarding this bacterial infection?

A. blister formation is mediated by epidermolytic toxin
B. the toxin binds to desmoglein 1
C. acantholysis occurs within the epidermal granular layer
D. the organism can be cultured from fluid wihtin the blisters
E. the organism cannot be cultured from fluid within the blisters

A

Correct choice: D. the organism can be cultured from fluid wihtin the blisters
Explanation: Most cases of staphylococcal scalded skin syndrome (SSSS) are caused by phage group II strains (e.g. types 55, 71) of S. aureus (which can be methicillin-sensitive or resistant) that produce exfoliative (also know as epidermolytic) toxins. ETA (chromosomally encoded) and ETB (plasmid encoded) are serine proteases that bind and cleave desmoglein 1. This causes splitting of the desmosomes, which leads to disruption of the epidermal granular layer and bulla formation. Desmoglein 1 is targeted by autoantibodies in pemphigus foliaceus, which has identical histological features to SSSS. In contrast to bullous impetigo, where the effects of the ETs are limited to the site(s) of infection, in SSSS the toxin diffuses from a focus of infection and (in the absence of specific antitoxin antibody) spreads hematogenously to produce widespread effects.

How well did you know this?
1
Not at all
2
3
4
5
Perfectly
55
Q

55- The etiology of this infection falls into what category?

A. Amoeba
B. Protozoa
C. Bacteria
D. Fungus
E. Spirochete

A

correct choice: B. Protozoa

Explanation: This is granulomatous leishmania. Leishmania is a protozoa and is located within the macrophages. It is 2-3 microns in size and has a kinetoplast. It rims the macrophages.

How well did you know this?
1
Not at all
2
3
4
5
Perfectly
56
Q

56- Which of the following is NOT associated with Pseudomonas aeruginosa?

A. Ecthyma gangrenosum
B. Rhinoscleroma
C. Green nail syndrome
D. Hot tub folliculitis
E. External otitis

A

Correct choice: B. Rhinoscleroma
Explanation: Rhinoscleroma is associated with Klebsiella pneumoniae, ssp. rhinoscleromatis.

How well did you know this?
1
Not at all
2
3
4
5
Perfectly
57
Q

57- The visceral form of the infection depicted in this image is associated with which three major protozoan species:

A. L donovani, L infantum, L chagasi
B. L major, L donovani, L infantum
C. L aethiopica, L donovani, L major
D. L brazilensis, L major, L infantum
E. L Mexicana, L brazilensis, L infantum

A

Correct choice: A. L donovani, L infantum, L chagasi
Explanation: Infections with L. donovani (e.g. India, Bangladesh and the Sudan), L. infantum (e.g. Europe, especially in the setting of HIV infection) and L.chagasi (e.g. Central and South America) are the major causes of visceral leishmaniasis

How well did you know this?
1
Not at all
2
3
4
5
Perfectly
58
Q

58- What is name of the possible reaction with treatment of this disease?

A. Tumor lysis syndrome
B. Jarisch-Herxheimer reaction
C. Asboe-Hansen
D. Rumple-Leede sign
E. Isomorphic phenomenon

A

Correct choice: B. Jarisch-Herxheimer reaction

Explanation: The Jarisch-Herxheimer reaction occurs with treatment of syphilis. It is due to TNF- alpha and other cytokines released when the spirochetes are phagocytosed. Tumor lysis syndrome occurs when chemotherapy is started in cancer patients. The isomorphic phenomenon is another name for the Koebner phenomenon. Rumple-Leede sign occurs after release of a tourniquet and there is a shower of petechiae. The Asboe-Hansen sign is when perpendicular pressure is applied to a blister and the blister enlarges. It is seen in pemphigus vulgaris and TEN.

How well did you know this?
1
Not at all
2
3
4
5
Perfectly
59
Q

59- A teenage patient with several weeks of fatigue, fever, and pharyngitis develops a morbilliform eruption after receiving amoxicillin. The causative organism of his systemic symptoms may cause all of the following diseases EXCEPT:

A. Infectious mononucleosis
B. Extranodal NK/T-cell lymphoma (nasal type)
C. Oral hairy leukoplakia
D. Non-sexually related acute genital ulcers (Lipschutz ulcers)
E. Progressive multifocal leukoencephalopathy

A

Correct choice: E. Progressive multifocal leukoencephalopathy

Explanation: The correct answer is E. The question describes the ampicillin/amoxicillin rash associated with infectious mononucleosis and EBV infection. EBV causes all of the following except for E; JC virus causes PML.

How well did you know this?
1
Not at all
2
3
4
5
Perfectly
60
Q

60- What is the vector for this disease?

A. Mosquito
B. Black fly
C. Tsetse fly
D. Sand fly
E. Mango fly

A

Correct choice: D. Sand fly

Explanation: The sand fly causes leishmania which is pictured here. The mango fly causes loa loa. The black fly causes onchocerciasis, the tsetse fly causes African trypanosomiasis.

How well did you know this?
1
Not at all
2
3
4
5
Perfectly
61
Q

61- A 45 year old septic female develops symmetric, large ecchymotic areas with irregular borders on the extremities, ears, and nose. Laboratory testing reveals the patient has disseminated intravascular coagulation. Which of the following is the most common underlying infection associated with this condition?

A. Group A streptococcus
B. Meningococcus
C. Staphylococcus
D. Pseudomonas
E. Mycobacterium

A

Correct choice: B. Meningococcus
Explanation: Purpura fulminans is an oftentimes dramatic presentation of large ecchymotic areas of the skin, favoring acral sites including the extremities, ears, and nose. The underlying pathophysiology is hemorrhagic infarction of the skin due to disseminated intravascular coagulation. Meningococcal infection is the leading underlying infection associated with sepsis and purpura fulminans. When Kids develop purpura fulminans after an infection the most common cause is group A Strep.

How well did you know this?
1
Not at all
2
3
4
5
Perfectly
62
Q

62- What is the causative organism?

A. Rubella
B. Parvovirus B19
C. Poxvirus
D. Paramyxovirus
E. Coxsackie

A

Correct choice: B. Parvovirus B19
Explanation: This is erythema infectiosum, with the slapped cheek appearance. It is due to parvovirus B19. Paramyxovrus causes the measles, molluscum is due to a poxvirus, orf is due to a parapoxvirus. Rubella is due to a togavirus. Coxsackie causes hand-foot-mouth disease and herpangina.

How well did you know this?
1
Not at all
2
3
4
5
Perfectly
63
Q

63- An infant presents to your pediatric dermatology clinic with acral vesiculopustules and intense itching. Confirmation of the diagnosis is achieved by light microscopic examination of a mineral oil preparation from a skin scraping (shown in photo). Which of the following statements is FALSE?

A. The mites that cause infestations in animals are not a source of human infestation

B. The incubation period before symptoms develop can range from days to months
C. Each day a female mite lays 3 eggs, which require approximately 10 days to mature
D. The mite is too small to be seen by the naked eye
E. The 30-day life cycle is completed within the dermis

A

Correct choice: E. The 30-day life cycle is completed within the dermis
Explanation: Human scabies is caused by the species-specific, eight-legged mite Sarcoptes scabiei var. hominis shown in the mineral oil prep in the above image. The 30-day life cycle is completed within the epidermis. Acral vesiculopustules can represent a clue to the diagnosis of scabies in infants. Confirmation of diagnosis can be achieved by light microscopic examination of mineral oil preparations of skin scrapings from infested areas for adult mite, eggs or fecal pellets. A scalpel or curette may be used to obtain the skin sample. Microscopic examination of transparent adhesive tape following its application to infested areas of skin represents another diagnostic technique. This image shows an adult female mite and is a light microscopic examination of a mineral oil preparation of a skin scraping in a patient with scabies.

1 – The specific Sarcoptes mite that causes infestation in animals is not a source of human infestation, but can produce bite reactions. 2 – The incubation period before symptoms of scabies develop can range from days to months. In first-time infestations it takes 2-6 weeks before the host’s immune system becomes sensitized to the mite or its by-products, resulting in pruritus and cutaneous lesions. 3 – Each day a female lays 3 eggs which require approximately 10 days to mature. Scabies mites usually live 3 days or fewer off a human host, but may live up to 7 days in patients with crusted scabies. 4 – The scabies mite is 0.35 x 0.3 mm in size and thus is too small to be seen by the naked eye.

How well did you know this?
1
Not at all
2
3
4
5
Perfectly
64
Q

64- If this patient were pregnant and allergic to penicillin what would be the appropriate treatment?

A. Treatment with penicillin and treatment of the allergic reaction
B. Desensitization to penicillin
C. Azithromycin
D. Ciprofloxacin
E. Gentamicin

A

Correct choice: B. Desensitization to penicillin
Explanation: This is a cutaneous gumma in late syphilis. The treatment for late syphilis is penicillin. If this patient was pregnant and allergic to penicillin the patient would need to undergo desensitization to penicillin and then be treated with penicillin

How well did you know this?
1
Not at all
2
3
4
5
Perfectly
65
Q

65- Which of the following is the best test to diagnose cutaneous herpes simplex virus infection?

A. Lesional viral culture
B. Lesional direct fluorescence antibody testing
C. Lesional polymerase chain reaction assay
D. Blood viral culture
E. Urine nucleic acid amplification test

A

Correct choice: C. Lesional polymerase chain reaction assay
Explanation: Recent recommendations from both the Infectious Diseases Society of America and the American Society for Microbiology characterize lesional nucleic acid amplification testing, such as PCR, as the “most sensitive, specific, and rapid test for diagnosis of HSV-associated skin or mucosal lesions.” Lesional DFA testing or lesional viral cultures may be used to diagnose cutaneous HSV infection, although these tests have lower sensitivity and/or specificity compared to PCR. Blood viral cultures are typically not helpful when trying to diagnose cutaneous HSV infection. Lastly, urine nucleic acid amplification tests can be used to diagnose certain sexually-trasmitted infections (e.g. Chlamydia, Gonorrhea), however they are not helpful for HSV.

How well did you know this?
1
Not at all
2
3
4
5
Perfectly
66
Q

66- This lesion develops how long after the infectious exposure?

A. 20 years
B. 10 days to 3 months
C. 10 years
D. 1-9 days
E. 4 to 6 months

A

Correct choice: B. 10 days to 3 months
Explanation: This is a perianal chancre of primary syphilis following anal intercourse. The chancre is non-tender and presents 10 days to 3 months after syphilis exposure.

How well did you know this?
1
Not at all
2
3
4
5
Perfectly
67
Q

67- This infectious lesion appeared very rapidly after surgery and started as a nodule that now looks like the photograph, what is the treatment of choice after drainage?

A. Clindamycin
B. Minocycline
C. Doxycycline
D. Clarithromycin
E. No treatment after drainage

A

Correct choice: D. Clarithromycin
Explanation: This is a mycobacterial infection after surgery which is usually due to M. Chelonei. It presents with SQ nodules that ulcerate and drain. The treatment is to drain and treat with clarithromycin as first-line antibiotic therapy.

How well did you know this?
1
Not at all
2
3
4
5
Perfectly
68
Q

68- A patient with any stage of this infection should always be tested for:

A. herpesvirus
B. hepatitis
C. Chlamydia trachomatis
D. Neisseria gonorrhea
E. HIV infection

A

Correct choice: E. HIV infection
Explanation: All patients who have syphilis should also be tested for HIV infection

How well did you know this?
1
Not at all
2
3
4
5
Perfectly
69
Q

This patient has less than three lesions which are
anesthetic and anhidrotic. What cytokine profile does this condition have?
A. Th2
B. IL-23
C. Th1

D. Th1 and Th2
E. Th17

A

Correct choice: C. Th1
Explanation: This is tuberculoid leprosy which classically has a low concentration of organisms because the body is reacting to the M. leprae. Thus the patients get anesthesia and anhidrosis. There are typically less than three hypopigmented patches. Tuberculoid leprosy is associated with a Th1 cytokine profile including IFN-gamma, IL-2 and IL-12. Lepromatous leprosy is Th2.

Tuberculoid leprosy can be either one large red patch with well-defined raised borders or a large hypopigmented asymmetrical spot. Lesions become dry and hairless. Loss of sensation may occur at the site of some lesions. Tender, thickened nerves with subsequent loss of function are common. Spontaneous resolution may occur in a few years or it may progress to borderline or rarely lepromatous types. Leprosy has very characteristic clinical features but the diagnosis must be confirmed because of the need for prolonged treatment with antibiotics. A skin biopsy may show characteristic histopathology with granulomas (mixed inflammatory cell infiltrate in the deeper layers of the skin, the dermis) and involvement of the nerves. Special staining of the tissue may show acid-fast bacilli, the number visible depending on the type of leprosy. Immunohistochemistry stains can be helpful when the bacilli are few in number.The bacteria may also be found in lepromatous leprosy on smears taken from skin slits made in the ear lobes, but the smears will be negative in the tuberculoid or borderline forms of the disease.
Lepromatous leprosy. Lepromatous leprosy is a form of leprosy characterized by pale macules in the skin. In lepromatous leprosy, TH2 response is turned on, and because of reciprocal inhibition (IL-4; IL-10), the cell-mediated response (TH1) is depressed.

How well did you know this?
1
Not at all
2
3
4
5
Perfectly
70
Q

70- Which of the following is true regarding sporotrichosis?

A. The disseminated form is common
B. Transmission usually occurs via direct inoculation
C. Sclerotic bodies are seen histologically
D. Supersaturated potassium iodide (SSKI) is the most common treatment
E. It is found in pigeon droppings

A

Correct choice: B. Transmission usually occurs via direct inoculation

Explanation: Sporotrichosis is caused by Sporothrix schenckii, a dimorphic fungus found in soil, thorns, moss, and bark. Transmission may occur via direct inoculation or inhalation, though it usually occurs when the fungus enters the skin through a small cut or scrape, usually after someone touches contaminated plant matter. Sporotrichosis has three different forms: lymphocutaneous, fixed cutaneous, and disseminated. The disseminated form is rare. While SSKI may be used to treat lymphocutaneous sporotrichosis, itraconazole is the most common treatment. Cryptococcus neoformans is found in pigeon droppings. Sclerotic bodies (aka Medlar bodies or copper pennies) are seen histologically in chromomycosis (aka chromoblastomycosis), whereas asteroid bodies are typically seen histologically in sporotrichosis.

How well did you know this?
1
Not at all
2
3
4
5
Perfectly
71
Q

71- What is the most likely organism?

A. Leishmania
B. Corynebacterium minutissimum
C. Trichophyton Rubrum
D. Hortaea werneckii
E. There is no organism, this is normal skin

A

Correct choice: C. Trichophyton Rubrum
Explanation: This is a dermatophyte infection located in the stratum corneum. Trichophyton Rubrum is a common cause of dermatophyte infection. Hortaea werneckii causes tinea nigra, which would show pigmented hyphae on histology. Leishmania presents with amastigotes (tiny dots) within macrophages (Leishman-Donovan bodies); the dermis contains an infiltrate of histiocytes, lymphocytes, and plasma cells. The organisms are best seen with the Giemsa stain.

How well did you know this?
1
Not at all
2
3
4
5
Perfectly
72
Q

72- What is the most likely cause of this eruption (buttocks tinea)?

A. Trichophyton tonsurans
B. Trichophyton mentagrophytes
C. Trichophyton rubrum
D. Microsporum canis
E. Microsporum gypseum

A

Correct choice: C. Trichophyton rubrum
Explanation: This is tinea corporis, which is most often due to T. rubrum. The remaining answer choices may also produce tinea corporis, but less commonly than T. rubrum.

How well did you know this?
1
Not at all
2
3
4
5
Perfectly
73
Q

73- Transmission of this eruption is primarily via which route?
A. Contact
B. Respiratory
C. Food borne
D. Bloodborne
E. Insect borne

A

Correct choice: B. Respiratory
Explanation: Varicella is primarily spread via respiratory droplets, and less frequently, direct contact. Varicella is not spread by bloodborne, insect borne, or food borne mechanisms.

How well did you know this?
1
Not at all
2
3
4
5
Perfectly
74
Q

74- A 28-year-old HIV-infected male presened with superficial angiomatous papules and nodules, violaceous lichenoid plaques, and deep subcutaneous nodules. The skin lesions resembles pyogenic granulomas and were smooth and pedunculated. In addition, the patient presented with nausea, vomiting, diarrhea, abdominal pain, hepatosplenomegaly, and elevated liver enzymes. Which of the following immunohistochemical stains would be useful in identifying this lesion?

A. Verfoeff-van Gieson
B. Alizarin red
C. Trichrome blue

D. Warthin-Starry
E. Sudan black

A

Correct choice: D. Warthin-Starry

Explanation: Biopsy specimens of bacillary angiomatosis reveal lobular proliferations of capillaries and venules, plump and protuberant endothelial cells, and a marked neutrophilic infiltrate with evidence of leukocytoclasis throughout the interstitium. In H&E-stained sections, interstitial clumps of bacilli appear as purplish granular material. Bacteria can be readily identified with a Warthin– Starry stain.
2- Trichrome stains collagen blue.
3- Verhoeff–Van Gieson stain is used to demonstrate normal or pathologic elastic fibers.
4- Sudan black stains lipids
5- Alzarin redis commonly used in histology to highlight calcium deposits by binding to it by a chelation process. The reaction enable the formation of alizarin red S-calcium, a birefringent compound which deposits appears optically in red.

How well did you know this?
1
Not at all
2
3
4
5
Perfectly
75
Q

75 -This HPV strain is responsible for Bushke-Lowenstein tumor and also for anogenital warts:

A. HPV 6, 11
B. HPV 1
C. HPV 2, 4
D. HPV 3, 10
E. HPV 7

A

►A

Anogenital and Buschke-Lowentein tumor is caused by HPV 6 and 11. HPV 1 is from palmoplantar warts. HPV 2, 4 causes common warts HPV 7 causes Butcher’s warts.

How well did you know this?
1
Not at all
2
3
4
5
Perfectly
76
Q

76 -The causative organism that is responsible for pitted keratolysis is:

A. Micrococcus sedentarius
B. Streptobacillus moniliformis
C. Klebsiella pneumonia
D. Neisseria meningitides
E. Burkholderia mallei

A

►A

Pitted keratolysis is caused by Micrococcus sedentarius. The clinical features are shallow 1 -3mm pits on plantar surface of feet. The treatment is topical erythromycin, clindamycin, or benzoyl peroxide.

How well did you know this?
1
Not at all
2
3
4
5
Perfectly
77
Q

77 -Most common causative agent for the lesion shown in this 5-year old child is

A. Staphylococcus aureus
B. group A β-hemolytic streptococci
C. Pseuomonas
D. kleibsiella
E. H.influenza

A

►A

The picture shown is Impetigo, which is a common, contagious superficial skin infection caused most commonly by staphylococci. Although seen in all age groups, the disease is most common in infants and children. Lesions may involve any body surface but occur most frequently on the
7 exposed parts of the body, especially the face, hands, neck, and extremities. There are two classic forms of impetigo, bullous and nonbullous. Nonbullous impetigo accounts for more than 70% of cases. Historically was caused primarily by group A β-hemolytic streptococci (GABHS), but now appears to be most commonly caused by S. aureus. Anaerobic organisms may also be recovered from lesions of nonbullous impetigo. (Reference: Brook I, Frazier EH, Yeager JK. Microbiology of nonbullous impetigo. Pediatr Dermatol 1997;14(3):192-195.)

How well did you know this?
1
Not at all
2
3
4
5
Perfectly
78
Q

78- A 4 year-old boy is diagnosed with Staphylococcal Scalded-Skin Syndrome. Vesiculation in this disease is secondary to exfoliative toxins binding what target protein?

A. Plectin
B. Bullous pemphigoid antigen 2
C. Bullous Pemphigoid antigen 1
D. Desmoglein 3
E. Desmoglein 1

A

►E

Staphylococcus Scalded-Skin Syndrome (SSSS) is caused by Staphylococcus exfoliative toxins A and B binding desmoglein 1. This results in the intraepidermal split at the granular layer seen in this illness. Targeting of desmoglein 3 would result in a suprabasilar split. Targeting of plectin, BPAG1, or BPAG2 would result in a subepidermal split. This is not seen in SSSS.

How well did you know this?
1
Not at all
2
3
4
5
Perfectly
79
Q

79- Tick-borne Relapsing Fever is caused by:

A. B. burgdorferi
B. B. recurrentis
C. B. duttonii
D. B. henselae

E. B. bacilliformis

A

►C

Relapsing Fever can be either Louse-borne or Tick-borne. Tick-borne Relapsing Fever is caused by
B. duttonii or B. hermsi, while Louse-borne Relapsing Fever is caused by B. recurrentis.

How well did you know this?
1
Not at all
2
3
4
5
Perfectly
80
Q

80- Staphylococcal scalded skin syndrome is caused by exfoliative toxins binding to:

A. Desmoglein-1
B. Desmokolin-1
C. TSST-1
D. Laminin-5
E. Desmoglein-3

A

►A

Staph scaled skin syndrome is caused by Et-A and ET-B that bind to desmoglein-1. Children under 5 years of age, adults with renal failure or immunodeficiency. Also a Nikolsky’s sign is seen in lesional and non-lesional skin with a subgranular blister.

How well did you know this?
1
Not at all
2
3
4
5
Perfectly
81
Q

81- Which human papillo virus (HPV) type is implicated in papillomatosis cutis carcinoides di Gottron?

A. 7
B. 11
C. 18
D. 57
E. 60

A

►B

Papillomatosis cutis carcinoides di Gottron, also known as Gottron’s tumor, is a verrucous carcinoma of the skin. In 1948, Ackerman first described verrucous carcinoma (VC), a low-grade tumor that generally is considered a clinicopathologic variant of squamous cell carcinoma. Verrucous carcinomas are caused by HPV types 6 and 11. When they occur on the feet they are called epithelioma caniculatum. When they occur on the genitals they are refered to as BuschkeLowenstein tumors. When they occur in the mouth they are called Ackerman’s tumors or oral florid papillomatosis.

How well did you know this?
1
Not at all
2
3
4
5
Perfectly
82
Q

82- Which of the following is a characteristic of papilloma viruses?

A. enveloped viruses
B. single stranded RNA viruses
C. replicate independent of the host cell
D. controlled by host antibody regulated response
E. can cause a condition that manifests with stridor and hoarseness in children

A

►E

Papilloma viruses are non-enveloped,double stranded DNA viruses. Cell mediated responses are primarily responsible for controlling papillomavirus infections. Unlike viruses such as HSV, HPV does not have enzymes required for replication of viral DNA, and is entirely dependent on the host cellular machinery. Recurrent respiratory papillomatosis can be caused by HPV -6 and 11, with verrucous lesions of the airways. It can occur as a juvenile or adult -onset form and present with hoarseness in children. Unlike viruses such as HSV, HPV does not enzymes 9 required for replication of viral DNA, and is entirely dependent on the host cellular machinery for these functions.

How well did you know this?
1
Not at all
2
3
4
5
Perfectly
83
Q

83- The dominant organism present on oily regions of the skin is:

A. Gram positive coagulase negative staphylococcus
B. Staphylococcus aureus
C. Propionibacterium acnes
D. Corynebacterium spp.
E. Propionibacterium avidum

A

►C

The most common organism on sebaceous regions of human skin is Propionibacterium acnes. This is an aerotolerant, anaerobic gram positive club-shaped diptheroid that lives in the depths of the sebaceous follicle. It has a potent lipase which cleaves sebaceous triglycerides into glycerol and free fatty acids. The free fatty acids lower the pH and are thought to retard the growth of pathogens such as S. aureus. Other residents of oily regions are: Malassezia furfur, S. epidermidis, Micrococcus luteus and other catalase positive aerobic cocci. S. aureus is a pathogen, and is not considered normal flora of any region of the skin. The other options are found on dry or moist regions of the skin.

How well did you know this?
1
Not at all
2
3
4
5
Perfectly
84
Q

84- Measles is caused by which type of virus?

A. Paramyxovirus
B. Togavirus
C. Picornavirus
D. Rhabdovirus
E. Parvovirus

A

►A

Measles is caused by an RNA virus known as paramyxovirus. The illness is characterized by high fever, cough, coryza, conjunctivitis, and Koplik’s spots. Viral exanthem characterized by erythematous macules and papules beginning on the face and spreading caudally 3-4 days after prodromal symptoms.

How well did you know this?
1
Not at all
2
3
4
5
Perfectly
85
Q

85- A 12 year old girl presents complaining of multiple warts on the fingers that have been recalcitrant to common over-the-counter treatments. What is the clearance rate of untreated common warts in children at 2 years after diagnosis?

A. 25-30%
B. 50-55%
C. 70-75%
D. 1%

E. 5-10%

A

►C

While warts can be very difficult and frustrating to treat, the spontaneous clearance rate at two years is actually quite high (70-75%).

How well did you know this?
1
Not at all
2
3
4
5
Perfectly
86
Q

86- A 28 year old sexually active female presents to clinic complaining of new growths of her labia majora. Examination reveals condyloma acuminatum, and biopsy with immunohistochemistry confirms the diagnosis of human papilloma virus infection. Condyloma accuminatum is most commonly caused by what HPV type?

A. 6 and 11
B. 16 and 18
C. 1 and 3
D. 31 and 33
E. 48 and 60

A

►A

While many subtypes of HPV are associated with genital warts, 6 and 11 are the most common. In general, potentially oncogenic HPV subtypes such as 16 and 18 cause sessile genital warts.

How well did you know this?
1
Not at all
2
3
4
5
Perfectly
87
Q

87- What is the most common subtype of molluscum contagiosum virus infection seen in HIV patients?
A. MCV-1
B. MCV-2
C. MCV-3
D. MCV-4
E. MCV-5

A

►B

The most common subtype of molluscum contagiosum virus is MCV-1. In HIV patients, however, MCV-2 is more common.

How well did you know this?
1
Not at all
2
3
4
5
Perfectly
88
Q

88- The treatment of choice for Loiasis is:

A. Ivermectin
B. Diethylcarbamazine (DEC)
C. Permethrin
D. Malathion
E. Chloramphenicol

A

►B
The recommended treatment for loiasis is Diethylcarbamazine (DEC).

How well did you know this?
1
Not at all
2
3
4
5
Perfectly
89
Q

89 -Congenital Varicella Syndrome occurs after maternal varicella infection during which stage of pregnancy?

A. First 20 weeks
B. Third trimester
C. 5 days before and 2 days after delivery
D. 20-24 weeks
E. None of these answers are correct

A

►A

Congenital Varicella Syndrome occurs after maternal varicella-zoster virus infection early in pregnancy (up to 20 weeks gestation).

How well did you know this?
1
Not at all
2
3
4
5
Perfectly
90
Q

90- The most common agent of cutaneous and furuncular myiasis in North America is:

A. Dermatobia hominis

B. Gasterophilus intestinalis
C. Wohlfahrtia magnifica
D. Hypoderma hominis
E. Cuterebra intestinalis

A

►A

Dermatobia hominis is most common agent of cutaneous and furuncular myiasis in North America, but Cuterebra spp. are also a frequent cause of furuncular myiasis in the US. Complications of myiasis are tetanus and secondary bacterial infection.

How well did you know this?
1
Not at all
2
3
4
5
Perfectly
91
Q

91- A 28 year-old man presents with fever, chills, arthralgias and the cutaneous lesions shown. The most likely diagnosis is:

A. Purpura fulminans
B. Rickettsialpox
C. Anthrax
D. Gonoccemia
E. Primary herpes simplex infection

A

►D

The image shows sparsely distributed hemorrhagic vesiculopustules with erythematous bases on an acral surface. These features are characteristic of gonococcemia.

How well did you know this?
1
Not at all
2
3
4
5
Perfectly
92
Q

92- A 48 year-old woman presents to the Emergency Room with a 2 day history of fever and a well demarcated erythematous plaque on her right cheek. What is the treatment of choice?

A. Prednisone
B. Intravenous penicillin
C. Cephalexin
D. Fluconazole
E. Potent topical corticosteroid

A

►B

The most likely diagnosis is Erysipelas which is caused by beta -hemolytic group A streptococcus. The treatment of choice is intravenous penicillin.

How well did you know this?
1
Not at all
2
3
4
5
Perfectly
93
Q

93- Orf virus infection is also known as:

A. Ecthyma
B. Ecthyma contagiosum
C. Echtyma gangrenosum
D. Pseudocowpox
E. Milker’s nodule

A

►B

Orf virus infection is also known as Ecthyma contagiosum. Ecthyma Gangrenosum is caused by Pseudomonas aeruginosa. Ecthyma is caused by Staphylococcal or streptococcal pyoderma. Pseudocowpox and Milker’s nodule are also known as paravaccinia and caused by udders of infected cows.

How well did you know this?
1
Not at all
2
3
4
5
Perfectly
94
Q

94- A patient develops herpes zoster in the V1 distribution. A few days after development of the classic vesicular rash, her family brings her to the emergency room because of new onset seizures. She had been complaining of confusion, headache, fatigue, and loss of appetite prior to the convulsions. What is the diagnosis?

A. Syndrome of inappropriate anti-diuretic hormone
B. Ramsay-Hunt syndrome
C. Delayed contralateral hemiparesis
D. Post-zoster neuropathy
E. Intracerebral vasculitis

A

►A

The question stem describes signs and symptoms of hyponatremia. The syndrome of inappropriate antidiuretic hormone secretion is a known, albeit rare, complication of varicella zoster virus infection. The other four answer choices are also known complications of varicellazoster virus infection, but do not cause the disease presentation as described.

How well did you know this?
1
Not at all
2
3
4
5
Perfectly
95
Q

95- What laboratory finding is commonly seen in children with measles virus infection?

A. Lymphopenia
B. Thrombocytosis
C. Polycythemia
D. Anemia
E. Lymphocytosis

A

►A

Children who are actively infected with the measles virus are commonly found to have lymphopenia. While an abnormality of any of the blood cell lines is possible during viral infections, in the case of measles infections, lymphopenia is most common. However, cbc may reveal a relative lymphocytosis and thrombocytopenia.

How well did you know this?
1
Not at all
2
3
4
5
Perfectly
96
Q

96- The vector for relapsing fever that causes paroxysmal fevers, headaches, mylagias and erythematous or petechia macules in refugees is:

A. Pediculus humanus
B. Ornithodoros
C. Ixodes dammini
D. Ixodes pacificus
E. Ixodes ricinus

A

►A

Relapsing fever is caused by the human body louse Pediculus humanus and the borrelia species is
B. recurrentis. The clinical features are paroxysmal fevers, headache, lymphocytoma, mylagias,

erythematous or petechial macules on trunk and extremities.Louse-borne relapsing fever is transmitted by the human body louse and is generally restricted to refugee settings in developing regions of the world.Tick-borne relapsing fever has the same symptoms occurs in the
western United States and is usually linked to sleeping in rustic, rodent-infested cabins in mountainous areas. The vector for tick borne is ornithodoros and it is due to Borrellia duttonii.

How well did you know this?
1
Not at all
2
3
4
5
Perfectly
97
Q

97- A 19 year old sexually active male presents with a painful ulcer on the shaft of his penis. A biopsy specimen stained with Giemsa reveals a “school of fish.” Which of the following is the causative organism?

A. Hemophilus ducreyi
B. Treponema pallidum
C. Calymmatobacterium granulomatis
D. Chlamydia trachomatis L1
E. Herpes simplex virus

A

►A

Chancroid is a sexually transmitted disease caused by the bacteria Hemophilus ducreyi. It is one of many organisms that can cause a genital ulcer. Chancroid is characterized by a soft, painful chancre with ragged edges. Giemsa stained sections reveal a “school of fish” configuration which is typical. Treatment is with azithromycin, ceftriaxone, ciprofloxacin, or erythromycin.

How well did you know this?
1
Not at all
2
3
4
5
Perfectly
98
Q

98- Epithelioma cuniculatum is an HPV-associated verrucous carcinoma involving the:

A. Oral mucosa
B. Scalp
C. Sole
D. Penis
E. Trunk

A

►C

Epithelioma cuniculatum is a form of verrucous carcinoma involving the sole.

How well did you know this?
1
Not at all
2
3
4
5
Perfectly
99
Q

A patient had a severe febrile illness with hemolytic anemia and now has exophytic nodules on the trunk, extremities and face. Which of the following organisms is the cause?

A. Bartonella henselae
B. Bartonella quintana
C. Bartonella bacilliformis
D. Rickettsia rickettsii
E. Rickettsia akari

A

►C

The first stage of this condition is known as Carrion‟s disease, which can be fatal. It was in Dr. Carrion (as a medical student), who self inoculated himself to prove that the cause of verruga peruana (the second stage described above) was the same as that of the first stage. The cause of both Carrion‟s disease and verruga peruana is Bartonella bacilliformis. B. henselae is the cause of Cat Scratch disease while B. quintana causes Trench Fever. Rickettsia rickettsii is the cause of Rocky Mountain Spotted Fever and Rickettsia akari causes Rickettsial pox.

How well did you know this?
1
Not at all
2
3
4
5
Perfectly
100
Q

The diagnosis of leishmaniasis can be confirmed by which of the following techniques?

A. Weil-Felix test
B. Warthin-Starry stain
C. Culture in Sabourad’s agar
D. Culture in Novy-MacNeal-Nicolle (NNN) medium
E. Culture in Michel’s medium

A

►D

Confirmation of infection with Leishmaniasis is performed by culture in Novy-MacNeal-Nicolle (NNN) medium.

How well did you know this?
1
Not at all
2
3
4
5
Perfectly
101
Q

101- What is the most common presentation of mucous membrane involvement of EBV infection?

A. Buccal ulcers with superficial slough
B. Erosions of the gingivae
C. Plication of the tongue
D. Palatal petechiae
E. Oral hairy leukoplakia

A

►D

The most common presentation of intra-oral involvement of EBV infection is pinpoint petechiae of the junction of the hard and soft palate. The associated eponym is Forchheimer spots. Forchheimer spots are not pathognomonic for EBV infection, as they can be seen in measles, rubella, and scarlet fever. While oral hairy leukoplakia is associated with EBV infection, it is not the most common mucous membrane sign.

How well did you know this?
1
Not at all
2
3
4
5
Perfectly
102
Q

102- A 4 year old girl experiences systemic symptoms including fever, fatigue, headache, and vomiting. An associated exanthem consists of a non-pruritic eruption including the formation of blisters on the palms and soles and an intra-oral ulcer. Hand-foot-mouth syndrome is diagnosed. What nail finding can be seen approximately 1 month after infection with Coxsackie virus and other Enteroviruses?

A. Onychomadesis
B. Onycholysis
C. Trachyonychia
D. Terry’s nails
E. Paronychia

A

►A

Although rare, onychomadesis has been associated with enteroviral infections, including handfoot- mouth syndrome due to Coxsackie virus. Onychomadesis is an idiopathic shedding of the nails beginning at the proximal end. It may be due to temporary arrest of the matrix during the infection. In most cases, the nails return normally.

How well did you know this?
1
Not at all
2
3
4
5
Perfectly
103
Q

103 -A 1 year old female patient is brought in by her mother because of the development of warty growths peri-anally. A clinical examination and biopsy confirms the diagnosis of peri-anal warts. In 11 month old patients, what is the most common cause of acquisition of human papilloma virus.

A. Vertical transmission peri-natally
B. Sexual abuse
C. Contact with a wart-containing finger
D. Contact with a fomite
E. Aerosol transmission

A

►A

In any child with peri-anal warts, the possibility of sexual contact must be excluded. Fortunately, in infants up to 12 months of age, the most common route of acquisition of the human papilloma virus is vertical transmission at the time of delivery.

How well did you know this?
1
Not at all
2
3
4
5
Perfectly
104
Q
  • Epidermodysplasia verruciformis is caused by which human papilloma virus (HPV) types?

A. 6 and 11
B. 16 and 18
C. 5, 8, and 9
D. 3 and 10
E. 13 and 32

A

►C

Epidermodysplasia verruciformis is caused by HPV types 5, 8, 9, 12, 14, 15, 17, 19-26.

How well did you know this?
1
Not at all
2
3
4
5
Perfectly
105
Q

105 -A child presents with fever, cutaneous tenderness and erythema of flexural and periorifacial areas. Within 24 hours the erythema progresses to flaccid blisters and erosions. No organisms are cultured and Nikolsky’s sign is positive. First line therapy includes:

A. IV penicillinase resistant penicillin with clindamycin

B. Aspirin
C. IV IgG
D. PO tetracylcine
E. PO cephalexin

A

►A

First line therapy for SSSS includes IV penicillinase resistant penicillin. Second line therapy is IV macrolide treatment. Third line therapies are cephalosporin and vancomycin. Clindamycin is added to curtail toxin production.

How well did you know this?
1
Not at all
2
3
4
5
Perfectly
106
Q

106- Name the syndrome defined by gonorrhea with perihepatitis and liver adhesions.

A. Lhermitte-Duclos Syndrome
B. Turcot Syndrome
C. Fitz-Hugh-Curtis Syndrome
D. Leser-Trelat Syndrome
E. Clutton’s Syndrome

A

►C

Gonorrhea, caused by a gram negative intracellular diplococcus, often presents cu taneously with hemorrhagic vesiculopustules of the distal extremities. Septic arthritis is also an important presentation of gonococcemia, most commonly involving the knee.

How well did you know this?
1
Not at all
2
3
4
5
Perfectly
107
Q

107- A 48-year-old Latin American female with a diagnosis of leprosy develops bullous lesions that appear rapidly and ulcerate, especially below the knee. This most likely represents:
A. Reversal reaction
B. Upgrading reaction
C. Erythema nodosum leprosum
D. Lucio’s phenomenon
E. Lofgren’s syndrome

A

►D

This patient has a type III reactional state to leprosy called Lucio’s phenomenon. It mostly occurs in people of Latin American descent and is characterized by rapidly ulcerating and bullous lesions usually on the legs. It has a high mortality rate. A type I reaction, or a reversal/upgrading reaction, can occur after treatment and are associated with a change in cell-mediated immunity and result in accelerated destruction of bacilli. This is manifested as inflammation of existing lesions with no systemic symptoms. A type II reaction, or erythema nodosum leprosum, represents an immune complex deposition and features widespread erythematous nodules with systemic features. Lofgren’s is associated with sarcoidosis.

How well did you know this?
1
Not at all
2
3
4
5
Perfectly
108
Q

108- An AIDS patient develops a smooth papule with a collarette of scale with vascular proliferation seen on biopsy and is seen with those in contract with cats. The most likely cause of this is:

A. Bartonella hensalae
B. Brucellosis hensale
C. Pseudomonas mallei
D. Bacillus anthracis
E. Pasteurella

A

►A

This is bacillary angiomatosis and is caused by Bartonella hensalae and can be associated with contact with cats in 20% of cases and occurs in patients with AIDS. The disease is treated with a macrolide antibiotic or doxycycline.

How well did you know this?
1
Not at all
2
3
4
5
Perfectly
109
Q

109- A 27 year-old HIV positive presents with this pruritic eruption? What is the characteristic cell type found on histopathology?

A. Neutrophil
B. Plasma Cell
C. Basophils
D. Eosinophil

E. Lymphocyte

A

►D

Eosinophilic pustular folliculitis is a pruritic disorder that may be seen in HIV positive patients. Typically, the CD4 count is < 300. In patients with HIV, pruritus tends to improve after institution of anti-retroviral therapy.

How well did you know this?
1
Not at all
2
3
4
5
Perfectly
110
Q

110- A 43 year-old man with uncontrolled HIV disease presents with yellowish, red papules, draining sinuses and ulcers perianally and periurethrally. Which histopathological or immunohistochemical stain may aid in diagnosis?

A. CD3
B. CD20
C. Von Gieson (EVG)
D. Von Kossa
E. Steiner

A

►D

The most likely diagnosis is malacoplakia. Malacoplakia, an infection usually attributed to S. aureus, P. aeruginosa, or E. coli is characterized histopathologically by Michelis -Gutmann bodies. These are foamy eosinophilic macrophages containing calcified, laminated, intracytoplasmic bodies. They can be highlighted by a calcium stain such as von Kossa. CD3 and CD20 are lymphocyte markers, von Gieson is an elastic tissue stain, and Steiner stain is used to highlight spirochetes. These would not aid in the diagnosis of malacoplakia.

How well did you know this?
1
Not at all
2
3
4
5
Perfectly
111
Q
  1. -The main deterrent to staphylococcal infection of the skin is:

A. Complement
B. Neutrophils
C. Macrophages
D. Immunoglobulin
E. CD8+ T-cell mediated immunity

A

►B

The main deterrent to staphylococcal infection is the neutrophils. These contain bactericidal chemicals and proteins such as myeloperoxidase-H202-halide, lysozyem, serine proteases, defensins and bacterial permeability increasing peptide. Complement serves as an effective opsonin for phagocytosis by circulating neutrophils and macrophages. Immunoglobulin response hastens phagocytosis but is not absolutely required. Cytotoxic T-cells do not play a major role in controlling
S. aureus infections.

How well did you know this?
1
Not at all
2
3
4
5
Perfectly
112
Q

112- What named nerve is involved in the formation of Hutchinson’s sign (vesicles at the nasal tip) in herpes zoster?

A. Ophthalmic nerve
B. V2 (maxillary)
C. Nasociliary
D. Infratrochlear nerve
E. Labial nerve

A

►C

Hutchinson’s sign is defined by vesicles at the nasal tip in a patient with facial herpes zoster. It is caused by VZV invovlement of the nasociliary nerve. Patients suffering with this sign should be screened for ophthalmologic herpes zoster invovlement.

How well did you know this?
1
Not at all
2
3
4
5
Perfectly
113
Q

113- Cat scratch disease is from transmission from infected cats to humans and the treatment for the disease is:

A. Spontaneous resolution in the majority of cases
B. Erythromycin
C. Doxycycline
D. Cloramphenicol
E. Minocycline

A

►A

Cat scratch disease is caused by B. henselae. It is from the flea-Ctenocephalides felis. It spontaneous resolution in most of cases.

How well did you know this?
1
Not at all
2
3
4
5
Perfectly
114
Q
  1. A- goatherder develops this painless pustule that subsequently forms a black eschar. What causes the edema to form in this lesion?

A. Increased levels of cAMP
B. Release of TNF-alpha
C. Release of IL-1beta
D. Inhibition of phagocytosis
E. C1 esterase inhibitor

A

►A

Anthrax is caused by Bacillus anthracis, a gram positive spore-forming rod. Anthrax is primarily seen in individuals who are in contact with wild or domestic animals. Recent cases have been linked with bioterrorism. B. anthracis produces edema toxin and lethal toxin. Edema toxin is comprised of edema factor and protective antigen. Edema factor is a calmodulin dependent adenyl cyclase. The increased cAMP induces the gelatinous edema of anthrax skin lesions.

How well did you know this?
1
Not at all
2
3
4
5
Perfectly
115
Q

115- Which of the following vectors is responsible for transmitting Chagas‟ disease?

A. Tstse fly
B. Reduviid bug
C. Black fly
D. Mosquito
E. Sandfly

A

►B

The reduviid big is the vector of American trypanosomiasis (Chagas disease). The vector of African trypanosomiasis is the tstse fly.

How well did you know this?
1
Not at all
2
3
4
5
Perfectly
116
Q

116- What is the causative agent of this parasitic eruption which also causes patchy pulmonary infiltrate and eosinophilia?

A. Dracunculiasis
B. Gnathostomiasis
C. Ground itch
D. Larva migrans
E. Larva currens

A

►C

Larva migrans is caused by Ancylostoma braziliense, a hookworm of cats and dogs. It does not leave the epidermis because it lacks collagenase. Larva currens is due to strongyloides and presents with serpiginous lesions on the buttock, thighs or perianally. Ground itch is due to ancylostoma duodenale and necator americanus which clinically present like cutaneous larva migrans but are able to leave the epidermis, therefore causing fuplomany infiltrates and eosiniophilia. Loeffler’s syndrome, is the name for the patchy infiltrate of the lungs with eosinophilia. Treatment for this parasitic infection is ivermectin or albendazole or thiabendazole.

How well did you know this?
1
Not at all
2
3
4
5
Perfectly
117
Q

117- Treatment of severe nodulocystic acne with which of the following vitamin derivatives may completely arrest the disease process through decreasing P. acnes?

A. Vitamin D
B. Vitamin A
C. Vitamin K
D. Vitamin E
E. Vitamin B3

A

►B

13-cis-retinoic acid (isotretinoin/Accutane) is a derivative of vitamin A. It has several mechanisms of action in acne. It rapidly suppresses sebum production, causing a decrease in P. acnes populations. It also decreases follicular plugging by an uncertain mechanism. Vitamins D, K and E are fat soluble vitamins, but are not able to completely arrest the disease process in acne. Vitamin B3 is niacin and is associated with a diet of exclusively of corn, millet or sorghum. Other sources: carcinoid, Hartnup disease, parasites, GI disorders, IV nutrition, and psychiatric disease. Findings include the 3 D‟s: Diarrhea, Dementia, Dermatitis.

How well did you know this?
1
Not at all
2
3
4
5
Perfectly
118
Q

118- The Jarisch-Herxeimer reaction is most closely associated with the release of which of the following cytokines?

A. IFN-alpha
B. IL-12
C. TNF-alpha
D. IL-4
E. IL-10

A

►C

TNF-alpha. The Jarisch-Herxheimer reaction is associated with increased plasma concentrations of TNF-alpha after antibiotic treatment of syphilis. The Jarisch-Herxheimer reaction has also been described following treatment of louse-borne relapsing fever, lyme disease and leptospirosis.

How well did you know this?
1
Not at all
2
3
4
5
Perfectly
119
Q

119- A patient with HIV/AIDS presents with vesicles in a disseminated distribution. A biopsy is performed. Which histologic finding favors infection with varicella-zoster virus over herpes simplex virus?

A. Pseudoepitheliomatous hyperplasia
B. Margination of keratinocyte nuclei
C. Multinucleated cells
D. Superficial dermal leukocytoclastic vasculitis
E. Molding of nuclei

A

►D

Herpes simplex virus and disseminated varicella-zoster virus infection can be difficult to distinguish clinically and histologically. These viruses share the histologic findings of margination of nuclei, multinucleated cells, and nuclear molding. The presence of leukocytoclastic vasculitis favors a diagnosis of disseminated varicella-zoster virus.

How well did you know this?
1
Not at all
2
3
4
5
Perfectly
120
Q

120- Woolsorter‟s disease is caused by:
A. Erysipelothrix rhusiopathiae
B. Listeria monocytogenes
C. Corynebacterium minutissimum
D. Corynebacterium tenius
E. Bacillus anthracis

A

►E

Anthrax, also known as woolsorter‟s disease or malignant pustule is caused by Bacillus anthracis, a gram positive rod. It is primarily caused by contact with infected wild or domestic animals or their products (such as wool). This organism has 3 clinical forms: inhalation, gastrointestinal and cutaneous. The primary lesion is a “malignant pustule” which begins as a painless papule that evolves into a hemorrhagic bulla. Ultimately, a black eschar forms. The lack of pain is helpful in distinguishing between anthrax and other infections of the skin which are painful, such as ecthyma. The other listed organisms are not causes of this disease.

How well did you know this?
1
Not at all
2
3
4
5
Perfectly
121
Q

121- A patient with known chronic Hepatitis C virus infection presents with hyperkeratotic plaques of the bilateral feet. Biopsy reveals psoriasiform changes with superficial epidermal necrosis. Diet supplementation with what mineral has been shown to improve the appearance and symptoms of this cutaneous finding?

A. Zinc
B. Biotin
C. Magnesium
D. Calcium
E. Chromium

A

►A

Necrolytic acral erythema is characterized by hyperkeratotic well-defined plaques on the lower extremities, seen occasionally in patients with hepatitis C virus infection. Treatment consists primarily of treating the underlying infection. Diet supplementation with zinc has been shown to improve necrolytic acral erythema, even in the presence of normal serum zinc levels.

How well did you know this?
1
Not at all
2
3
4
5
Perfectly
122
Q

122 -A patient is experiencing posterior cervical lymphadenopathy and has been diagnosed with African trypanosomiasis. This sign is known as:

A. Winterbottom’s sign
B. Hutchinson sign
C. Romana’s sign
D. Sleeping sickness sign
E. Tsetse sign

A

►A

This patient has African trypanosomiasis. It is caused by T. brucei gambiense and T. brucei rhodesiense and the vector is the Tsetse fly. Posterior cervical lymphadenopathy is known as winterbottom’s sign. Romana’s sign is eyelid edema and conjunctivitis at the site of inoculation for American trypanosomiasis.

How well did you know this?
1
Not at all
2
3
4
5
Perfectly
123
Q

123- Which of the following is true regarding lymphogranuloma venereum?

A. The primary stage presents as a painful herpetiform ulcer.
B. Proctocolitis can be seen in the late stages of this disease.
C. It is caused by Chlamydia psittaci.
D. Buboes should be incised and drained.
E. Transmission does not occur via skin-to-skin contact.

A

►B

Lymphogranuloma venereum presents as a painless ulcer and is caused by Chlamydia trachomatis serotypes L1, L2, L3. While buboes can be aspirated, they should not be incised and drained. Transmission occurs through sexual contact, but can occur via skin-to-skin contact. Complications of late stage disease include proctocolitis, perirectal abscesses, rectovaginal fistulas, and anal fistulas.

How well did you know this?
1
Not at all
2
3
4
5
Perfectly
124
Q

124- Which of the following tests is the most sensitive serologic test in late primary syphilis?
A. FTA-ABS
B. VDRL
C. ELISA
D. RPR
E. MHA-TP

A

►A

The fluorescent treponemal antibody absorption test is the most sensitive serologic test in primary syphilis. The ELISA is the test of choice in early primary and congenital syphilis because it is the first to become reactive. However, it is less sensitive in late primary syphilis because of reduction of IgM production. The ELISA is 100% specific. MHA-TP is less sensitive than FTA-ABS during primary syphilis.
Syphilis can be detected during the early infectious stages by taking swabs of the chancre for polymerase chain reaction (PCR). Dark-field microscopy examination of tissue or tissue fluid from a primary ulcer (chancre) or condylomata lata is now less commonly undertaken than in the past. Blood for serological tests is necessary for diagnosis. Serological tests turn positive about 5 to 6 weeks after acquiring the infection. In some cases, cerebrospinal fluid can be tested to confirm neurosyphilis.
Non-specific non-Treponemal tests eg Rapid plasma reagin (RPR), Venereal disease research laboratory (VDRL) May cross-react resulting in low-level false-positive tests during pregnancy, other infections, drug abuse, connective tissue disease and aging. Levels usually relate to disease activity and are used for monitoring treatment. After effective treatment of syphilis, these tests usually become negative but in some people, may remain positive at low levels
Specific anti-treponemal antibody tests eg Treponenam pallidum particle agglutination assay (TPPA), enzyme immunoassays (EIA or AIA), fluorescent treponemal antibody absorption (FTA- ABS), microhaemagglutination assay (MHA-TP) and Western Blot (WB). These detect antibody due to past or present infection with T. pallidum or another Treponema species (eg yaws or pinta) They cannot distinguish between different types of Treponemal infection eg yaws or syphilis

or the duration of infection. Most people with reactive treponemal tests will continue to have reactive tests for the remainder of their lives, regardless of treatment or disease activity. An EIA test is sometimes used as a screening test, as it avoids the false positives found when RPR or VDRL are used.Skin biopsy may or may not show characteristic histopathological features. Molecular testing using polymerase chain reaction (PCR) can confirm the presence of genetic material from the bacteria in ulcers, on the skin surface, in blood, lymph nodes and other tissue.

Reference: Forrestel AK, Kovarik CL, Katz KA. Sexually acquired syphilis: Laboratory diagnosis, management, and prevention. J Am Acad Dermatol. 2020 Jan;82(1):17-28. doi: 10.1016/ j.jaad.2019.02.074. Epub 2019 Apr 12. Review. PMID: 30986474

How well did you know this?
1
Not at all
2
3
4
5
Perfectly
125
Q

125- A 16 year old female presents with a sore throat, painful cervical lymphadenopathy, and an enlarged spleen. An empiric trial of amoxicillin leads to the development of a diffuse morbilliform rash. What is the incubation period of the most likely viral cause of this patient’s symptoms?

A. 3-7 days
B. 7-10 days
C. 2-3 weeks
D. 3-7 weeks
E. 6 months

A

►D

Infectious mononucleosis is characterized by a sore throat, painful lymphadenopathy, fatigue, and splenomegaly. The formation of a morbilliform rash upon challenge with ampicillin or related antibiotics is a classic finding. The most common virus associated with infectious mononucleosis is Epstein-Barr virus. The incubation period is relatively long, 3-7 weeks.

How well did you know this?
1
Not at all
2
3
4
5
Perfectly
126
Q

126- A 39 year old male goatherder presents with a necrotic eschar which is found to be caused by cutaneous anthrax. Bacillus anthracis produces edema factor, which causes gelatinous edema of skin by lesions by increasing what?

A. Cyclic adenosine monophosphate
B. Tumor necrosis factor alpha

C. Interleukin-1 beta
D. Transforming growth factor
E. Endothelial growth factor

A

►A

Edema factor is a portion of the edema toxin, one of two exotoxins secreted by Bacillus anthracis. Edema factor causes gelatinous edema of anthrax skin lesions by inducing an increase in cyclic adenosine monophophate levels. Lethal toxin, the other exotoxin, works by releasing tumor necrosis factor-alpha and interleukin-1 beta.

How well did you know this?
1
Not at all
2
3
4
5
Perfectly
127
Q

127- A 20-month old child develops a high fever followed 2 days later by a sudden eruption of rose pink macules and papules with white halos as the fever subsides. What is the most likely diagnosis?

A. Measles
B. Rubella
C. Scarlet fever
D. Exanthem Subitum
E. Erythema infectiosum

A

►D

The cutaneous features of Exanthem Subitum (Roseola infantum, Sixth Disease)as described in the question develop as the fever subsides. While the other diagnoses are part of the differential diagnosis of “fever and rash” in children, only roseola infantum has this characteristic clinical course.

How well did you know this?
1
Not at all
2
3
4
5
Perfectly
128
Q

128- A 69 year old man who had recently returned from a boating trip in the Atlantic developed pain with hemorrhagic bullae of his right leg. He had substained a cut on his right leg prior to leaving for the trip, but swam in the water anyway. Which of the following treatments should be started?

A. Doxycycline
B. Erythromycin

C. Penicillin
D. Rifampin
E. Dapsone

A

►A

The case presentation describes an infection with Vibrio vulnificus, a Gram-negative rod, which may lead to a rapidly expanding cellulitis, with hemorrhagic bullae and necrosis, and lifethreatening septicemia. This infection mainly occurs along the Atlantic seacoast. It may be acquired after ingesting raw oysters or other seafood. Alternatively, localized skin infection may result after exposure of an open wound to seawater. Doxycycline together with ceftazidime is the treatment of choice. In patients with preexisting hepatic dysfunction or immunocompromise and whose wounds are exposed to or acquired in saltwater, prophylactic antibiotic coverage with doxycycline, 100 mg every 12 h, and cleansing with 0.025% sodium hypochlorite solution may prevent progressive infection.

How well did you know this?
1
Not at all
2
3
4
5
Perfectly
129
Q

129- Which of the following is not a tick born disease?

A. Lyme disease
B. Tularemia
C. Rocky Mountain spotted fever
D. Yaws
E. Ehrlichiosis

A

►D

Ticks are capable of transmitting:Lyme disease-Borrelia burgdorferi, a tickborne spirochete. Tularmia-Francisella tularensis, bite of a tick. Rocky Mountain spotted fever-Rickettsia rickettsii, tick typhus. Yaws-Treponema Palladium, subspecies pertunue-transmitted nonsexually, by contact with infectious lesions. Ehrlichiosis-Ehrlichia chaffeensis, tickborne

How well did you know this?
1
Not at all
2
3
4
5
Perfectly
130
Q

130 -Herpetic whitlow is usually a localized infection to the index finger. The most common cause of Herpetic whitlow is from:

A. HSV2
B. HSV1
C. EBV
D. CMV
E. VZV

A

►A

Herpetic whitlow is more commonly caused by HSV2 from digital/genital contact. It can also occur in dentist.

How well did you know this?
1
Not at all
2
3
4
5
Perfectly
131
Q

131- A patient with atopic dermatitis presents with frequent flares of dermatitis. They have multiple lichenified plaques, some with serum crust. Which of the following organisms is most likely colonizing these areas?

A. Aerobic diphtheroids
B. Gram positive coagulase negative cocci
C. Staphylococcus epidermidis
D. Staphylococcus aureus
E. Propionibacterium avidum

A

►D

S. aureus rapidly becomes a member of the cutaneous flora in locations where there is serum or plasma on the skin. Often patients with atopic dermatitis will have colonization of this organism in lesions and may benefit from topical or oral antibiotics. P. avidum and aerobic diphtheroids are present in moist regions of the skin. Gram positive coagulase negative cocci are the dominant organism in the dry regions of the body. S. epidermidis is most commonly found in oily body regions.

How well did you know this?
1
Not at all
2
3
4
5
Perfectly
132
Q

132- Focal epithelial hyperplasia (Heck‟s Disease) is associated with which of the following human papillomavirus types?

A. 3
B. 4
C. 7
D. 10
E. 13

A

►E

Focal epithelial hyperplasia is caused by HPV-13 and 32.

How well did you know this?
1
Not at all
2
3
4
5
Perfectly
133
Q

133 -A 23 year old intravenous drug user presents with the onset of purpura on the legs. Biopsy reveals vasculopathic changes with intravascular thrombi seen in superficial dermal vessels. Serum studies for cryoglobulins and Hepatitis C virus are both positive. Patients with HCV and mixed cryoglobulinemia are more likely to develop what malignancy?

A. Non-Hodgkins lymphoma
B. Cutaneous T-cell lymphoma
C. Anaplastic large cell lymphoma
D. Kaposi’s sarcoma
E. Acute myelocytic leukemia

A

►A

Mixed cryoglobulinemia is characterized by LCV and the finding of monoclonal or polyclonal IgG and IgM cryoglobulins in the blood. Mixed cryoglobulinemia is commonly associated with HCV infection. Patients with HCV-associated mixed cryoglobulinemia are at an increased risk of developing non-Hodgkins lymphoma, approximately 35 times the general population.

How well did you know this?
1
Not at all
2
3
4
5
Perfectly
134
Q

134- Cat scratch disease is caused by infection with:

A. Bartonella species
B. Gram-positive cocci
C. Gram-positive bacilli
D. Rickettsiae
E. Spirochetes

A

►A

Cat scratch disease is caused by infection with B. henselae - a gram-negative bacillus of the genus Bartonella.

How well did you know this?
1
Not at all
2
3
4
5
Perfectly
135
Q

135- Guarnieri bodies are associated with:

A. Measles
B. Orf
C. CMV
D. Smallpox
E. Anthrax

A

►D

Guarnieri bodies are cytoplasmic eosinophillic inclusions found on light microscopy.

How well did you know this?
1
Not at all
2
3
4
5
Perfectly
136
Q

136 -A patient from India presents with fever, hepatosplenomegaly, and darkening of the skin. Which of the following is a cause of visceral leishmaniasis?

A. L. donovani
B. L. major
C. L. mexicana
D. L. aethiopica
E. L. tropicana

A

►A

Visceral leishmaniasis is the most severe form of leishmaniasis. Complications can be fatal, and include pneumonia, tuberculosis, and dysentery, which are potential opportunistic infections. Visceral leishmaniasis is most commonly caused by L. donovani and L. infantum.

How well did you know this?
1
Not at all
2
3
4
5
Perfectly
137
Q

137- Which of the following factors is most important in the skin providing protection against infectious agents?

A. The mechanical barrier of intact skin
B. The resident flora present on intact skin
C. The relative dryness of intact skin
D. The cornified cell envelope
E. The presence of immunoglobulins on intact skin

A

►A

The most important determinant of the skin‟s protection against infectious agents is the mechanical barrier of intact skin. The dryness of the skin is also important in retarding infection. The resident flora may help prevent colonization by pathogenic organisms, but is not the most important factor. Immunoglobulins are present on intact skin in miniscule amounts. The cornified cell envelope is not a major determinant of infection prevention.

How well did you know this?
1
Not at all
2
3
4
5
Perfectly
138
Q

138- Which exotoxin(s) are involved in mediating the effects of Bacillus anthracis - the causative organism for anthrax?

A. Exotoxin B
B. Edema Toxin
C. Alpha toxin
D. Lethal Toxin
E. Edema Toxin and Lethal Toxin

A

►E

B. anthracis has 2 exotoxins: edema toxin and lethal toxin, each comprised of a pair of noncovalently linked proteins: “ Edema toxin = Edema Factor (EF) + Protective Antigen (PA) “ Lethal toxin = Lethal Factor (LF) + PA “ EF causes gelatinous edema of anthrax skin lesions by inducing an increase in cyclic adenosine monophosphate (cAMP) levels “ LF causes shock and death in disseminated anthrax via the release of tumor necrosis factor-alpha (TNF-a)and interleukin-1b (IL-1b) “ PA is responsible for entry of exotoxins into the cell by receptormediated endocytosis.

How well did you know this?
1
Not at all
2
3
4
5
Perfectly
139
Q

139- A rabbit farmer presents with fevers, myalgia, and a rash. A serum study reveals antibodies to Francisella tularensis. Which of the following is the most common form of tularemia?

A. Ulceroglandular
B. Chancriform
C. Oculoglandular
D. Typhoidal
E. Meningeal

A

►A

Tularemia represents a wide range of clinical syndromes caused by Francisella tularensis. The bacterium is most commonly acquired through contact with infected animals, including rabbits. There are a wide variety of variants, with the ulceroglandular form being most common. Ulceroglandular tularemia is characterized by a chancre-like ulcer with raised borders and regional lymphadenopathy, most commonly on the finger or hand. Treatment is with streptomycin.

How well did you know this?
1
Not at all
2
3
4
5
Perfectly
140
Q

140- Which human papillomavirus type is associated with giant condylomata of Bushke and Lowenstein (Bushke-Lowenstein tumor)?

A. 1
B. 2
C. 6
D. 16
E. 31

A

►C

The Bushke-Lowenstein tumor is caused by HPV-6 and 11.

How well did you know this?
1
Not at all
2
3
4
5
Perfectly
141
Q
  1. A- 48 year-old man develops headache, myalgias, and high fever 5 days after a hunting trip. On examination he has a tender ulcer with raised margins and an eschar on his index finger, as well as tender axillary lymphadenopathy. The most likely diagnosis is:

A. Anthrax
B. Orf
C. Glanders
D. Tularemia
E. Erysipeloid

A

►D

Tularemia, which is caused by Francisella tularensis, occurs after exposure to infected animals, including, rabbits, foxes, and squirrels. The clinical description is that of the ulceroglandular form, which is the most common presentation.

How well did you know this?
1
Not at all
2
3
4
5
Perfectly
142
Q

142- The treatment of choice of the tropical infectious disease shown is:

A. Ivermectin
B. Permethrin
C. Diethylcarbamazine
D. Pentavalent antimony
E. Mebendazole

A

►D

The ulcerated nodule shown here is characteristic Leishmaniasis is treated with pentavalent antimony. of cutaneous leishmaniasis (CL).

How well did you know this?
1
Not at all
2
3
4
5
Perfectly
143
Q

143- A homeless patient with a history of diabetes and alcoholism presents with chronic suppurative nodules with granular bodies on light microscopy. Culture rapidly grows Staphylococcus aureus colonies. What is the clinical term describing this type of infectious lesion?
A. Botryomycosis
B. Scrofuloderma
C. Rhinoscleroma
D. Ecthyma gangrenosum
E. Erysipeloid

A

►A

Chronic purulent nodules with granular bodies seen in patients with immunosuppresion, diabetes, and alcoholism is called botryomycosis. Causative organisms include S. aureus, P. aeruginosa, E. coli, and Proteus species. Scrofuloderma is cutaneous extension of underlying Mycobacteria tuberculosis infection. Rhinoscleroma is caused by Klebsiella pneumoniae rhinoscleromatis. Ecthyma gangrenosum is typically seen following Pseudomonas septicemia. Erysipeloid is caused by Erysipelothrix rhusiopathiae.

How well did you know this?
1
Not at all
2
3
4
5
Perfectly
144
Q

144- Corynebacterium tenuis is responsible for which of the following infections?

A. Majocchi’s granuloma
B. Trichomycosis axillaris
C. Erysipelas
D. Malakoplakia
E. Erysipeloid

A

►B

Corynebacterium tenuis is responsible for causing trichomycosis axillaris, a superficial bacterial overgrowth of axillary hairs characterized by circumferential yellow (most common), red, or black granular concretions. Majocchi’s granuloma is a dermatophyte infection. Erysipelas is caused by beta-hemolytic group A streptococcus. Malakoplakia is usually caused by S. aureus, P. aeruginosa, or E. coli. Erysipeloid is caused by Erysipelothrix rhusiopathiae. Corynebacterium minutussimum is the causative organism in erythrasma and pitted keratolysis.

How well did you know this?
1
Not at all
2
3
4
5
Perfectly
145
Q

145- A 32 year-old woman presents with meningitis and palpable purpura. A diagnosis of meningococcemia is confirmed via culture of cerebrospinal fluid. Which complement components are most likely to be deficient in this patient?
A. C1
B. C2
C. C3
D. C4
E. C5

A

►E

Patients with meningococcemia often demonstrate deficiencies of late complement components C5-9.

How well did you know this?
1
Not at all
2
3
4
5
Perfectly
146
Q

146- Trichomycosis axillaris has a yellowish brown concretions on axillary hair shaft and is caused by:
A. Corynebacterium tenuis
B. Francisella tularensis
C. Epstein-Barr Virus
D. Staphylococcal aureus
E. Micrococcus sedenatrius

A

►A

Trichomycosis axillaris is caused by Corynebacterium tenius. It is a yellowish brown concretions on axillary hair shafts. Treatment is shaving, benzoyl peroxide gel, and topical erythromycin.

147
Q

147- What is the vector for this nematodal infection?

A. Water flea (Cyclops)
B. Tsetse fly (Glossine morsitans)
C. Reduviid bug
D. Mango fly (Chrysops)
E. Black fly (Simulium)

A

►D

Loiasis is caused by the nematode loa loa. It is transmitted by the mango fly (Chrysops dimidia or CHrysops silacea). It may manifest with painful, localized subcutaneous nonpitting edema called calabar swellings. Conjunctival migrations are also common.

148
Q

Human bites result in infection with this gram negative bacillus that is the part of the normal flora of the human mouth. The infection can also follow fist fights from blows to the mouth. The infection is caused by:

A. Eikenella corrodens
B. Bartonella hensalae
C. Pastuerella mutocida
D. Pseudomonas
E. Crotalidae

A

►A

Eikenella corrodens is common in human bite infections. Pasteurella multocida is usually from cat bites and Pasteurella canis is from dog bites.

149
Q

149- A patient is suspected to have contracted rabies after a dog bite. The best site for diagnostic biopsy would be:

A. Neck
B. Acral area
C. Site of bite
D. Buccal mucosa
E. Buttocks

A

►A

The best site for diagnosis of rabies is a biopsy from the neck. This virus has an incubation period of 5 days to 1 year and demonstrates retrograde axoplasmic flow until it reaches the spinal cord. It subsequently undergoes centrifugal spread along the peripheral nerves to the skin, intestine, and into the salivary glands where it is shed.

150
Q

150 -What is the annual risk of developing herpes zoster in an HIV infected patient?

A. 3% per month
B. 10% per month
C. 3% per year
D. 10% per year
E. 25% per year

A

►C

The annual risk of developing herpes zoster in an HIV infected individual, regardless of the CD4 count, is approximately 3%.

151
Q

151- A 59-year-old male returns from traveling abroad where he ate many fish. Since then he complaints of recurring migratory 1 cm/day erythematous urticarial plaques. Biopsy shows eosinophilic panniculitis. What is the most likely condition?

A. Drancunculiasis
B. Gnathostomiasis
C. Enterobiasis
D. Hookworm
E. Larva currents

A

►B

This patient has gnathostomiasis caused by Gnathostoma dolorosi or spinigerum. It is commonly acquired from freshwater fish. It manifests with migratory intermittent erythematous urticarial

plaques that recur every 2-6 weeks. The plaques move about 1 cm/day. Histology shows eosinophilic panniculitis. Treatment is surgical removal or albendazole.

152
Q

152- Foamy macrophages containing Klebsiella pneumoniae are called:

A. Virchow cells
B. Hansemann’s cells
C. Mikulicz’s cells
D. Michaelis-Gutman cells
E. Russell cells

A

►C

Mikulicz”s cells are foamy macrophages found in Rhinoscleroma “ a chronic, inflammatory, granulomatous disease of the upper respiratory tract. Virchow cells are found in Hansen”s disease. Hansemann cells and Michaelis-Gutman bodies are found in Malakoplakia.

153
Q

153- A 37 year old male with a history of HIV/AIDS presents complaining of the new onset of purplish nodules of the feet and lower legs. He lost his insurance eight months ago and has been unable to afford the HAART therapy that he was prescribed. HHV-8 is found in the semen of what percent of men with this disease?

A. 5%
B. 20%
C. 35%
D. 50%
E. 65%

A

►B

Kaposi’s sarcoma is a vascular neoplasm seen most commonly on the lower extremities, back, and genitaliae. In the United States, this disease is most commonly seen in patients with AIDS. Kaposi’s

sarcoma has been linked to infection with HHV-8. 20% of men with Kaposi’s sarcoma will have detectable HHV-8 in their semen.

154
Q

154- Which bacterial disease carries a risk of mortality due to coinfection with salmonella?

A. Carrion’s disease
B. Cat-scratch disease
C. Trench fever
D. Rocky Mountain Spotted Fever
E. Q fever

A

►A

Carrion’s disease, or Oroyo fever, is caused by Bartonella bacilliformis and is transmitted via the Lutzomyia sandfly. It is characterized by fevers, headache, arthralgias, and an acute eruptive stage that resembles bacillary angiomatosis. It has an untreated fatality rate of 40% secondary to salmonella infection; chloramphenicol is given with tetracycline for treatment. Epidemic typhus may also coexist with salmonella.

155
Q

155- Xenopsylla cheopis is the vector responsible for which infectious disease?

A. Human Monocytic Ehrlichiosis
B. Human Granulocytic Ehrlichiosis
C. Scrub typhus
D. Epidemic typhus
E. Endemic typhus

A

►E

Xenopsylla cheopis (rat flea) is the vector of endemic typhus, which is caused by R. typhi.

156
Q

156- A 25 year old woman who is 13 weeks pregnant is exposed to cytomegalovirus (CMV). Three weeks later, she develops infectious mononucleosis symptoms. What percentage of neonates with congenital CMV infection are asymptomatic?

A. 10%
B. 25%
C. 50%
D. 75%
E. 90%

A

►E
Symptoms of congenital CMV infection include low birth weight, microcephaly, seizures, petechial rash, and hepatosplenomegaly. Fortunately, the vast majority (90%) of congenital CMV infections are asymptomatic.

157
Q

157- Which of the extracellular enzymes of S. aureus plays a role in toxic shock syndrome (TSS)?

A. Coagulase
B. Penicillinase
C. Enterotoxin
D. Hemolysin
E. Exfoliatin

A

►C

In addition to TSST-1 which is present in 50% of non-menstrual cases of TSS, enterotoxins, especially B & C, cause TSS. These enterotoxins are also superantigens, which recognize the V
-beta region of the T-cell receptor. Coagulase clots plasma, Penicillinase degrades penicillin, Hemolysin lyses red blood cells and exfoliatin splits the epidermis (antigen is desmoglein 1).

158
Q

158- A patient with primary syphilis is treated with Benzathine penicillin G. Soon after treatment, he develops fever, headache, myalgias, and elevated white blood cell count, consistent with the

Jarisch-Herxheimer Reaction. This reaction is primarily mediated through which inflammatory cytokine?

A. TNF-alpha
B. IFN-gamma
C. IL-6
D. EGFR
E. IL-10

A

►A

The Jarisch-Herxheimer Reaction is characterized by fever, headache, lymphadenopathy, myalgias, and elevated white blood cell count. It is caused by the release of inflammatory cytokines, particularly TNF-alpha, due to phagocytosis of spirochetes following antibiotic administration.

159
Q

159 -A 23 year old Peace Corp volunteer returns from Africa with an extensive undermined, nonhealing ulceration on the right lower leg. Which of the following infections is the most likely cause of this presentation?

A. M. marinum
B. M. kansasii
C. M. chelonei
D. M. ulcerans
E. Leishmania donovani

A

►D

The most likely cause of a non-healing ulceration in someone who has been in a tropical location is
M. ulcerans. M. marinum causes inflammatory nodules following injury in an aquatic environment.
M. chelonei is a rapidly growing organism most commonly found in soil, water, dust and animals which presents as single/multiple erythematous SC nodules on an extremity or following a surgical procedure. M. kansasii rarely causes skin lesions. L. donovani is a causse of visceral leishmaniasis in India and Kenya (kala-azar).

160
Q

160- Calabar swellings are a characteristic feature of which disease?

A. Onchocerciasis
B. Eumycetoma
C. Tertiary Syphilis
D. Carrion’s Disease
E. Loiasis

A

►E

Calabar swellings are characteristic of Loiasis. They are transient, non-tender soft tissue swellings found most commonly around joints.

161
Q

161- A 8 year-old girl presents to the pediatrician with a poorly marginated blue-red single indurated plaque on her right cheek. What is the most likely causative organism?

A. Beta-hemolytic group A streptococcus
B. Staphyloccus aureus
C. Pseudomonas aeruginosa
D. Haemophilus influenzae
E. Klebsiella pneumoniae

A

►D

The most likely diagnosis is H. flu cellulitis. This infection usually occurs in children and is characterized by a single indurated plaque with poorly delineated margins. Erysipelas, an infection caused by beta-hemolytic streptococcus group A, usually presents with well delineated margins in adult patients.

162
Q

162- Which virus is most commonly associated with oral hairy leukoplakia?

A. Pox virus
B. Herpes virus

C. Epstein-Barr virus
D. Human papilloma virus
E. Parvovirus

A

►C

Oral hairy leukoplakia is an oral mucosal infection most often caused by the Epstein-Barr virus, which occurs in immunocompromised patients. Clinically, thick, white plaques are noted on the lateral sides of the tongue, often fissured.

163
Q

163 -A 29-year-old woman presents with multiple crateriform pits on the soles of her feet following a month of hiking on the Appalachian Trail. She wants a complete skin exam and wonders about her feet. Which of the following do you plan to do next for her foot problem.

A. These pits are a variant of normal
B. You explain that she has a hereditary syndrome that causes these pits
C. You explain that this is a superficial bacterial infection and will resolve with topical therapy
D. You ask for a skin biopsy
E. You use a Wood‟s lamp to confirm your diagnosis

A

►C

Multiple crateriform pits on the feet following suboptimal hygiene most likely is pitted keratolysis caused by Micrococcus sedentarius. Treatment is topical erythromycin, clindamycin or benzoyl peroxide. This is not Gorlin‟s syndrome (Basal Cell Nevus syndrome). While these pits are common, it is not a variant of normal. A skin biopsy could confirm the diagnosis, but is not necessary. A Wood‟s lamp would not be particularly helpful. Some believe that Corynebacterium spp. also cause this condition, but that is still theory and not proven.

164
Q

164- Roseola infantum is caused by which virus?

A. Coxsackie virus A16

B. Human herpes virus 6
C. Parvovirus B19
D. Epstein-Barr virus
E. Pox virus

A

►B

Roseola infantum may be cause be either human herpes virus types 6 or 7. It is the sixth of the traditional exanthems of childhood. A diffuse, maculopapular eruption is typically preceded by a prodrome of fever and either respiratory or gastrointestinal symptoms. The infection usually occurs in the spring and the fall and nearly all children are seropositive for the virus by the age of 4.

165
Q

165- Butcher’s warts are most commonly caused by which human papilloma virus (HPV) type?

A. 7
B. 2
C. 3
D. 5
E. 13

A

►A

Butcher’s warts are caused by HPV type 7.

166
Q

166- For which of the following smallpox vaccination complications is the administration of vaccine immune globulin indicated?

A. Post-vaccinal encephalitis
B. Erythema multiforme
C. Eczema vaccinatum
D. Vaccinia keratitis
E. Mild generalized vaccinia

A

►C

Vaccinia immune globulin can be administered to treat some of the complications associated with vaccinia vaccination. Vaccinia immune globulin is indicated for inadvertent inoculation with extensive involvement or ocular implantations, eczema vaccinatum, generalized vaccinia (severe or recurrent), and progressive vaccinia. It is not recommended for inadvertent inoculation with mild reaction, generalized vaccinia (mild or limited), post-vaccination encephalitis, and isolated vaccinia keratitis.

167
Q

167- Nonmenstrual cases of the staphylococcal toxic shock syndrome are mediated by which of the following toxins?

A. Enterotoxins B and C
B. Exfoliative toxins A and B
C. Edema toxin
D. Epidermolytic toxins A and B
E. Pyrogenic exotoxins A, B, C

A

►A

Toxic shock syndrome (TSS) is mediated by Enterotoxins B and C (50% of nonmenstrual cases) and TSS toxin 1 (TSST-1), which is the leading cause of menstrual cases.

168
Q

168- Which of the following skin manifestations of M. tuberculosis infection is a tuberculid?

A. Erythema induratum of Bazin
B. Lupus vulgaris
C. Miliary TB
D. Scrofuloderma
E. Tuberculous gumma

A

►A

A tuberculid is a cutaneous immunologic reaction to tuberculosis elsewhere in the body. By definition, cultures and stains for M. tuberculosis are negative. Erythema induratum of Bazin/Nodular vasculitis present as erythematous or cyanotic nodules on the posterior calves. 85% of cases occur in women. The other listed presentations of TB are all due to secondary spread from an established TB infection at another location. In some cases, organisms can be recovered and cultures may be positive.

169
Q

The treatment for patients that are pregnant or for children <9 for lyme disease is:

A. Doxycycline
B. Levoquin
C. Amoxicillin
D. Tetracycline
E. Penicillin

A

►C

Patients that are pregnant or for children <9 years old, the treatment for lyme disease is amoxicillin. Lyme disease is caused by B. burgdorferi. Clinical signs are erythema migrans and acrodermatitis chronica atrophicans.

170
Q

170 -A 35 year old male has had frequent sex with prostitutes. He presents complaining of the rapid onset of a new rash on his legs. A review of symptoms is otherwise unremarkable. Examination reveals palpable purpura and tender nodules of the lower legs. Biopsy reveals vasculitis of small- and medium-sized vessels. What percentage of patients with this disease are found to be infected with Hepatitis B virus?

A. 7-8%
B. 20%
C. 35%
D. 1%
E. 50%

A

►A

Polyarteritis nodosa is characterized by the onset of painful nodules on the lower extremities. It can be limited to the skin (cutaneous PAN), or can involve a wide variety of organ systems (systemic PAN). Approximately 7-8% of cases of polyarteritis nodosa cases are associated with Hepatitis B virus infection.

171
Q

171- A 43 year-old man presents with suppurative nodules and sinus tracts over the mandible. He also is noted to have poor oral hygiene. What is the most likely causative organism?

A. Bartonella Henselae
B. Nocardia brasilensis
C. Streptococcus somaliensis
D. Actinomyces israelii
E. Staphylococcus aureus

A

►D

Actinomyces israelii is an anaerobic gram-positive rod which causes chronic suppurative nodules and sinus tracts with an exudate containing sulfur granules. It most commonly effects the cervicofacial area, especially near the mandible. The abdomen and thoracic areas can also be affected. The source of infection is endogenous and patients with poor oral hygiene, penetrating foreign bodies, and recent dental procedures are at increased risk. Treatment of choice is penicillin.

172
Q

172- A young male patient presents with a painful ulcer with a ragged edge after a new sexual encounter. What is the treatment of choice?

A. Penicillin
B. Doxycycline
C. Azithromycin
D. Ciprofloxacin
E. Cephalexin

A

►C

Ulcers in the genital area in sexually active patient generally fall into two groups: painful and painless. Painless ulcers include syphilllis, lymphogranuloma venereum, and granuloma inguinale. The primary cause of painful erosion or ulcer is H. ducreyi or herpes simplex. An ulcer with a ragged edge that is painful is characteristic of H. ducreyi. The treatment of choice is azithromycin.

173
Q

173- A 35 year-old woman presents with a painful vaginal erosion and tender left-sided inguinal lymphadenopathy. Gram stain reveals clusters of coccobacilli in a “school of fish” pattern. What is the most likely diagnosis?

A. Primary syphilis
B. Lymphogranuloma venereum
C. Granuloma inguinale
D. Chancroid
E. Secondary syphilis

A

►D

Chancroid classically presents with a soft, tender chancre with ragged edges and unilateral, tender inguinal adenopathy (bubo). A “school of fish” pattern on Gram or Giemsa stain is diagnostic.

174
Q

174- Botryomycosis is:

A. A fungal infection caused by M. canis
B. A bacterial infection commonly associated with Clostridium botulinum
C. A bacterial infection commonly associated with Staphylococcus aureus
D. A fungal infection caused by T. tonsurans
E. An acute, disseminated infection involving the genitourinary system

A

►C

Botryomycosis is an uncommon, chronic, idonlent disorder characterized by nodular, crusted, purulent lesions. Sinuses that discharge sulfur granules are present. These heal with atrophic scars. The granules yield most commonly Staphylococcus aureus on culture, although cases caused by Pseudomonas aeruginosa, E. coli, Proteus, Bacteroides, and Streptococcus have been reported

175
Q

175- Which of the following is not a DNA virus?

A. Herpesvirus
B. Poxvirus
C. Parvovirus
D. Paramyxovirus
E. Adenovirus

A

►D

Paramyxovirus is a single-stranded RNA virus. All other listed choices are DNA-based viruses.

176
Q

176- A patient is diagnosed with a unilateral enlargement of the medial clavicle after being diagnosed with congenital late syphilis. This is called the:

A. Higoumenaskis sign
B. Hutchinson’s sign
C. Saber sign
D. Saddle sign
E. Gummas

A

►A

This patient has congenital late syphilis, seen in children over two years of age. The unilateral enlargement of the medial clavicle is called the Higoumenaki’s sign. These patients also can have Hutchinson’s teeth or widely-spaced, peg-shaped teeth in the upper incisors. They can also have a saddle nose and saber shins.

177
Q

178- Histoplasmosis is a self limiting pulmonary disease that is related to the dose of conidia and can be seen in AIDS and immunosuppressed patients. It is associated with:

A. Bird and bat droppings
B. Rat droppings
C. Deer droppings
D. Deer mites
E. Tic bites

A

►A

Histoplasmosis is associated with bird and bat droppings and is seen in the mississippi and ohio river valleys in conjunction with inhalation of microconidia.

178
Q

179- A patient with HIV/AIDS presents complaining of an asymptomatic patch on the tongue. Examination reveals a white patch with a corrugated surface on the lateral tongue. What percentage of HIV patients will have this finding?

A. 5%
B. 20%
C. 33%
D. 50%
E. 67%

A

►C

Oral hairy leukoplakia is characterized by a white patch with a corrugated surface on the side of the tongue. It is generally asymptomatic. Oral hairy leukoplakia is seen most commonly in patients with defective cell-mediated immunity, in particular HIV infection. Oral hairy leukoplakia is associated with EBV opportunisitc infection, among other viruses. 33% of patients with HIV will have oral hairy leukoplakia.

179
Q

180 -The vector of New World Leishmaniasis is the:

A. Phlebotomus sandfly
B. Lutzomyia sandfly
C. Tsteste fly
D. Deer fly (Chrysops spp.)
E. Simulium black fly

A

►B

The sandfly belonging to the genus Lutzomyia is the vector of New World Leishmaniasis. Sandflies of the genus Phlebotomus are the vectors of Old World Leishmaniasis.

180
Q

181- A 34 year old woman complains of recurrent painful blisters on her lower lip. These occur 4 times per year, and are preceded by painful sensations for a day before the blisters appear. What is the most common trigger of recurrent orolabial herpes simplex virus infection?

A. UVB radiation
B. UVA radiation
C. Visible light radiation
D. Emotional stress
E. Medication use, including opioids

A

►A

Recurrent orolabial herpes simplex virus infection is a very common problem. In many cases, the cause of the recurrences is idiopathic. Identifiable triggers include light exposure and emotional stress, among others. UVB is the most commonly implicated factor.

181
Q

182- Verruga peruana is transmitted by:

A. Tick
B. Blackfly
C. Sandfly
D. Fecal oral contact

E. Fecal fecal contact

A

►C

Verruga peruana, a disease endemic to Peru and a few neighboring countries, is caused by Bartonella bacilliformis. It is transmitted by the sandfly, Lutzomyia verrucarum. It is preceded by an acute febrile stage called Oroyo fever or Carrion”s disease. The treatment of choice is chloramphenicol.

182
Q

183- Which of the following is the vector of lymphatic filariasis caused by Wuchereria bancrofti?

A. Black fly
B. Tsteste fly
C. Mosquito
D. Deerfly
E. Botfly

A

►C

Lymphatic filariasis is spread by mosquitoes belonging to genera Aedes, Anopheles, Culex, or Mansonia.

183
Q

184- A patient is diagnosed with Loiasis. This patient most likely has clinical features such as:

A. Adult worm migrating across the conjunctivae
B. Nodules over bony prominences on the extremities
C. Ocular involvement that leads to blindness
D. Depigmented macules/patches also known as “leopard skin”
E. Pruritic papules

A

►A

Loiasis is caused by the organism Loa Loa. The vector is mango flies. The clinical characteristic is calabar swelling and adult worm migrating across conjunctivae or “eye worm”. The treatment is DEC or diethylcarbamazine.

184
Q

185- A patient on the bone marrow transplant service has a fever, neutropenia, hemorrhagic bullae and erythematous nodules with dusky gray centers. The organism most likely to be responsible is:

A. Streptococcus pyogenes
B. Mycobacterium tuberculosis
C. Candida albicans
D. Pseudomonas aeruginosa
E. Staphylococcus aureus

A

►D

Ecthyma gangrenosum is an infection of critically ill or immunocompromised individuals by Pseudomonas aeruginosa. Clinically, patients develop opalescent, tense vesicles or pustules surrounded by a narrow pink to violaceous halo. The lesions rapidly become hemorrhagic, then violaceous and necrotic leaving ulcers. The most common location is on the buttocks and lower extremities. Treatment is with aminoglycosides.

185
Q

186- What is the principal vector of Lyme Disease in the Northeastern U.S.?

A. Ixodes ricinus
B. Soft-bodied ticks (Ornithodoros)
C. Ixodes dammini
D. Ambylomma americanum
E. Dermacentor variabilis

A

►C

The proncipal vector of Lyme Disease in the Northeastern U.S. is Ixodes dammini (Ixodes scapularis).

186
Q

187- Patients that have diagnosed with congenital syphilis have dental findings called:

A. Hutchinson teeth
B. Erythrodontia
C. Hypodontia
D. Peg-shaped teeth
E. Enamel hypoplasia

A

►A

Congential sphyilis can be passed from the mother to the child. Dental findings are called hutchinson teeth and mulberry molars.

187
Q

188- Each of the following is true regarding leishmaniasis except:

A. Arthropod vector is the sand fly
B. Mazzotti‟s test is diagnostic
C. Pentavalent antimony used for visceral disease
D. Cutaneous disease is the most common form
E. Espundia seen mucocutaneous disease

A

►B

Leishmaniasis is a parasitic infection born by a sandfly vector. No treatment is necessary for cutaneous leishmaniasis, pentavalent antimony for visceral leishmaniasis and mucocutaneous. Mazzotti reaction is used to test for onchocerciasis in which a single dose of dimethycarbamazine is given for reaction. Amphotericin B is only used in pentavalent antimo ny resistant cases.

188
Q

189- All of the following are true regarding smallpox except:

A. Caused by variola virus
B. Transmitted by respiratory secretions
C. Virus is found in skin lesions

D. Vaccination is contraindicated in children under 5
E. Associated with thrombocytopenia

A

►D

Smallpox is an acute exanthematous infection caused by the variolae poxvirus. It is transmitted primarily by respiratory secretions as well as more infrequently skin inoculation and fomite spread. Following contact, there is an asymptomatic period of 12-13 days. Following the asymptomatic period, patients develop a prodrome of fever, headache, back pain and vomiting for 3 days. Then, the characteristic deep seated papules that vesicles appear. These lesions mature into a pustule with a central umbilication. Occasionally, patients may develop a hemorrhagic form of the disease which is associated with thrombocytopenia and poor prognosis. Vaccination is not contraindicated in children under 5.

189
Q

190- This type of syphilis has the clinical characteristic of widespread cutaneous eruptions, hamcolored macules on the palms and soles, moth eaten alopecia and condyloma lata.

A. Primary
B. Secondary
C. Tertiary
D. Latent
E. Congenital

A

►B

This is an example of secondary syphilis. It involves the skin and mucous membranes. It also lasts for 4 to 12 weeks. It involved widespread cutaneous eruptions, ham-colored macules on the palms and soles, mucous patches, condyloma lata, split papules and moth eaten alopecia.

190
Q

191- A 67 year old homeless man comes to clinic complaining of a painful lump along his lateral jawline which has been present for many months. He has noticed that it occasionally drains a purulent material. What is the most likely diagnosis?

A. Actinomyces israelii
B. Nocardia brasilensis
C. Actinomadura madurae
D. Streptomyces somaliensis
E. Mycobacteria tuberculosis

A

►A

Actinomycosis is a chronic suppurative infection forming nodules and sinus tracts with an exudate containing sulfur granules. Risk factors include poor oral hygiene, penetrating foreign bodies, and dental procedures. The causative organism is Actinomyces israelii, an anaerobic gram-positive rod.

191
Q

192- What are the three rapid grower mycobacteria and what is the current treatment of choice?

A. leprae, chelonae, abscessus; clarithromcyin
B. Tuberculum, abscessus, fortuitum; rifampin
C. Chelonae, asbcessus, fortuitum; clarithromycin
D. Lepra, fortuitum, abscessus; minocycline
E. Chelonae, absccessus, fortuitum; minocycline

A

►C

These three are rapid grower mycobacteria and the treatment of choice is clarithryomycin. Minocycline can also be used, but it considered second line in WHO treatment.

192
Q

193- All of the following are potential causes of a false positive RPR except:

A. Systemic Lupus Erythematosus (SLE)
B. Pregnancy
C. Malignant Melanoma
D. Lepromatous Leprosy
E. Malaria

A

►C

Non-treponemal tests for syphilis measure antibodies against phospholipids released from treponemes and damaged host cells. False positive non-treponemal tests can occur in the setting of pregnancy, autoimmune diseases, other spirochete infections, and infectious diseases such as leprosy and malaria. It does not occur in cases of melanoma.

193
Q

194- A 23 year old female patient is found to have genital warts. A referral to an gynecologist reveals that she has cervical dysplasia. What viral proteins does HPV use to induce neoplasia?

A. E6 and E7
B. E1 and E2
C. L1 and L2
D. E8 and L1
E. E3 and E4

A

►A

Certain HPV subtypes have been shown to induce neoplasia, including cervical cancer. There are many high-risk subtypes, including 16, 18, 31, and 33. The HPV early genes E6 and E7 bind the tumor suppressors p53 and retinoblastoma protein, respectively, inactivating them and initiating oncogenesis.

194
Q

195- A patient is given a single dose of dimethylcarbamazine and soon develop edema, itching, fever, arthralgias, and exacerbation of pruritus. He most likely has which underlying condition?

A. Onchocerciasis
B. Loiasis
C. Filariasis
D. Gnothostomiasis
E. Trichinosis

A

►A

This patient developed an acute reaction to dimethylcarbamazine indicating a Mazzotti reaction. This reaction is seen with onchocerciasis. Appropriate treatment for onchocerciasis, caused by Onchocerca volvulus, is ivermectin. These patients may demonstrate pruritus, dermatitis, leopard skin, onchocercomas (nodules with microfilariae) and blindness.

195
Q

196- How long is the life cycle for the mite that causes this infestation?

A. 1 day
B. 1 week
C. 1 month
D. 6 months
E. 1 year

A

►C

Scabies is caused by the mite Sarcoptes scabiei var. hominis. The life cycle of the mite is 30 days. A female mite will lay 60-90 eggs during her life.

196
Q

197 - Winterbottom‟s sign is a characteristic feature of which of the following diseases?

A. Leishmaniasis
B. Onchocerciasis
C. Loaiasis
D. African Trypanosomiasis
E. Strongyloidiasis

A

►D

Winterbottom‟s sign (posterior cervical lymphadenopathy) is a clinical feature of African trypanosomiasis. Romaña‟s sign (eyelid edema and conjunctivitis at site of incoculation) is a diagnostic finding in American trypanosomiasis (Chagas‟ disease).

197
Q

198- A 40-year-old male with HIV (CD4 <200) presents with asymptomatic, slowly enlarging yellowpink papules and nodules localized to his perianal skin. A skin biopsy is performed, which reveals foamy dermal interstitial histiocytes with characteristic basophilic laminated inclusion bodies. The latter are noted to stain positively with von Kossa, Perls, and PAS stains. The most likely diagnosis is:

A. eruptive xanthomas
B. malakoplakia
C. hidradenitits suppurtiva
D. granuloma inguinale
E. chancroid

A

►B

Malakoplakia is a multi-organ inflammatory granulomatous disease that most frequently arises in the setting of immunosuppression. Common sites of involvement include the genitourinary, lung, lymph nodes and bone. Cutaneous involvement is rare, and typically presents as asymptomatic papules, nodules, or plaques. Lesions may have associated erosions or ulcerations, and sinus tracts may be seen. While skin lesions are non-specific, histopathology reveals foamy dermal histiocytes known as von Hansemann cells which contain basophilic granular inclusions. The latter are known as Michaelis-Gutmann bodies and are considered pathognomonic for malakoplakia. Michaelis- Gutmann bodies are comprised of calcium, iron, and phosphate salts, and hence stain positively with von Kossa, Perls, and PAS, respectively. The pathophysiology of malakoplakia is thought to arise from an acquired defect in macrophage activation in response to bacterial infection, most commonly. E. coli. Treatment for cutaneous lesions is surgical excision, although recurrence is not uncommon.

198
Q

199 -Carrion’s disease is characterized by fevers, headaches and arthralgias and is accompanied by severe hemolytic anemia. Superinfection with which organism is the most frequent cause of death?

A. Bartonella bacilliformis
B. Bartonella henselae
C. Bartonella Quintana
D. Salmonella
E. Shigella

A

►D

Carrion’s disease (Oroya fever) is characterized by fevers, headaches and arthralgias and is accompanied by severe hemolytic anemia. Superinfection with Salmonella is the most frequent cause of death. Protection from sandfly bites is all-important.

199
Q

200- All of the following are true regarding tularemia except:

A. Caused by the gram negative Francisella tularensis
B. Ticks serve as a vector
C. No longer one of the CDC’s list of reportable diseases
D. Presents as an acute febrile illness
E. Potential for use as a biologic weapon

A

►C

Tularemia is a zoonotic disease caused by the gram-negative coccobacillus Francisella tularensis. It is transmitted by ticks: dog tick, wood tick, lone star tick, or from deer flies (Chrysops) or from contact with infected mammals (usually rabbits) or from infected water. It characteristically presents as an acute febrile illness with other variable clinical manifestations depending on the route of inoculation. Possible symptoms could include an ulcer at the site of inoculation, pharyngitis, ocular lesions, lymphadenopathy, and pneumonia. Diagnosis can be made by culture or a fourfold titer change. Because of its potential use as a biologic agent, tularemia is once again a reportable disease.

200
Q

201- The characteristics of tuberculoid leprosy or TT is:

A. Lepromin test +
B. IL4, IL-10,
C. Multibacillary
D. Lepromin test -
E. No loss of sensation or sweating

A

►A

Tuberculoid leprosy has a TH1 cytokine profile of IFN gamma, IL-2, IL-12. It is paucibacillary, Lepromin test +. There is less than 3 lesions and anesthetic and anhidrotic lesions.

201
Q

202- Exanthem Subitum is caused by which of the following?

A. Group A Steptococcus
B. Coxsackie virus
C. Human Herpes Virus-6 (HHV-6)
D. Parvovirus B19
E. Epstein-Barr Virus

A

►C

Exanthem Subitum (Roseola Infantum, Sixth Disease) is caused by human herpesvirus 6, and 7 (HHV-6, HHV-7).

202
Q

203- This is strain of HPV is associated with Butcher’s warts:

A. HPV-7
B. HPV 10, 13
C. HPV 30
D. HPV 2,4
E. HPV 28, 29

A

►A

The butcher warts are associated with HPV-7. Laryngeal carcinoma is associated with HPV-30. Common warts are HPV 2, 4.

203
Q

204- Which porphyrin is responsible for the fluorescence under Wood’s lamp examination of erythrasma?

A. Porphobilinogen
B. Protoporphyrin IX
C. Coproporphyrin III
D. Uroporphobilinogen
E. Coproporphyrinogen

A

►C

Erythrasma is caused by corynebacterium minutissimum. The presence of coprophyrinogen III created by the bacteria is responsible for the characteristic coral-red fluorescence under Wood’s lamp.

204
Q

205- Similar lesions were also seen on this infant’s face. Treatment of choice is:

A. phototherapy
B. oral steroids
C. acyclovir
D. topical retinoic acid
E. oral erythromycin

A

►C

This child has eczema herpeticum, a cutaneous infection with HSV which occurs in patients with pre-existing skin disorders such as atopic dermatitis. It begins as clusters of umbilicated and hemorrhagic vesicles in areas of abnormal skin. Lesions spread rapidly but can respond quickly to antiviral treatment. Children with underlying AD who come into frequent contact with an individual with orolabial HSV may have recurrent outbreaks of eczema herpeticum.

205
Q

206- Erythema infectiosum is caused by a:

A. RNA virus

B. Herpes virus
C. Double stranded DNA virus
D. Single stranded DNA virus
E. None of these answers are correct

A

►D

Erythema infectiosum is caused by parvovirus B19 which is a single stranded DNA virus.

206
Q

207- Blueberry Muffin Baby can be a feature of all of the following EXCEPT:

A. TORCH
B. Klippel-Trenaunay-Weber syndrome
C. Hemolytic disease of the newborn
D. Rhabdomyosarcoma
E. Langerhans‟ cell histiocytosis

A

►B

Blueberry muffin lesions can be seen in the setting of prenatal infections (e.g. TORCH), severe anemia (e.g. Hemolytic Disease of the newborn) and neoplastic diseases (e.g. rhabdomyosarcoma). It is not associated with the Klippel-Trenaunay-Weber syndrome.

207
Q

208- Which of the following is the causative organism of cat-scratch disease?

A. B. quintana
B. B. henselae
C. B. bacilliformis
D. B. duttonii
E. B. burgdorferi

A

►B

B. henselae is the cause of cat-scratch disease. Humans are infected by a cat bite or scratch, while the cat flea is responsible for cat to cat transmission.

208
Q

209- The most common location of herpes gladiatorum is the:

A. Groin
B. Chest
C. Anterior thigh
D. Face
E. Hand

A

►D
The face is the most common location for Herpes Gladiatorum.

209
Q

210- You are on a medical mission in the Peruvian Amazonia and are asked to examine a patient. The patient is concerned about a skin lesion on her abdomen. What is the most likely diagnosis in your differential?

A. Yellow fever
B. Dengue fever
C. Malaria
D. Strongyloidosis
E. Leptospirosis

A

►D

Strongyloides stercoralis has a very low prevalence in societies where fecal contamination of soil or water is rare. It is a very rare infection in developed economies and most common in tropical climates. Strongyloides stercoralis causes morphologically variable skin lesions. Rarely, a petechial purpuric eruption which resembles multiple thumbprints may initially present over the periumbilical area. Later on it may involve the whole abdomen, thighs, legs and arms. The petechial purpuric skin

lesions in disseminated strongyloidiasis are due to damage to the blood vessels caused by larval migration.

210
Q

211- Which of the following are potential forms of perinatally-acquired neonatal herpes disease?

A. Skin, eyes, mouth (SEM)
B. CNS
C. Disseminated
D. None of the above
E. All of the above

A

►E

Perinatally acquired neonatal herpes disease can present in three different, occasionally overlapping, forms: 1. skin, eyes, and mouth; 2. CNS; and 3. Disseminated, causing encephalitis, hepatitis, pneumonia, and coagulopathy.

211
Q

212- Measles is caused by a:

A. Paramyxovirus
B. Togavirus
C. Picornavirus
D. Rhabdovirus
E. Adenovirus

A

►A

The measles virus is an RNA virus belonging to the Paramyxovirus family.

212
Q

213- A patient presents with this anesthestic plaque. Which cytokines are upregulated in this disease?

A. IL-2
B. IL-4
C. IL-5
D. IL-10
E. All of these cytokines are upregulated

A

►A
Leprosy is caused by infections to Mycobacterium leprae. It is a acid fast bacilli. In tuberculoid leprosy, Th1 cytokines (IL-2, IFN-gamma) are upregulated. Clinically, patients have few, wellcircumscribed anesthetic plaques.

213
Q

214- The Mikulicz is the histologic hallmark of which of the following conditions:

A. Leishmaniasis
B. Granuloma inguinale
C. Rhinoscleroma
D. Rhinosporidiosis
E. Histoplasmosis

A

►C

Rhinoscleroma is caused by Klebsiella rhinoscleromatis, a gram negative rod that causes a chronic granulomatous infection of the upper respiratory tract. It is predominantly seen in Mexico, Central and South America. The histologic hallmark is the Mikulicz cell, a large histiocyte that contains the bacteria in its cytoplasm. Rhinosporidiosis is not an intracellular bacteria, but an extracellular fungi caused by Rhinosporidium seeberi mainly seen in India, Sri Lanka and occasionally the southeastern United States. Extremely large endospores are present in tissue. The other listed organisms are intracellular pathogens but do not have the Mikulicz cell as a feature of infection.

214
Q

215- A 24 year-old man from New York City develops fever, chills, malaise, and a severe headache. His apartment has a mouse problem. On examination, multiple, discrete papulovesicles and a single eschar are found. What is the most likely diagnosis?

A. Ehrlichiosis
B. Rickettsialpox
C. Q Fever
D. Meningococcemia
E. Rocky Mountain Spotted Fever

A

►B

The clinical description is that of rickettsialpox, which is caused by R. akari and spread by liponyssoides sanguineus - the house mouse mite.

215
Q

216- High doses of which of the following antiviral agents has been associated with thrombotic thrombocytopenic purpura in immunosuppressed patients?
A. Valcyclovir
B. Acyclovir
C. Foscarnet
D. Famciclovir
E. Cidofovir

A

►A

Thrombotic thrombocytopenic purpura has been reported with high doses of valcyclovir in immunosuppressed patients.

216
Q

217- A 42 year-old woman presents with a large, vegetating ulcer involving her left labia majora and groin for over 1 year. A Giemsa”s stained touch preparation reveals bipolar, safety pin-shaped intracytoplasmic inclusions. What is the most likely diagnosis?

A. Lymphogranuloma venereum
B. Granuloma Inguinale
C. Chancroid
D. Primary herpes simplex

E. Gonorrhea

A

►B

Bipolar, safety pin-shaped intracytoplasmic inclusions on Giemsa stain - known as Donovan bodies
- are diagnostic of granuloma inguinale.

217
Q

218- Sun exposure has been associated with the development of what type of warts?

A. Flat warts
B. Myrmecial warts
C. Common warts
D. Butcher’s warts
E. Condyloma accuminatum

A

►A

Warts are common cutaneous lesions caused by the various human papilloma virus subtypes. Flat warts are commonly caused by HPV subtypes 3, 10, 28, and 41. Sun exposure has been implicated as a potential factor in the formation of flat warts.

218
Q

219 -The culture medium that is used for Leishmaniasis is:

A. Novy-MacNeal-Nicolle
B. Giemsa
C. Sodium stibogluconate
D. FTA-ABS
E. ELISA

A

►A

Leishmaniasis should be cultured in Novy-MacNeal Nicolle (NNN) medium. Sodium stibogluconate is the treatment for cutaneous Leishmaniasis. FTA-ABS and ELISA are used to diagnose syphilis.

219
Q

220- Of the choices listed, which antiobiotic is the best to treat a 7-year old child with erythema migrans?

A. Doxycycline
B. Amoxicillin
C. Azithromycin
D. Tetracycline
E. Rifampin

A

►B

Doxycycline is the usual treatment for erythema migrans in adults in the absence of neurologic or cardiovascular complications. In children less than 8, all tetracyclines are relatively contraindicated because of they can cause tooth discoloration. Therefore amoxicillin should be used as the first line therapy for children less than 8. Note: Changes have been made to the treatment of RMSF. Children of any age should be treated with doxycycline for that disease.

220
Q

221- A patient with sparsely distributed hemorrhagic vesiculopustules with erythematous bases on palms, soles and over joint. The causative agent is Neisseria gonorrhoeae and the treatment is:

A. Ceftriaxone IV
B. Amphicillin
C. Rifampin
D. Penicillin
E. TMP-SMX

A

►A

Gonococcemia is caused by Neisseria gonorrhoeae. The clinical features are fevers, chills, arthralgias, malaise with hemorrhagic vesiculopustules with erythematous base on palms, soles and over joints.

221
Q

-A 24 year old male presents with a high fever, arthralgias, and a rash characterized by nonspecific erythematous macules in a generalized distribution. He was recently cleaning his family’s barn, and was bitten more than once by rats and mice. What is the treatment of choice for this infection?

A. Penicillin
B. Erythromycin
C. Clindamycin
D. Ciprofloxacin
E. Amikacin

A

►A

Rat-bite fever is also called Haverhill Fever. It is caused by Streptobacillus moniliformis and is acquired from infected rodents or by touching or eating contaminated food. The signs and symptoms are nonspecific, but the diagnosis can be suspected based on history. Treatment is with penicillin.

222
Q

223- A 10 year-old girl presents to the dermatology clinic with a pruritic eruption on her trunk after swimming at a beach in Florida. Of note, the lesions are limited to skin that was covered by her bathing suit. The most likely diagnosis is:

A. Seabather?s eruption
B. Swimmer?s itch
C. Cercarial dermatitis
D. Swimming pool granuloma
E. Cutaneous larva migrans

A

►A

Seabather?s eruption characteristically involves areas covered by swimwear, as a result of coelenterate larvae becoming trapped underneath bathing suits. The causative organisms are larval forms of Edwardsiella lineata (sea anemone) and Linuche unquiculata (thimble jellyfish).

223
Q

224- The mechanism of action of this virulence factor of Bacillus anthracis is via increasing the activity of tumor necrosis factor alpha:

A. Lethal toxin
B. Edema toxin
C. Polyglutamate acid capsule
D. All of the above answers are correct
E. None of the above answers are correct

A

►A

Bacillus anthracis is responsible for causing Anthrax. Anthrax of the skin is characterized by a painless edematous nodule which rapidly degenerates into an eschar. B. anthracis has 3 virulence factors: a polyglutamate acid capsule which inhibits phagocytosis of the bacteria, edema toxin (edema factor and protective factor) which results in edema secondary to induction of cAMP pathway, and lethal toxin (lethal factor and protective factor) which leads to shock and death via increasing TNFalpha activity.

224
Q

225- A 62 year-old man presents with tinnitus, facial paralysis, and vesicles of the external ear. What is the most likely diagnosis?

A. Primary Herpes Simplex infection
B. Primary Varicella Infection
C. Herpes Zoster Infection
D. Cytomegalovirus infection
E. Coxsackievirus A16 infection

A

►C

This patient has Ramsay Hunt Syndrome, herpes zoster infection of the geniculate ganglion. Vesicles can be seen on the tympanic membrane and the external ear. Symptoms include ipsilateral facial paralysis, tinnitus, or other auditory symptoms. Treatment is with acyclovir and systemic corticosteroids.

225
Q

Scrofuloderma is most commonly associated with an underlying infection of what organ system?

A. Lymph nodes
B. Spine
C. Lungs
D. Gastrointestinal tract
E. No underlying systemic disease

A

►A

Scrofuloderma is a tuberculous or nontuberculous mycobacterial infection affecting children and young adults, representing direct extension of tuberculosis into the skin from underlying structures such as lymph nodes, bone, or lung. It most commonly presents as a neck abscess secondary to a tuberculosis infection of the cervical lymph nodes. It is manifested by the development of painless subcutaneous swellings that evolve into cold abscesses, multiple ulcers, and draining sinus tracts.
The diagnosis is based on positive Mantoux test/PPD test, histologic
features of caseating granulomatous inflammation, culture, and favorable response to anti tuberculosis therapy.

226
Q

227- An 18-year old man presents to the Dermatology Clinic with a nontender penile erosion that has been present for 2 weeks. An indurated border and nontender bilateral inguinal lymphadenopathy are also noted. What is the most likely diagnosis?

A. Primary syphilis
B. Chancroid
C. Herpes simplex
D. Lymphogranuloma venereum

E. HIV

A

►A

A painless chancre with an indurated border is characteristic of primary syphilis. Associated painless lymphadenopathy (“buboes”) is also a common feature.

227
Q

228- A 40-year-old male patient from South America with HIV (CD4+ <200) presents with new purple-red papules and nodules on his face that have been progressively enlarging. A biopsy is performed, showing lobular vascular proliferations with plump endothelial cells on H&E staining. Immunohistochemical staining of skin tissue for human herpes virus 8 (HHV8) is negative. Upon closer inspection of H&E-stained sections, faint purple granular interstitial deposits are noted in the dermis. Which of the following tissue stains is most appropriate for further work-up?

A. Fite
B. Chloroacetate esterase
C. Warthin-Starry
D. Giemsa
E. Ziehl-Neelsen

A

►C

Causes of angiomatous papulonodules arising in immunocompromised patients include Kaposi‟s sarcoma, bacillary angiomatosis, and verruga peruana, as well as pyogenic granulomas and cherry angiomas. Bartonella, a gram negative bacillus, appears as faint purple-blue granular dermal deposits on routine H&E-stained sections. These deposits appear black upon WarthinStarry staining. Prominent endothelial cells may also be seen. Both B. henselae and B. quintana have been implicated in the pathogenesis of bacillary angiomatosus, while B. bacilliformis is the causative agent in verruga peruana (as well as Bartonellosis). Erythromycin is the first -line treatment for bacillary angiomatosus, while fluoroquinolones plus a tetracycline are the first-line agents for treatment of verruga peruana.

228
Q

229- Bacillary angiomatosis is caused by which of the following organisms?

A. B. quintana
B. B. henselae
C. B. bacilliformis
D. B. quintana and B. henselae
E. All of these answers are correct

A

►D
B. quintana or B. henselae are both causative organisms for bacillary angiomatosis.

229
Q

230- One of your colleagues returned from a vacation to India with fever, vomiting, pleural effusions, ascites and conjunctival petechiae. She also has a diffuse macular rash with notable areas of sparing that your attending refers to as “white islands in a sea of red”. What is your diagnosis?

A. Leishmania
B. Dengue hemorrhagic fever
C. Malaria
D. Typhoid
E. Leptospirosis

A

►B

Dengue fever is caused by an Arbovirus and transmitted by the mosquito, Aedes aegypti and may cause Dengue Shock Syndrome and Dengue Hemorrhagic Fever. The infection starts with sudden onset of high fever, backache, retroorbital pain, bone and joint pain, depression and malaise. The disease is also called “break-bone fever.” One to seven days after onset of fever, rash presents characteristically starting on the dorsum and hands and feet spreading to limbs and torso. The eruption may become confluent with small, round islands of sparing, the so called “white islands in a sea of red.” Treatment is generally supportive as no antivirals are effective.

230
Q

231- Acrodermatitis chronica atrophicans is typically caused by:

A. Borrelia burgdorferi
B. Borrelia burgdorferi senso stricto

C. Borrelia garinii
D. Borrelia afzelii
E. Unknown

A

►D

Acrodermatitis chronica atrophicans, also known as primary diffuse atrophy, is characterized by the appearance on the extremities of diffuse reddish or bluish red, paper-thin skin allowing the blood vessels to easily be seen beneath the skin. It is caused by Borrelia afzelii and is tick transmitted by Ixodes ricinus. It is seen almost exclusively in Europe.

231
Q

232- Patients that are diagnosed with purpura fulminans with hemorrhagic infarction of the skin are infected with:
A. Group A streptococcus
B. Staphylococcus aureus
C. Francisella tularensis
D. Bacillus anthracis
E. Streptomyces somaliensis

A

►A

Purpura fulminans is causes by group A streptococcal infection. Clinically there are hemorrhagic infarction of the skin caused by disseminated intravascular coagulation. One can see symmetric large ecchymotic areas with irregular (“geographic”) borders on extremities, ears and nose most commonly.

232
Q

233- A 67-year-old man presents with grouped painful vesicles on an erythematous base on his lateral sacrum. He denies any history of herpes simplex type I or II. He is angry with you for suggesting that he may have herpes. What could explain this presentation?

A. He actually has “shingles” and your diagnosis is incorrect
B. A significant number of people are asymptomatic for HSV infection
C. He is lying

D. He has the early stages of Alzheimer‟s disease
E. This presentation represents a primary infection with Epstein Barr Virus, (HHV4)

A

►B

A significant number of people are asymptomatic for HSV infection but are seropositive for anti HSV antibodies. Lumbosacral HSV occurs in about 10% of HSV infections and is more common in older age groups. In some cases, this may be the “first” presentation of HSV that the patient is aware of. Shingles would be a reasonable consideration for the differential diagnosis and a direct immunofluorescence scraping and viral culture would confirm that this typical presentation of
HSV actually is. It is possible that the patient is lying or that he has early Alzheimer‟s disease, but less likely. EBV does not present in this manner.

233
Q

234- An 18 year old male presents complaining of pruritus, erythema, and edema of the hands and feet. A serum study for parvovirus antibodies is negative. What is the next most common cause of this syndrome?

A. Epstein-Barr virus
B. Measles virus
C. Coxsackie virus
D. Cytomegalovirus
E. Human herpes virus 6

A

►A

Papular-purpuric gloves and socks syndrome is characterized by pruritus, erythema, and edema of the hands and feet, primarily in teenagers and young adults. Parvovirus B19 is classically associated with this eruption. While all of the viruses listed as answer choices have been associated with this syndrome; Epstein-Barr virus is the best answer choice.

234
Q

235- A middle-aged female presents with purplish pruritic plaques on her dorsal wrist and whitish plaques on her buccal mucosa. A diagnosis of lichen planus is made. Which type of lichen planus is most strongly associated with Hepatitis C virus infection?

A. Mucosal ulcerative lichen planus
B. Hypertrophic lichen planus
C. Lichen planus pemphigoides
D. Lichen planus of the nails
E. Vulvar lichen planus

A

►A

Lichen planus presents a wide variety of cutaneous morphologies that share common histologic findings of a lichenoid lymphocytic infiltrate, basal vacuolization, and dyskeratotic keratinocytes. The form of lichen planus most strongly associated with Hepatitis C virus infection is mucosal ulcerative lichen planus.

235
Q

236- Pediculus humanus var. corporis (human body louse) is the vector in which of the following diseases:

A. Endemic typhus (R. typhus)
B. Epidemic typhus (R. prowazekii)
C. Q fever (C. burnetii)
D. Rickettsialpox (R. akari)
E. Rocky Mountain Spotted Fever (R. rickettsii)

A

►B

The human body louse is the implicated Trench fever, epidemic typhus, and relapsing fever. Trench fever usually affects alcoholic men and is manifested by fevers. Treatment is with ceftriaxone, erythromycin, or doxycycline. Epidemic typhus is manifested by fevers, chills, malaise, and a pink macular eruption beginning in the axilla and trunk. Treatment is with tetracycline or chloramphenicol. Relapsing fever is manifested by paroxysmal fevers, headache, lymphocytoma, and erythematous/petechial macules on trunk/extremities. Treatment is with doxycycline.

236
Q

237- Which of the following are inconsistent with the diagnosis of staphylococcal scalded skin syndrome?

A. Epidermal changes are produced by exfoliative toxins of staphlococcus
B. Initial event is usually a localized staph infection
C. Swabs and cultures of fluid filled blisters overwhelmingly grow staph
D. Prognosis is good in children with low mortality when anitbiotics are administered early
E. Cell necrosis does not occur in staphylococcal scalded skin syndrome as it does in T.E.N

A

►C

Staphylococcal scalded skin syndrome: Lesions extend far beyond areas of actual staphylococcal infection, by action of the epidermolytic exotoxin elaborated by the staphylococcus in remote sites. Usually the staphylococci are present at a distant focus such as the parynx, nose ear, or conjuctiva. If cultures are taken they should be obtained fromthe mucous membranes because the skin erythema and desquamation is due to the distant effects of the exfoliative toxin, unlike the sitaution in bullous impetigo, where S. aureus is present in the lesions. Epidermal changes are produced by exfoliative toxins of Staphylococcus. Inital event is usually a localized Staph infection. Prognosis is good in children with low mortality when anitbiotics are administered earyl. Cell necrosis does not occur in SSS as it does in toxic epidermal necrolysis.

237
Q

238- Milker‟s nodule is caused by infection with which of the following viruses?

A. Human herpesvirus-6 (HHV-6)
B. Human herpesvirus-8 (HHV-8)
C. Paravaccinia virus
D. Vaccinia virus
E. Orf virus

A

►C

Milker‟s nodule is caused by paravaccinia virus, a poxvirus of the genus Parapoxvirus. It is transmitted to humans from infected cows.

238
Q

239 -In addition to Kaposi”s sarcoma, HHV-8 infection is also the causative agent in:

A. Pityriasis rosea
B. Angiosarcoma
C. Primary effusion lymphoma
D. Bacillary angiomatosis
E. Pyogenic granuloma

A

►C

Human herpes virus 8 is a double stranded DNA virus. It is thought to be pathogenic in Kaposi”s sarcoma, primary effusion lymphoma, and Castleman”s disease. Primary effusion lymphoma is a rare B-cell lymphoma seen predominantly in patients with AIDS.

239
Q

240 -WHO recommendations for the treatment of multibacillary leprosy include all of the following EXCEPT:

A. Dapsone
B. Clofazimine
C. Clarithromycin
D. Rifampin
E. All of these answers are recommended for the treatment of multibacillary leprosy.

A

►C

Clarithromycin is not part of the WHO recommendations for the treatment of multibacillary leprosy.

240
Q

241 -Granuloma Inguinale is caused by:

A. Chlamydia trachomatis types I, II & III
B. Hemophilis ducreyi
C. Klebsiella granulomatis

D. Treponema pertenue
E. Treponema pallidum

A

►C

Granuloma inguinale is caused by an infection of Klebsiella granulomatis. It is a granulomatous skin disease of the genitals caused by sexual transmission. It is a gram negative rod that is similar to the Enterobacteriaceae. It is a smll, raised papule or subcutaneous nodule that rapidly ulcerates, rarely with lymphadenopathy. It is not self healing and spreads by radial extension.
Hemophilis ducreyi is the cause of chancroid, Chlamydia trachomatis causes Lymphogranuloma Venereum, Treponema pertenue causes Yaws and T. pallidum is the cause of syphilis.

241
Q

242 -A patient is diagnosed with tularemia. The clinical features seen is the ulceroglandular form that looks like a chancre like ulcer with raised boarders and regional lymphadenopathy seen on t he finger or hand. The patient mostly likely came into contact with:

A. Rabbits
B. Horses
C. Cats
D. Dogs
E. Deer

A

►A

Tularemia is caused by Francisella tularensis. It is most likely caused by infected rabbits seen in hunters. There are other forms such as glandular, chancriform, oculoglandular, pulmonary, oropharyngeal and meningeal. The treatment is streptomycin.

242
Q

243 -What is the definition of disseminated herpes zoster?

A. Involvement of two or more defined dermatomes
B. Symmetric bilateral involvement
C. Involvement of one cranial and one spinal nerve.

D. More than 20 vesicles outside of the primarily affected dermatome
E. Involvement of the trunk and at least one extremity.

A

►D

The definition of dissemianted herpes zoster is at least 20 vesicles seen outside of the primarily affected dermatome. The other option choices are distractors.

243
Q

244 -Each of the following species may be involved in necrotizing fasciitis except:

A. Pseudomonas
B. Clostridium
C. Streptococcus
D. Mycobacterium
E. Bacteroides

A

►D

Necrotizing fasciitis is a rapidly advancing acute necrotizing infection that may follow trauma, surgery, or occur spontaneously. It is associated with systemic toxicity and high mortality rat e. Clinically, erythema, edema, and edema progresses to dusky cyanosis, blistering and necrosis. Many virulent bacteria have been culturesd including Pseudomonas, Bacteroides, streptococci, staphylococcus, enterococci, and clostridium. Both aerobic and anaerobic cultures should be taken. Mycobacterium is not associated with necrotizing fasciitis.

244
Q

245- The usual culture medium for Mycobacterium leprae is:

A. Chocolate agar in 10% CO2
B. Lowenstein-Jensen
C. It cannot be cultured
D. Sheep‟s blood agar
E. Agar supplemented with heme and nicotinamide

A

►C

M. leprae cannot be grown in media or cell culture. It has been grown in mice footpads and in armadillos. Humans are the only natural host. It is endemic in the tropics and found worldwide. The Lowenstein-Jensen medium is used to culture mycobacteria most commonly. Neisseria gonorrheae is grown on chocolate agar in 10% CO2. Agar supplemented with heme (x-factor) and nicotinamide (v-factor) is needed for Hemophilus influenzae cultures. Sheep‟s blood agar is useful for identifying hemolytic strains of streptococcus and staphylococcus.

245
Q

246- A patient presents with multiple asensate patches on the abdomen. A diagnosis of leprosy is made. Which of the following regimens is the WHO recommendation for treatment of paucibacillary leprosy?

A. Dapsone and rifampin
B. Dapsone and clofazimine
C. Clofazimine and rifampin
D. Minocycline and rifampin
E. Minocycline and clofazmine

A

►A
Paucibacillary leprosy is characterized by three or fewer anesthetic and anhidrotic lesions. This form of leprosy develops when the body mounts a Th1 cytokine profile leading to rare bacilli found in the lesions. The WHO recommends treatment of paucibacillary leprosy with dapsone 100 mg daily for 6 months and rifampin 600 mg monthly for 6 months.

246
Q

247- Hutchinson”s teeth are a feature of which stage of syphilis?

A. Early congenital
B. Late congenital
C. Primary
D. Secondary
E. Tertiary

A

►B

Hutchinson”s teeth (widely-spaced, peg-shaped upper incisors) are a seen in Late Congenital Syphilis (> 2 years of age).

247
Q

248- A patient with HIV/AIDS develops disseminated flesh-colored papules with central umbilication. A diagnosis of molluscum contagiosum is made. What is the CD4 cell count below which molluscum contagiosum lesions are first seen in HIV patients?

A. 100
B. 200
C. 400
D. 800
E. 1000

A

►A

HIV/AIDS patients are afflicted with a wide variety of opportunistic pathogens. The spectrum of infectious diseases can be used to predict a patient’s CD4 helper T cell count. Molluscum contagiosum is usually not seen until a patient’s CD4 count drops below 100.

248
Q

249- Swimmer’s itch caused by the flat worms of the family Schistosomatidae have clinical features of:
A. pruritic papules and vesicles on uncovered skin
B. pruritic erythematous papules on covered swim wear
C. Serpiginous urticarial plaques on buttocks
D. Thumbprint purpura on periumbilical skin
E. Widespread petechiae on trunk/proximal extremities

A

►A

Swimmers itch also known as Clam Digger’s itch is caused by the cercarial forms flatworms of the family Schistosomatidae. The mode of infection is penetration of skin while bathing in infested

fresh or salt water in Northern United States and Canada. The clinical characteristics are pruritic papules and papulovesicles on uncovered skin.

249
Q

250- What is the first line treatment of streptococcal toxic shock syndrome?

A. Penicillin
B. Ciprofloxacin
C. Clindamycin
D. Doxycycline
E. Vancomycin

A

►C
Clindamycin is considered 1st line treatment of streptococcal toxic shock syndrome, as it inhibits secretion of the exotoxin.

250
Q

251- A 12 month old boy has the sudden onset of a fever of 40 degrees Celsius. When the fever breaks spontaneously in three days, an erythematous, morbilliform rash arises on the trunk and spreads to the arms and legs. The rash is not itchy, and lasts 1-2 days. What is the most likely viral cause of this syndrome?

A. HHV-6
B. Measles virus
C. Rubella virus
D. Coxsackie virus
E. Cytomegalovirus

A

►A

Roseola infantum is also known as exanthem subitum or sixth disease. It is seen in infants ages 6-36 months, and is characterized by the signs and symptoms as described. The syndrome is caused by Human Herpes Virus 6.

251
Q

252- Xenopsylla cheopis transmits:

A. Epidemic typhus
B. Murine typhus
C. Rickettsia prowazekii
D. Scrub typhus
E. Rickettsia akari

A

►B

Xenopsylla cheopis along with Ctenocephalides felis are fleas that transmit R. typhi, the organism responsible for murine or endemic typhus. The body louse or Pediculus humanus corporis transmits Rickettsia prowasekii the organism responsible for epidemic typhus. Scrub typhus is caused by R. tsutsugamushi and is transmitted by chiggers or trombiculid mite larvae.

252
Q

253- A 25-year old man with a history of a healed genital ulcer develops a diffuse papulosquamous eruption resembling pityriasis rosea. What is the most likely time lapse between the appearance of the genital lesion and the appearance of this eruption?

A. 3-12 weeks
B. 1-2 weeks
C. 3-5 months
D. 6-12 months
E. 1-2 years

A

►A

The lesions of secondary syphilis are often diffusely distributed due to spirochetemia. Papulosquamous lesions are characteristic, and the presence of constitutional symptoms such as fever, fatigue, headaches, and bone pain may help distinguish secondary syphilis from pityriasis rosea. Other lesions of secondary syphilis include ˜moth-eaten” alopecia, mucous patches, split papules, and condyloma lata. The mucocutaneous manifestations of secondary syphilis typically occur 3-12 weeks after the appearance of the chancre and last 4-12 weeks.

253
Q

254- Through which cellular protein does molluscum contagiosum evade host immunity?

A. IL-18 binding protein
B. Beta-4 integrin
C. T cell receptor
D. MHC II
E. Toll-like receptor 7

A

►A

Molluscum contagiosum virus, like other viral and bacterial pathogens, attempts to exploit the host immune system to avoid destruction. In this case, molluscum contagiosum virus usesIL-18 binding protein, which blocks the host helper T (Th1) response by reducing local IFN-gamma production.

254
Q

255- Which of the following is the vector responsible for the transmission of Verruga Peruana?

A. Lutzomyia verrucarum
B. Tsetse fly
C. Ctenocephalides felis
D. Pediculus humanus corporis
E. Simulium slossonae

A

►A

Verruga Peruana is caused by infection with B. bacilliformis, which is transmitted by the sandfly, Lutzomyia verrucarum.

255
Q

256- The causative organism of epidemic typhus is:

A. Francisella tularensis
B. R. prowazekii
C. R. akari
D. R. typhi

E. Salmonella typhi

A

►B

Epidemic typhus is caused by infection with R. prowazekii.

256
Q

257- Dracunculiasis is typically acquired from:

A. Soil
B. Salt water
C. Drinking water
D. Chiggers
E. Black flies

A

►C

Dracunculiasis, which is caused by Dracunculus medinensis, is acquired by ingestion of copepod- infested water.

257
Q

258- A young patient presents to you after participating in a bar brawl a few nights before with a painful swollen erythematous right hand. The most likely organism is:

A. Staphylococus aureus
B. Eikenella corrodens
C. Pasturella multocida
D. Streptococcus pyogenes
E. Bacteriodes fragilis

A

►B

Eikenella corrodens is a member of normal oral flora, that is commonly the cause of human bite and fist fight infections. Pasturella multocida is a common organism found in dog and cat bites. Staph

and Strep could be the right answers for a cellulitis, but with this history are not the correct choices. Baceriodes fragilis is found in normal bowel flora.

258
Q

259- Which of the following manifestations of syphilis would you expect to be present 2-6 months after the individual was exposed?

A. Osteitis
B. Aortitis
C. Tabes dorsalis
D. Condylomata lata
E. Pseudochancre redux

A

►D

All of the listed options except condylomata lata are manifestations of tertiary syphilis, which does not manifest for years after the initial infection. Secondary syphilis presents 2 -6 months following infection. Condylomata lata are moist, flat and smooth. Do not confuse them with Condyloma accuminatum which are usually dry, cauliflower-like projections on the genitalia.

259
Q

260- Which of the following forms of syphilis is characterized by ham-colored macules on the palms and soles, condyloma lata, and “moth-eaten” alopecia?

A. Primary
B. Secondary
C. Latent
D. Tertiary
E. Congenital

A

►B

Secondary syphilis is characterized by ham-colored macules on the palms and soles, condyloma lata, and “moth-eaten” alopecia, in addition to mucous patches and split papules. It typically lasts 4-12 weeks.

260
Q

261- The treatment of choice for acyclovir-resistant herpes simplex virus infection is:

A. Valcyclovir
B. Famciclovir
C. Indinivir
D. Saquinivir
E. Foscarnet

A

►E

Foscarnet directly inhibits viral DNA polymerase (without requiring phosphorylation by TK) and is therefore effective in acyclovir-resistant HSV infections. Cidofovir can also be used.

261
Q

262- What is the characteristic body on histopathology for this disease?

A. Cowdry Type A
B. Cowdry Type B
C. Henderson-Patterson
D. Guarnieri
E. Dutcher

A

►A

Herpes simlex virus is characterized by Cowdry type A bodies on histopathology. They appear as sharply demarcated eosinophilic structures separated by a clear halo from a basophilic rim of the infected cell’s marginated nuclear chromatin. Cowdry B bodies may be seen in polio infections.

262
Q

263- A 10-year-old boy develops an acute illness and rash along with marked enlargement of the posterior cervical lymph nodes. This presentation is most consistent with:

A. West African sleeping sickness
B. East African sleeping sickness

C. Chagas disease
D. Schistosomiasis
E. Sparaganosis

A

►A

West African sleeping sickness is caused by Trypanosoma bruci gambiense and leads to acute illness with rapid fatal course and pronounced posterior cervical lymphadenopathy (Winterbottom’s sign). East African sleeping sickness is due to T. brucei rhodesiense (east like rhode island) and is more chronic . American trypanosomiasis, or Chagas disease, features unilateral conjunctivitis and edema of the face (Romana’s sign) and heart and gastrointestinal sequelae. Schistomsomiasis causes swimmer’s itch, while sparaganosis from ingestion of Spirometra leads to painful edematous lumps.

263
Q

264- A patient develops gastroenteritis after a trip to the seacoast. He reports eating raw oysters at his favorite seafood restaurant. Which of the following organisms is the most likely cause of his gastrointestinal problems?

A. Vibrio vulnificus
B. Vibrio parahemolyticus
C. Mycobacterium marinum
D. Erysipelothrix rhusiopathiae
E. Treponema carateum

A

►B

Vibrio parahemolyticus is a cause of gastroenteritis after consuming contaminated seafood or shellfish. V. vulnificus infection occurs in cuts/wounds that contact contaminated seawater or marine organisms. M. marinum occurs after injury in an aquatic environment such as a fish tank or swimming pool. Lesions typically occur on hands, elbows or knees and are characterized by painless inflammatory nodules that may ulcerate and may occur in a sporotrichoid pattern. E. rhusiopathiae infection occurs after injuries in fishermen, butchers or those handling raw meat. Initially there is burning pain at the inoculation site followed by a gradually enlarging purple plaque with well defined margins. T. carateum is the cause of Pinta.

264
Q

A patient with a chronic nose-bleed is treated with nasal packing. The packing is accidentally left in place, and he develops Toxic Shock Syndrome. What is the implicated toxin?

A. Enterotoxin B
B. Toxic Shock Syndrome toxin 1
C. Exfoliative Toxin A
D. Edema toxin
E. Lethal factor

A

►A

Toxic Shock Syndrome is characterized by fever, hypotension, and a generalized scarlatiniform eruption followed by desquamation. Menstrual-associated TSS is generally driven by Staphylococcus aureus, which released Toxic Shock Syndrome toxin 1. In nonmenstrual cases, however, Enterotoxins B>C are more commonly implicated, although TSST-1 has also been associated. Exfoliative toxins A and B are involved in bullous impetigo, and edema toxin and lethal factor are involved in Bacillus anthracis infections.

265
Q

266- What rare hematologic disorder can accompany varicella-zoster virus infection?

A. Thrombocytopenia
B. Neutropenia
C. Lymphopenia
D. Lymphocytosis
E. Anemia

A

►A

Symptomatic thrombocytopenia, while rare, has been associated with acute varicella-zoster virus infection.

266
Q

267- Which anatomic locations are most commonly involved in herpes gladiatorum?

A. Lateral neck, lateral face, forearm
B. Tips of fingers
C. Feet
D. Upper thighs and buttocks
E. Scalp

A

►A

Herpes gladiatorum is most commonly seen on the lateral neck, lateral face, and forearm, areas which most commonly contact a competing wrestler or the wrestling mat.

267
Q

268- Post herpetic neuralgia can be prevented with the VZV vaccine. It is a Oka/Merck strain of VZV and is FDA approved for adults:

A. 60 and older
B. 50 and older
C. 55 and older
D. 65 and older
E. 70 and older

A

►A

Herpes zoster vaccine is a live attenuated vaccine and can help reduce the incidence of herpes zoster and decrease the chance of post-herpetic neuralgia.

268
Q

269- A 35 year-old woman who works in the animal product industry presents with a painless hemorrhagic bulla. A diagnosis of anthrax is suspected. Gram stain of the vesicle fluid should reveal which of the following?

A. Guarnieri bodies
B. Gram-negative cocci
C. Gram-negative bacilli

D. Gram-positive cocci
E. Gram-positive bacilli

A

►E

Anthrax is caused by Bacillus anthracis, which is a gram-positive spore-forming rod.

269
Q

270- What is the most important predictor of materal-fetal transmission of HSV-2?

A. Use of pre-natal antiviral medications
B. Length of delivery
C. Nature of maternal infection
D. Nutrition status of the mother
E. Use of peri-natal monitoring equipment.

A

►C

The nature of the maternal infection (primary versus recurrent) is the most important predictor of maternal-fetal transmission of HSV-2. In mothers with a primary infection, the risk of transmission is 25-50%. In mothers experiencing a recurrent infection, the risk is 2-5%

270
Q

271 -Contagious pustular dermatitis (Orf) is caused by a:

A. Herpesvirus
B. Poxvirus
C. Gram-positive spore-forming rod
D. Paramyxovirus
E. Papovavirus

A

►B

Orf is caused by orf virus (OV), a poxvirus of the genus Parapoxvirus.

271
Q

272- The treatment of choice of cat-scratch disease is:

A. Azithromycin
B. Ceftriaxone
C. Trimethoprim-sulfamethoxazole
D. Itraconazole
E. Mebendazole

A

►A

Cat-scratch disease is caused by Bartonella henselae, which is spread among cats by fleas. The hallmark of the disease is unilateral and regional lymphadenopathy. The treatment of choice for cat- scratch disease is Azithromycin.

272
Q

273- The most likely target for exfoliative toxin A in bullous impetigo is:

A. Desmocollin
B. BPAG1
C. BPAG2
D. Desmoglein 1
E. Desmoglein 3

A

►D

Bullous impetigo is caused primarily by staphylococcus aureus phage type 71. This bacteria produces exfoliative toxin A binds to desmoglein 1 and produces produces the blisters characteristic of bullous impetigo.

273
Q

274- Which Rickettsial infection has a negative Weil-Felix test?

A. Rocky Mountain Spotted Fever (R. rickettsii)
B. Mediterranean fever (R. conorii)
C. Epidemic typhus (R. prowazekii)

D. Endemic typhus (R. typhi)
E. Rickettsialpox (R. akari)

A

►E

Rickettsia are short, gram-negative rods which are strict intracellular parasites. The bacteria is transmitted via tick which much be attached for more than 6 hours. The Weil-Felix test exploits cross-reactivity between the Proteus vulgaris antigen and RIckettsia.

274
Q

275- Most common causative organism of the attached picture is

A. Corynebacterium minutissimum
B. Corynebacterium tenuis
C. Micrococcus sedentarius
D. Staphylococcus
E. Streptococcus

A

►C

The image shows Pitted keratolysis. It is a non-inflammatory bacterial infection caused by microcolonies of Micrococcus sedentarius or Corynebacterium species that produce a specific proteolytic enzyme which digests the stratum corneum. It consists of small crater-like depressions in the stratum corneum, and usually appear on the weight-bearing regions of the soles of the feet, but can occur in other non weight-bearing areas as shown in image. This was confirmed with shave biopsy of epidermis that is step-sectioned and stained with methenamine silver as this is found to be more helpful than a punch biopsy. The disease usually goes unnoticed by patients and is most often an incidental finding. Hyperhidrosis and malodor of the skin are very common findings. Corynebacterium minutissimum is the causative agent for erythrasma, and Corynebacterium tenius causes trichomycosis Axillaris.

275
Q

276- All of the following are features of scarlet fever except:

A. Pharyngitis

B. Pastia‟s lines
C. S. aureus infection
D. Circumoral pallor
E. Sandpaper-like texture

A

►C

Scarlet fever is primarily a disease of children with most cases occurring between the ages of 1 and 10 years of age. Streptococcus pyogenes is the causative organism. It produces the
streptococcus pyrogenic exotoxin which elicits the cutaneous manifestations by enhancing delayed type hypersensitivity to streptococcal products. Clinical findings include fever, sore throat, headache, chills, sandpaper rash on the trunks, strawberry tongue, and Pastia‟s lines (linear petechial streaks found in flexural locations).

276
Q

277- What is the most common cause of erythema multiforme?

A. Herpes simplex virus
B. Mycoplasma pneumonia
C. Amoxicillin
D. Ibuprofen
E. Cytomegalovirus

A

►A

The most common cause of erythema multiforme (EM) is herpes simplex virus, which may not be active at the time of the EM eruption. Patients with recurrent EM are typically treated with acyclovir or valacyclovir. Mycoplasma pneumonia is a cause of EM, but is not the most common. Amoxicillin, ibuprofen, and cytomegalovirus may cause EM, but are not as common.

277
Q

278- The patient in this photograph has claw hands, hypoesthesia of the distal extremities, madarosis and leonine facies. What type of leprosy does she have?

A. Lepromatous leprosy
B. Tuberculoid leprosy
C. Borderline leprosy
D. Erythema nodosum leprosum
E. Lucio phenomenon

A

►A

Hansen’s disease (aka Leprosy) is a chronic granulomatous disease principally affecting the skin and peripheral nervous system, caused by Mycobacterium leprae. Lepromatous leprosy is described where there is low immune response to the bacteria. The areas most commonly affected are the superficial peripheral nerves, skin, mucous membranes of the upper respiratory tract, anterior chamber of the eyes, and testes. These areas tend to be cool parts of the body. Tissue damage depends on the degree to which cell-mediated immunity is expressed, the type and extent of bacillary spread and multiplication, the appearance of tissue-damaging immunologic complications (ie, lepra reactions), and the development of nerve damage and its sequelae. The late complications of lepromatous leprosy include madarosis = loss of eyebrows, leonine facies, stockin gglove neuropathy and lagopthalmosis.

278
Q

279- This is the term that is used from tuberculosis of the skin and is spread by hematogenous, lymphatic, or contiguous spread. The clinical characteristic is brownish red plaque, apple jelly color on diascopy.

A. Lupus vulgaris
B. Scrofuloderma
C. Tuberculous Gumma
D. Tuberculosis verrucosa cutis
E. Tuberculosis cutis orificialis

A

►A

Lupus vulgaris spreads from hematogenous, lymphatic, or contiguous spread from distant site of tuberculous infection. It is sensitizes the host with moderate to high immunity. It is pauci bacillarry. It also involved the head/neck in 90% of cases.

279
Q

280- Which of the following diseases is caused by a virus?

A. Rocky Mountain spotted fever
B. Human monicytic ehrlichiosis
C. Colorado tick fever
D. Boutonneuse spotted fever
E. Q fever

A

►C

Rocky Mountain spotted fever is caused by Rickettsia rickettsii. Human monicytic ehrlichiosis is caused by Ehrlichia chaffeensis. Colorado tick fever is caused by Reoviiridae. Boutonneuse fever is caused by Rickettsia conorii. Q fever is caused by Coxiella burnetti.

280
Q

281- Which of the following statements best describes the pathogenesis of scrofuloderma?

A. Hematogenous spread of M. tuberculosis from a distant site of infection
B. Autoinoculation of M. tuberculosis from advanced pulmonary tuberculosis
C. Contiguous spread from an underlying focus of tuberculous lymphadenitis
D. Primary incoculation of M. tuberculosis in a sensitized host
E. An cutaneous manifestation of military tuberculosis

A

►C

Scrofuloderma is the result of contiguous spread onto skin from an underlying focus of tuberculous infection. This typically occurs in a sensitized host with low immunity.

281
Q

282- Which of the following tests is a specific test for infection with Treponema pallidum?

A. RPR
B. VDRL
C. FTA-ABS
D. CBC
E. Darkfield microscopy

A

►C

The FTA-ABS test measures the presence of antibodies in the patient‟s serum to the syphilis antigens, either on spirochetes or on attached red blood cells. The RPR and VDRL are non specific tests that rely on cross-reactive anti-phospholipid antibodies. Darkfield microscopy is not specific to
T. pallidum, but will identify other spirochetes also. A CBC would not be specific for T. pallidum.

282
Q

283- What percentage of patients undergoing stem cell transplantation for leukemia will develop herpes zoster within the first year?

A. 5%
B. 30%
C. 50%
D. 70%
E. 90%

A

►D

The risk of developing herpes zoster following stem cell transplantation for leukemia is quite high, approaching 70% in some studies.

283
Q

284- Herpes simplex virus infection and this condition has been linked to which HLA type?

A. HLA Cw6
B. HLA B15

C. HLA B27
D. HLA B51
E. HLA DR3

A

►B

HSV related erythema multiforme has been associated with an increased frequency of HLA-B15.

284
Q

285 -Heck‟s disease is associated with which of the following types of human papillomavirus?

A. 1
B. 3
C. 7
D. 6/11
E. 13/32

A

►E

Heck‟s disease (focal epithelial hyperplasia) is caused by HPV 13 and 32 infections. These lesions are multiple circumscribed papules on the oral mucosa that may resemble oral condyloma. They are not pre-cancerous. HPV-1 is associated with plantar warts, while HPV-3 is associated with flat warts. HPV-7 cause “butcher‟s warts” and are believed to be acquired from handling meat, poultry and fish.

285
Q

286- What is the first-line recommended therapy for orf?

A. Penicillin
B. Erythromycin
C. Doxycycline
D. Bactrim
E. No therapy is needed since the condition is self-resolving

A

►E

No therapy is needed, since orf is usually a benign, self-resolving infection. Orf is caused by a parapoxvirus, often associated with sheep and goats. The infection undergoes 6 different stages of evolution: the papular, target, acute, regenerative, papillomatous, and regressive stages.

286
Q

287- A patient with hypertophic plaques on the external nares with Mikulicz cells seen on histopathology is infected with:
A. Klebsiella pneumoniae rhinoscleromatis
B. Streptobacillus moniformis
C. Erysipelothrix rhusiopathiae
D. Bukholderia mallei
E. Pseudomonas aeruginosa

A

►A

This patient has rhinoscleroma, which looks like hypertrophic plaques on external nares. The causative organism is Klebsiella pneumoniae rhinoscleromatis. Mikulicz cells are seen on histopathology and the treatment is ciprofloxacin.

287
Q

288- This is the cause of bacillary angiomatosis:

A. B. burgdorferi
B. B. henselae and B. quintana
C. B. bacilliformis
D. B. dammini
E. B. pacificus

A

►B

Bacillary angiomatosis is caused by both B. henselae and B. quintana. The vector is unknown and the treatment is erythromycin and doxycycline.

288
Q

289- The usual culture medium for mycobacteria is:

A. Chocolate agar in 10% CO2
B. Lowenstein-Jensen
C. New York City
D. Sheep”s blood agar
E. Agar supplemented with heme and nicotinamide

A

►B

The Lowenstein-Jensen medium is used to culture mycobacteria most commonly. Neisseria gonorrheae is be grown on chocolate agar in 10% CO2. Thayer “Martin, Martin-Lewis or New York City medium also allow for N. gonorrheae growth. Agar supplemented with heme (x factor) and nicotinamide (v-factor) is needed for Hemophilus influenzae cultures. Sheep”s blood agar is useful for identifying hemolytic strains of streptococcus and staphylococcus.

289
Q

290- Herpangina is caused by which of the following?

A. Group A coxsackievirus
B. Epstein Barr Virus
C. Parvovirus B19
D. Human Herpesvirus 6 (HHV-6)
E. Human Herpesvirus 6 (HHV-7)

A

►A

Herpangina is caused by Group A coxsackievirus.

290
Q

291 -What is the most common site of infection from Streptococcus Iniae?

A. Lower leg
B. Face
C. Hand
D. Foot
E. Nails

A

►C

Streptococcus iniae most commonly causes bacteremic cellulitis of the hand in persons who have recently handled fresh fish. S. iniae is a fish pathogen that causes sporadic infection in tilapia, yellowtail, rainbow trout, and coho salmon. Treatment of choice is with penicillin antibiotics.

291
Q

292- An HIV infected male, presents with a two-week history of a painless 2 cm ulcer on the penile shaft, with associated inguinal lymphadenopathy. What is the best treatment?

A. Valacyclovir
B. Benzathine penicillin G
C. Ceftriaxone
D. Doxycycline
E. Azithromycin

A

►B

A painless ulcer in an HIV patient is most likely a syphilitic chancre. The treatment of choice is benzathine penicillin G in patients without a penicillin allergy. Alternative agents in allergic patients include doxycycline, tetracycline, erythromycin, and ceftriaxone. Valacyclovir would be the treatment of choice for genital herpes, which is typically painful. Azithromycin in a single 1 g oral dose is effective for chancroid, which is painful.

292
Q

293- All of the following statements are true of Bacillus anthracis infection except:

A. Spores remain stable for decades
B. 20% mortality rate in untreated cutaneous infections
C. Inhalation, GI and cutaneous forms exist
D. Ciprofloxacin and doxycycline are first line treatments
E. All statements are true

A

►E

Anthrax can present in three different clinical forms: cutaneous, inhalational, and gastrointestinal.
The cutaneous form appears as a painless vesicle that later forms an eschar
(malignant pustule). The cutaneous form has a mortality rate of less than 1% if treated an up to 20% if untreated.

293
Q

294- This animal is a nonhuman source of infections for patients for Mycobacterium leprae:

A. Armadillos
B. Horses
C. Tic
D. Fleas
E. Pigs

A

►A

Leprosy is caused by mycobacterium leprae. Armadillos are a source of nonhuman source for leprosy. They are also transmitted from human to human most likely from respiratory secretions.

294
Q

295- These are all old world Leishmaniasis except for:

A. L. mexicana
B. L. major
C. L. tropica
D. L. aethiopica
E. L. infantum

A

►A

The organisms that are responsible for old world Leishmaniasis are L. major, L. tropica, L. aethiopica, L. infantum, L. donovani. Those that are responsible for new world Leishmaniasis are in south and central America which are L. mexicana, L. braziliensis, and L. amazonensis.

295
Q

296- Which subtype of the human papilloma virus is most likely to have caused this infection?

A. HPV, type 1
B. HPV, type 2
C. HPV, type 3
D. HPV, type 5
E. HPV, type 6

A

►E

The human papilloma virus is a member of the Papovavirus family, a double-stranded DNA virus. HPV, type 6 has been implicated in giant conduloma of Buschke and Lowenstein and anogenital condyloma.

296
Q

297- Streptococcus Iniae has been shown to cause:

A. Perianal dermatitis in neonates
B. Necrotizing fasciitis
C. Bullous impetigo
D. Hand cellulitis in fish handlers
E. Perineal erysipelas in postpartum women

A

►D

Streptococcus iniae has been demonstrated to cause hand cellulitis from puncture wounds sustained form the dorsal fin, fish bone or knife of usually a tilapia. Treatment with PCN is curative. Group A Streptococci are the most common cause of perianal dermatitis. Many different bacteria have been implicated in necrotizing fasciitis. Bullous impetigo is most frequently caused by phage type 71 S. aureus or a related group 2 phage type. Group B streptococcus is most often responsible for perineal erysipelas in postpartum women.

297
Q

298- Which of the following organisms is spread by non-sexual person-to-person contact?

A. Treponema carateum
B. Chlamydia trachomatis types I, II & III
C. Hemophilis ducreyi
D. Calymmatobacterium granulomatis
E. Treponema pallidum

A

►A

Treponema carateum is the cause of Pinta which is primarily transmitted by direct contact. Occasional insect vectors have also been implicated, but sexual contact is not the primary cause of spread. The other options (C. trachomatis, H. ducreyi, C. granulomatis and T. pallidum) are all venereal diseases.

298
Q

299- A patient has a painless, soft erosion that heals spontaneously. The p atient also has secondary inguinal adenopathy with fluctuant, tender nodes above and below Poupart’s ligament. This patient most likely has:

A. Lymphogranuloma venereum
B. Granuloma inguinale
C. Chancroid
D. Chancre
E. Tertiary syphillis

A

►A

The patient has lymphogranuloma venereum that is caused by Chlamydia trachomatis L1, L2, L3. It is a painless soft erosion that heals spontaneously. Serologic diagnosis is by complement fixation test. The treatment is doxycycline.

299
Q

300- A 52-year old male develops a brownish-red plaque on his forehead. The lesion has an “apple jelly” color on diascopy, and the patient has a positive PPD skin test. What is the most likely mechanism of disease in this patient?

A. Spread from a distant site
B. Exogenous re-infection
C. Primary inoculation
D. Contiguous spread to skin from underlying infection
E. Id reaction

A

►A

This patient has lupus vulgaris, a pauci-bacillary manifestation of tuberculosis infection. Lesions occur in sensitized patients with a moderate to high immune response and are due to hematogenous, lymphatic, or contiguous spread from a distant site of infection. Lesions typically occur on the head and neck. “Apple jelly” color on diascopy is described in lupus vulgaris, but can also be seen in cutaneous lesions of sarcoidosis.

300
Q

301- A patient has palpable purpura, arthralgias and glomerulonephritis and is diagnosed with cryoglobulinemia. The most common infection associated with this is:

A. Hep C
B. Hep A
C. Hep B
D. Epstein Barr Virus
E. CMV

A

►A

This patient has cryoglobulinemia. This is associated with HCV infection. Approximately 80% of cases of mixed cryoglobulinemia are associated with HCV infection.

301
Q

302- A patient presents with painful vesicles in a dermatomal distribution on his forehead and on the tip of his nose. He is sent by his ophthalmologist who diagnosed ophthalmic zoster. Which nerve branch is involved?

A. Ophthalmic

B. Nasal
C. Infraorbital
D. Supraorbital
E. Temporal

A

►A

The ophthalmic branch of the trigeminal branch is involved in ophthalmic zoster. This presentation accounts for 10-15% of all cases of VZV. The other nerve branches listed are not involved in ophthalmic zoster.

302
Q

303- Coxsackievirus A16 is implicated in the pathogenesis of which of the following dieases?

A. Exanthem subitum
B. Fifth disease
C. Papular purpuric “gloves and socks” syndrome
D. Erythema infectiousum
E. Hand-Foot-and-Mouth disease

A

►E

Hand-Foot-and-Mouth disease is caused by coxsackievirus A16. Group A coxsackievirus infection is also associated with herpangina.

303
Q

304- A patient is diagnosed with hepatitis C. The most common associated autoimmune disorder associated with hepatitis C is:

A. autoimmune thyroiditis
B. aplastic anemia
C. autoimmune thrombocytopenic purpura
D. Peripheral neuropathy
E. Arthralgias

A

►A

The most common associated autoimmune disorder is autoimmune thyroiditis. The others can be associated with hepatitis C extra systemic manifestation but is not as common.

304
Q

305- A middle aged man of southern-eastern european heritage has red to blue-black patches on the feet. The most likely cause of this disease is:

A. HHV-8
B. HHV-7
C. HHV-6
D. HHV-5
E. HHV-4

A

►A

This patient above has classic kaposi’s sarcoma. It is caused by HHV-8 that causes the abnormal vascular proliferation associated with this condition. Classic KS is seen in middle aged men from southern-eastern european heritage.

305
Q

306- Which of the following is a gram positive rod?

A. Meningococcemia
B. Gonorrhea
C. Glanders
D. Melioidosis
E. Anthrax

A

►E

Anthrax is a gram positive, spore-forming rod. Meningococcemia, gonorrhea, Glanders, and Meloioidosis are all gram negative. Infection with anthrax initially begins after skin onoculation or may follow after ingestion or inhalation of spores. Cutaneous anthrax initially begins as a localized

infection and subsequently evolves to toxemia. It appears as a painless papule that ay later become hemorrhagic or nerotic. It produces two exotoxins: edema toxin and lethal toxin. Edema toxin is comprised of edema factor and protective antigen. Lethal toxin is composed of lethal factor and protective antigen.

306
Q

307- The vector of Trench Fever is the:

A. Human body louse (Pediculus humanus corporis)
B. Cat flea (Ctenocephalides felis)
C. Sandfly (Phlebotamus perniciosus)
D. Rat flea (Xenopsylla cheopis)
E. Trombiculid mite

A

►A

The human body louse (Pediculus humanus corporis)is the vector of Trench Fever, Epidemic Typhus, and Relapsing Fever.

307
Q

308 -The causative organism for Brucellosis is which of the following:

A. Gram positive rod
B. Gram positive cocci
C. Gram negative rod
D. Gram negative cocci
E. Acid fast bacilli

A

►C

Brucella species which cause Brucellosis are gram negative rods. Brucellosis, aka undulant fever, is characterized by an acute febrile illness with headache and joint pain. CNS and cardiac manifestations can also occur. It is acquired by contact with infected animals or contaminated dairy products. Treatment is with doxycycline and rifampin.

308
Q

309- What is the treatment for secondary syphillis in a penacillin allergic patient?

A. Penicillin
B. Doxcycyline
C. Rifampiin
D. Clindamycin
E. Ceftriaxone

A

►B

Secondary syphillis is caused by the spirochete, T. pallidum. Penicillin is the treatment of choice for secondary syphillis. In penicillin allergic individuals, doxycyline may be used. Treatment for neurosyphilis, congenital syphilis and syphilis in pregnant women is desensitization to penicillin and then treatment with pcn.

309
Q

310- A 27 year-old man with a 1-month history of bilateral inguinal lymphadenopathy and a positive “groove sign”. What is the most likely diagnosis?

A. Syphilis
B. Herpes simplex
C. Chancroid
D. Lymphogranuloma venereum
E. Gonorrhea

A

►D

Inguinal adenopathy with fluctuant, tender nodes above and below Poupart”s ligament - referred to as “groove sign” - is a characteristic clinical feature of lymphogranuloma venereum.

310
Q

311- A 45 year old septic female develops symmetric, large ecchymotic areas with irregular borders on the extremities, ears, and nose. Laboratory testing reveals the patient has disseminated intravascular coagulation. Which of the following is the most common underlying infection associated with this condition?

A. Group A streptococcus
B. Meningococcus
C. Staphylococcus
D. Pseudomonas
E. Mycobacterium

A

►B

Purpura fulminans is an oftentimes dramatic presentation of large ecchymotic areas of the skin, favoring acral sites including the extremities, ears, and nose. The underlying pathophysiology is hemorrhagic infarction of the skin due to disseminated intravascular coagulation. Meningococcal infection is the leading underlying infection associated with sepsis and purpura fulminans. When Kids develop purpura fulminans after an infection the most common cause is group A Strep.

311
Q

312- What characteristic color is seen on diascopy of this lesion?

A. Apple jelly
B. Blue
C. Orange
D. Red
E. Coral red

A

►A

Leishmaniasis recidivans is a type of Old World leishmaniasis. Clinically, it appears as a red papule covered with white scale. On diascopy, it has a characteristic apple jelly color.

312
Q

313- Neisseria gonorrheae would be identified with:

A. Sheep”s blood agar
B. Agar supplemented with heme and nicotinamide
C. Chocolate agar with 10% CO2
D. Chocolate agar

E. Darkfield microscopy

A

►C

Neisseria gonorrheae should be grown on chocolate agar in 10% CO2. Thayer”Martin, Martin- Lewis or New York City medium also allow for N. gonorrheae growth. Agar supplemented with heme (x-factor) and nicotinamide (v-factor) is needed for Hemophilus influenzae cultures.
Darkfield microscopy is helpful in diagnosing infections with Treponema pallidum (syphilis). Sheep”s blood agar is useful for identifying hemolytic strains of streptococcus and staphylococcus.

313
Q

314 -A patient from South America develops a recurrent fever after living in crowded conditions with multiple roommates having known human body louse infestation. A diagnosis of louse-born Relapsing Fever is made. Which of the following is the implicated organism?

A. Borrelia recurrentis
B. Borrelia duttonii
C. Borrelia hermsii
D. Borrelia burgdorferi
E. Streptobacillus moniliformis

A

►A

Louse-born relapsing fever is caused by Borrelia recurrentis. It is seen in Africa and South America, and is spread by the human body louse (Pediculus humanus). Symptoms include paroxysmal fevers, headache, myalgias, and petechial macules on trunk and extremities. Borrelia burgdorferi causes Lyme disease; Borrelia duttonii and Borrelia hermsii cause tick-born Relapsing Fever; Streptobacillus moniliformis causes Rat-bite fever.

314
Q

315-A 27 year-old woman who is 30 weeks pregnant presents with erythema migrans. The treatment of choice for this patient is:

A. Doxycycline

B. Erythromycin
C. Chloramphenicol
D. Clindamycin
E. Amoxicillin

A

►E

In the setting of pregnancy, amoxicillin is the treatment of choice for Lyme Disease since doxycycline is contraindicated.

315
Q

316 -Which of the following HPV types causes a ridged wart?

A. HPV 1
B. HPV 7
C. HPV 11
D. HPV 18
E. HPV 60

A

►E

HPV 60 infection leads to a particular type of plantar wart called the ridged wart. HPV 1 leads to plantar warts and myrmecia. HPV 7 causes butcher’s warts. HPV 11 along with 6 causes anogenital condyloma and giant condyloma of Bucke and Lowenstein (verrucous carcinoma). HPV 18 causes anogenital dysplasia and neoplasms.

316
Q

317- The most common extracutaneous complications of varicella zoster virus is:

A. Lymphoreticular
B. Musculoskeletal
C. Cardiovascular
D. Central nervous system
E. Genitourinary

A

►D

Zoster usually resolves without sequelae in children and young adults with intact immune systems. However, the pain, cutaneous eruption and complications of zoster become more severe with increasing age and immune compromise. Complications of zoster include post-herpetic neuralgia (PHN), secondary bacterial infection, scarring, ophthalmic zoster, Ramsay-Hunt syndrome, meningoencephalitis, motor paralysis, pneumonitis and hepatitis.

317
Q

318- Which of the following is a pox virus?

A. Molluscum contagiosum
B. Herpes simplex virus
C. Pediculosis capitis
D. Sarcoptes scabei
E. Human immunodeficiency virus

A

►A

Of the answer choices, molluscum contagiosum is the only pox virus. Pediculosis capitis, the cause of lice, and sarcoptes scabei, the cause of scabies are not viruses. Herpes simplex virus is a herpes virus and HIV is a lentivirus.

318
Q

319- Where do you see Gamma-Favre bodies?

A. Granuloma inguinale
B. Lymphogranuloma venereum (LGV)
C. Syphilis
D. Chancroid
E. Herpes virus

A

►B

LGV is a sexually transmitted disease that is characterized by suppurative inguinal adenitis with matted lymph nodes, inguinal bubo with secondary ulceration, and constitutional symptoms. It is caused by Chlamydia trachomatis serotypes L1, L2, and L3. First line treatment is doxycycline 100mg bid for 3 weeks as well as treating the sexual partners. Gamma-Favre bodies are found in histiocytes in LGV.

319
Q

320- An otherwise healthy six-day-old boy is brought into clinic by his mother with the complaint of tiny thin-roofed vesicles with a rim of inflammation and a few lesions covered with a “honeycolored” crust. Which organism is the most likely cause in this newborn?

A. Listeria monocytogenes
B. Group B streptococcus
C. Group D streptococcus
D. Staphylococcus Aureus
E. Lancefield type O organisms

A

►B

Group B streptococcus is S. agalactae, commonly isolated from normal vaginal mucosa. In neonates, impetigo caused by group B streptococci may develop. Listeria monocytogenes is a small, gram-positive organism that may present as petechial, papular or pustular eruptions in acutely ill infants, usually those that were meconium stained at birth. The usual cause of ecthyma is Group A streptococcus, though other organisms can occasionally cause similar cutaneous infections. Group D streptococcus/Lancefield type O organisms are enterococci. S. aureus can secondarily infect a lesion of ecthyma but is not the usual cause of this condition.

320
Q

321- The leading infectious cause of deafness and mental retardation in the U.S. is:

A. Congenital Rubella
B. Congenital varicella
C. Congenital Cytomegalovirus (CMV)
D. Congenital syphilis
E. Congenital herpes simplex

A

►C

Congenital CMV is the leading infectious cause of deafness and mental retardation in the U.S.

321
Q

322- A patient with AIDS develops umbilicated papules consistent with molluscum contagiosum. What anatomic sites are favored for molluscum papules in patients with HIV?

A. Face and genitalia
B. Head and neck
C. Abdomen and back
D. Upper and lower extremities
E. Chest and shoulders

A

►A

Molluscum contagiosum is a common opportunistic infection of HIV patients. Lesions can be seen anywhere, but the most common locations are on the face and genitalia.

322
Q

323- A 56 year old homeless man has many waxy concretions on his axilla and groin hairs. Which of the following organisms is the likely cause?

A. Corynebacterium afermentans
B. Corynebacterium tenius
C. Corynebacterium diptheriae
D. Micrococcus sedentarius
E. Corynebacterium minutissimum

A

►B

This condition described is trichomycosis axillaris. Despite the “mycosis” in the name, it is not fungal in etiology, but caused by C. tenius. Treatment is topical erythromycin or simply shaving the infected hairs. The other listed organisms do not cause this type of infection.

323
Q

324- Herpes-associated erythema multiforme is most commonly associated with which HLA type?

A. HLA B27
B. HLA DR2
C. HLA DR3
D. HLA B15
E. HLA B8

A

►D

Recurrent erythema multiforme (EM minor) is usually caused by recurrent herpes simplex, most commonly HSV-1 orolabial disease. This is more correctly now called herpes-associated erythema multiforme(HAEM) and has been more commonly associated with people with HLA type B15.

324
Q

325 -A patient presents with fever, irritability and a scarlatiniform erythema with accentuation in the flexural areas. Nikolsky positive blisters develop within 48 hours. Which of the following would predict the best outcome?

A. Frequent NSAID use
B. Normal renal function
C. Normal immune system
D. Age < 5 years old
E. Age > 18 years old

A

►D

Staphylococcal scalded skin syndrome is described in the question. This is predominantly a disease of young children, < 5 years old. The mortality is low in children, but in adults it exceeds 50% due to sepsis and electrolyte and fluid imbalance. NSAID use may predispose patients to a poorer outcome. Normal renal function and immune system are helpful, but do not predict the best outcome.

325
Q

326- The most common bacterial cause of purpura fulminans is:

A. S. aureus
B. P. aeruginosa
C. H. influenza
D. E. coli
E. Group A streptococcus

A

►E

Group A streptococcus and Neisseria meningitidis are the leading bacterial causes of purpura fulminans.

326
Q

327- Which of the following mycobacterial organisms is classified as a photochromogen?

A. M. Kansasii
B. M. grodonae
C. M. fortuitum
D. M. ulcerans
E. M. tuberculosis

A

►A

M. Kansasii, M. marinum, and M. simiae are photochromogens. They form pigment when exposed to light.

327
Q

328- Which of the following serologic tests for syphilis is the most specific early-on in the disease course?
A. RPR
B. FTA-Abs
C. MHA-TP
D. ELISA - Captia (IgM) EIA
E. VDRL

A

►D

ELISA - Captia (IgM) EIA is 100% specific for treponemes and is the serology of choice in early primary or congenital syphilis because it is the first to become reactive. FTA-Abs is the most sensitive serologic test for syphilis and MTA-TP stays positive for life (but is less sensitive than FTA-ABs during early disease). The two nontreponemal tests for syphilis include VDRL and RPR. VDRL is used to follow response to therapy.

328
Q

329- A 36 year-old homeless man presents with a tender suppurative nodule on the mandible. “Sulfur granules” are present on microscopy. The most likely diagnosis is:

A. Actinomycetoma
B. Actinomycosis
C. Anthrax
D. Acne Conglobata
E. Aspergillosis

A

►B

The presentation of a suppurative nodule discharging sulfur granules on the jawline of a patient with poor oral hygeine is characteristic of actinomycosis - caused by the anaerobic gram-positive rod, Actinomyces israelii. This differs from Actinomycetoma, which is a slowly progressive subcutaneous infection characterized by tumefaction, draining sinuses, and an exudate containing grains on microscopy. Unlike Eumycetomas - caused by fungi - Actinomycetomas are bacterial infections. The foot is the most common location, followed by the thoracic area.

329
Q

330- A child presents with fever, cutaneous tenderness and erythema of flexural and periorifacial areas. Within 24 hours the erythema progresses to flaccid blisters and erosions. No organisms are cultured and Nikolsky’s sign is positive. What is the most likely diagnosis:

A. Staph scalded skin syndrome
B. Bullous impetigo
C. Fogo selvagem
D. Pemphigus
E. Pemphigoid

A

►A

Staph scaled skin syndrome is caused by exfoliative toxins A and B (ET-A and ET-B) produced by
S. aureus (phage group II, types 71 and 55 most common). Histopathology of the skin shows superficial intraepidermal splitting without epidermal necrosis with very few inflammatory cells. Desmoglein 1 is the specific receptor for exfoliative toxin A cleavage. Desmoglein 1 is the antigen targeted in Fogo selvagem, the vector for which is the black fly. Bullous impetigo is caused by Staph and is a subgranular blister with bacteria and neutrophils on histopahtology. Pemphigoid and pemphigus would be less likely in a child.

330
Q

331- Which antibiotic is best to treat this condition caused by Bartonella hensalae.

A. Erythromycin
B. Ceftriaxone
C. Clindamycin
D. Trimethoprim/sulfamethoxasole
E. Fluconazole

A

►A

Bartonella hensalae is the causative agent of bacillary angiomatosis. The differential diagnosis of bacillary angiomatosis may include pyogenic granulomas and Kaposi’s sarcoma. The treatment of choice is erythromycin.

331
Q

332- Most common causative agent for the lesion immunocompetent man is shown in image for this 34-year old

A. Staphaylococcus aureus
B. Group A Beta hemolytic streptococcus
C. Pseudomonas aeruginosa
D. Klebsiella
E. Proteus

A

►B

Ecthyma is a deep or ulcerative type of pyoderma commonly seen on the lower extremities and buttocks and caused most often by GABHS, in addition to staphyloccous aureus which also can be involved in the infection. Ecthyma can present as small punched-out ulcers or a deep spreading ulcerative process. The disorder begins in the same manner as impetigo, often following infected insect bites or minor trauma, but penetrates through the epidermis to produce a shallow ulcer. The initial lesion is a vesiculopustule with an erythematous base and firmly adherent crust. Removal of the crust reveals a lesion deeper than that seen in impetigo, with an underlying saucer-shaped ulcer and raised margin. The lesions are painful and heal slowly over a few weeks, often with scar formation. Ecthyma gangrenosum is a cutaneous finding that may be seen in patients with Pseudomonas aeruginosa bacteremia. Most of the affected individuals have an underlying immunodeficiency (either congenital or acquired) or a history of cancer chemotherapy. Neutropenia may be a risk factor for ecthyma gangrenosum

332
Q

333- Unilateral palpebral and periorbital edema is characteristic of which of the following infectious diseases?

A. African trypanosomiasis
B. American trypanosomiasis
C. Amebiasis
D. Schistosomiasis
E. Onchocerciasis

A

►B

American trypanosomiasis (Chagas‟ disease) is caused by infection with Trypanosoma cruzi via Triatoma arthropod vectors (reduviid bugs) and is prevalent predominantly in Central and South America. Transmission of this protozoal infection may occur following the bite of an infected reduviid bug, who then defecates on human skin after feeding, allowing the metacyclic trypomastigotes entry into the bite wound. Alternatively, infected feces may gain direct entry through mucosal surfaces, including the conjunctivae. When the latter occurs, characteristic unilateral palpebral and periorbital edema ensues, known as “Romana‟s sign.” Entry into the skin may result in a localized plaque of erythema and induration with regional lymphadenopathy, termed a “chagoma.” Other systemic complications resulting from chronic infection include cardiac arrhythmias, congestive heart failure, megacolon, and megaesophagus. Treatment is with nifurtimox or benznidazole.

333
Q

334- What is the following is the best choice for treatment of herpes zoster?

A. Ganciclovir
B. Foscarnet
C. Valacyclovir
D. Gabapentin
E. Cidofovir

A

►C

First line treatment of herpes zoster (shingles) is with valacyclovir, acyclovir, or famciclovir. Gabapentin may be used for post-herpetic neuralgia. Cidofovir and ganciclovir are treatment options for cytomegalovirus. Foscarnet is used to treat acyclovir resistant herpes simplex infections.

334
Q

335- Which of the following is true regarding the Gardasil vaccine?

A. It is a live vaccine
B. It is FDA approved for females aged 18 to 25
C. It protects against HPV 16 and 18 only
D. It can be administered regardless of history of abnormal pap smear

E. It does not protect against genital warts

A

►D

Gardasil is a quadrivalent vaccine containing HPV types 6, 11, 16 and 18. It is approved for females ages 9 to 26. Neither HPV testing nor pap smears are necessary before vaccine administration. It is recommended for the prevention of cervical, vaginal, and vulvar cancers, as well as for genital warts.

335
Q

336- Non tuberculosis mycobacterial infections include M. marinum treatment would include:

A. Clarithromycin
B. Cephalosporin
C. Penicillins
D. Quinolones
E. Azithromycin

A

►A

M. marium is also known as fish tank granuloma. Empiric therapy is suggested with M. marinum infection and the first line treatment is minocycline, #2 is clarithromycin.
c

336
Q

337 -American trypanosomiasis is transmitted by humans via reduviid or kissing bugs. It is caused by:
A. Trypanosome cruzi
B. Toxoplasma gondii
C. Pneumocystis carinii
D. Entamoeba histolytica
E. Tamandua tetradactyla

A

►A

African trypanosomas is also known as sleeping sickness and is caused by Trypanosoma brucei and transmitted by the tsetse fly Glossina. It can occasionally have cutaneous manifestations such as chancre and trypanids.

337
Q

338- Streptobacillus moniliformis is the causative organism of which infectious disease?

A. Rat-bite fever (Haverhill fever)
B. Scrub typhus
C. Tularemia
D. Glanders
E. Cat scratch disease

A

►A

Rat-bite fever (Haverhill fever) is caused by Streptobacillus moniliformis.

338
Q

339- Which of the following is the causative organism of Tularemia?

A. Burkholderia mallei
B. Streptobacillus moniliformis
C. Francisella tularensis
D. Orientia tsutsugamushi
E. Klebsiella pneumoniae
c

A

►C

Tularemia is caused by infection with Francisella tularensis - most commonly acquired after contact with infected rabbits (e.g. hunters).

339
Q

340- A woman who handles fish tanks develops a slow-growing cluster of papules on the hand. What temperature (Celsius) is most optimal for this organism’s growth?

A. 25 degrees
B. 30 degrees
C. 37 degrees
D. 40 degrees
E. 42 degrees

A

►B

This patient most likely has contract Mycobacterium marinum, otherwise known as “fish tank granuloma.” On primary isolation M. marinum grows best at 30-33 degrees C in 7-21 days. Unlike Mycobacterium tuberculosis, most strains of M. marinum will not grow at the usual incubation temperature of 37 degrees C. Dimorphic fungi have both yeast and fungal forms: hyphae morphology at 25 degrees C and yeast phase at 37 degrees C.

340
Q

341- A patient recently had pharyngitis and now has an eruption of guttate psoriasis. Which of the following tests would be helpful in determining that this patient had a Streptococcal infection?

A. A complete blood count
B. FTA-ABS
C. U1-RNP
D. DNAse B
E. Serum calcium

A

►D

The DNAse B, hyaluronidase or streptolysin O (ASLO) antibodies in serum can be helpful for confirming streptococcal infection in situations where streptococci cannot be isolated. The FTA ABS is used for detecting syphilis, and is the only test that is positive in the first 14 days of infection. U1-RNP is an extractable nuclear antibody which is a marker for Mixed Connective Tissue Disease.

341
Q

342- At what rate of speed does this parasite migrate through the skin?

A. 0.1 cm/day
B. 1 cm/day
C. 2 cm/day
D. 10 cm/day
E. 100 cm/day

A

►C

The infection depicted is larva migrans or creeping eruption which is caused by Ancylostoma braziliense. This nematode is typically a hookworm of cats and dogs. It migrates at a rate of 2 cm/ day. Larva currens migrates at a rate of 10 cm/day.

342
Q

343- A male patient experiences painful vesicles on the palate, tongue, and ipsilateral eardrum. Additionally, he complains of vertigo and difficulty with speaking and eating. Involvement of what anatomic structure is responsible for this constellation of symptoms?

A. Geniculate ganglion
B. Dorsal root ganglion
C. Hippocampus
D. Basal portion of pons
E. Hypoglossal nerve

A

►A

The question stem describes Ramsay-Hunt syndrome, which is characterized by unilateral vesicles on palate, tongue, earlobe, and eardrum, in association with neurologic symptoms including vertigo, difficulty speaking, difficulty eating, hearing loss, and facial droop. The syndrome is caused by varicella-virus infection of the geniculate ganglion.

343
Q

344- A 37-year-old man presents with a red, tender papule on his lateral right hand that subsequently becomes necrotic with regional lymphadenopathy. An exanthem is also present. Which animal was this individual hunting?
A. Rabbits
B. Grouse
C. Squirrels
D. Deer
E. Moose

A

►A

The presentation described above is that of Tularemia, caused by Francisella tularensis. The most common form of Tularemia is the ulceroglandular form, which is described above. Infected animals, most commonly rabbits, transmit the organism though occasionally tick bites or deer flies may be the vector. Squirrels may be the vector of bubonic plague in endemic areas. The other animals would not cause a syndrome as described.

344
Q

345- Which of the following Rickettsial diseases would be rarely positive on Weil-Felix test?

A. Rocky Mountain Spotted Fever
B. Epidemic Typhus
C. Rickettsial pox
D. Endemic typhus
E. Scrub Typhus

A

►C

Rickettsial pox is diagnosed by clinical suspicion and confirmatory biopsy, the Weil -Felix test is rarely positive. The other listed Rickettsial diseases will have positive Weil-Felix tests. The Weil- Felix test makes use of the cross-reaction between antigens from the gram-negative Proteus vulgaris species and Rickettsia spp. It is not specific and rising titers in the appropriate clinical presentation is necessary for confirmation. The Proteus antigens OX-K, OX-19 and OX-2 are employed.

345
Q

346- A 10 year-old with a 3-day history of fever and headache presents to the clinic with multiple erythematous papulopustules on his extremities, many of which have central eschars. The most likely diagnosis is:

A. Lyme disease
B. Rickettsialpox
C. Erlichiosis
D. Rocky mountain spotted fever
E. Meningococcemia

A

►B

Rickettsialpox is an acute febrile illness caused by the bacteria Rickettsia akari, a member of the spotted fever group. R. akari is transmitted by the house mouse mite, Liponyssus sangineus. Clinically, patients have an eschar at the inoculation site with a subsequent papulovesicular rash, fevers, headache, chills, diaphoresis, myalgia, and anorexia.

346
Q

347- There are four clinical signs of granuloma inguinale and one can see Donovan bodies on microscopy. Granuloma inguinale is caused by:

A. Klebsiella granulomatis
B. Chlamydia trachomatis
C. H. ducreyi
D. T. pallidum
E. Pseudomonas aeruginosa

A

►A

Granuloma inguinale is caused by Klebsiella (old name: Calymmatobacterium) granulomatis. The primary lesions are pseudobubo or ulcer. There are four clinical forms: ulcerovegetative, nodular, hypertrophic, and cicatricial. Donovan bodies are seen on microscopy.

347
Q

348- The usual cause ecthyma is:

A. Group A streptococcus
B. Group B streptococcus
C. Group D streptococcus

D. Staphylococcus Aureus
E. Lancefield type O organisms

A

►A

The usual cause of ecthyma is Group A streptococcus, though other organisms can occasional ly cause similar cutaneous infections. Group B streptococcus is S. agalactae, commonly isolated from normal vaginal mucosa. Infections can be problematic for neonates, but not a frequent cause of ecthyma. Group D streptococcus/Lancefield type O organisms are enterococci. S. aureus can secondarily infect a lesion of ecthyma but is not the usual cause of this condition.

348
Q

349- Which of the following is the vector for Dengue fever?

A. Phlebotomus papatasii
B. Culex mosquito
C. Aedes aegypti
D. Lutzomyia verrucarum
E. Xenopsylla cheopis

A

►C

Dengue fever also known as “break-bone” fever is characterized by sudden high fever, backache, retro-orbital pain, bone/joint pain, weakness, and malaise. It is caused by an arbovirus (RNA virus) which is transmitted by Aedes aegypti, a species of mosquito.

349
Q

350- What is the most common cause of focal epithelial hyperplasia?

A. Ebstein Bar virus(EBV)
B. Pachyonychia Congenita
C. human immunodeficiency virus(HIV)
D. human papilloma virus(HPV)
E. Dyskeratosis Congenita

A

►D

Focal epithelia hyperplasia, also known as Heck’s disease, is caused by HPV types 13, 32 and 57. It is most commonly found on the lower lip but also on the buccal mucosa, gums and tongue. It is mainly a disease of native Americans and Greenlander Eskimos. EBV causes an oral hairy leukoplakia in patients with AIDS. Pachyonychia Congenita patients commonly develop a benign oral leukoplakia, while Dykeratosis Congenita patients develop a premalignant oral leukoplakia.

350
Q

351- Tricomycosis axillaris is caused by:

A. Burkholderia mallei
B. Proteus species
C. Corynebacterium tenuis
D. Micrococcus sedentarius
E. Corynebacterium minitissimum

A

►C

Corynebacterium tenuis is the causative organism associated with most cases. Although up to 33% of adults have colonization by this bacterium in the inguinal or axillary regions, factors such as hyperhidrosis predispose to more extensive growth and resultant clinical manifestations.

351
Q

352- What is the classic CXR finding of a patient with inhalational anthrax?

A. Alveolar infiltrates
B. Increased interstitial markings
C. Pleural effusions
D. Widened mediastinum
E. Normal x-ray

A

►D

Inhalation anthrax is the most lethal form of anthrax. Other forms include cutaneous anthrax and GI anthrax. The classic radiographic finding is a widened mediastinum.

352
Q

353- A 21 year old female with a family history of C8 complement deficiency presents with sparsely distributed hemorrhagic vesiculopustules on the palms, soles, and over joints. She has associated fever, chills, arthralgias, and malaise. What is the treatment of choice for disseminated gonococcemia?

A. Ceftriaxone IV
B. Penicillin IM
C. Piperacillin-tazobactam IV
D. Levofloxacin PO
E. Azithromycin

A

►A

Gonococcemia is characterized by sparsely distributed hemorrhagic vesiculopustules on the palms, soles, and over joints with associated fever, chills, arthralgias, and malaise. Recurrent cases associated with complement deficiencies (C5-C8). Treatment is with IV ceftriaxone.

353
Q

354- What is the most common single nerve involved with herpes zoster?

A. Facial nerve
B. Trigeminal nerve
C. Spinal nerve C7
D. Spinal nerve T4
E. Spinal nerve T10

A

►B

While herpes zoster can be seen in the dermatome of any cranial or spinal nerve, the most common single nerve involved is the trigeminal nerve.

354
Q

355- Hebra nose deformity is characteristic of infection with which organism?

A. Klebsiella pneumoniae
B. E. coli
C. Streptococcus pyogenes
D. Haemophilus influenzae
E. Serratia marcesens

A

►A

Hebra nose deformity is seen in Rhinoscleroma which is caused by Klebsiella pneumoniae subspecies rhinoscleomatis.

355
Q

356- A 52 year-old butcher presents with a tender lesion on his left hand as shown. The treatment of choice for this condition is:

A. Penicillin
B. Azithromycin
C. Ciprofloxacin
D. Tetracycline
E. Chloramphenicol

A

►A

The clinical description and lesion shown in the image suggest a diagnosis of erysipeloid, caused by Erysipelothrix rhusiopathiae. The treatment of choice is penicillin.

356
Q

357- A laboratory worker undergoes injection of vaccinia virus for vaccination against smallpox. What type of reaction is needed to ensure adequate development of immunity?

A. Vesicle or ulcer surrounded by 4 cm of erythema
B. Lichenoid plaque
C. Development of vesicles at site separate from injection
D. Targetoid plaque
E. Systemic flu-like reaction with fever and myalgia

A

►A

Vaccinia virus is used to vaccinate high-risk individuals against smallpox virus. Development of a vesicle or ulcer with 4 cm of surrounding erythema is a reaction considered to have a high rate of adequate immunization.

357
Q

358- What is the risk of transmission of HSV from active cervical lesions when a C-section is performed?

A. 1%
B. 8%
C. 17%
D. 31%
E. 43%

A

►A

When a C-section is performed in the setting of active cervical lesions of HSV, the risk of maternal- fetal transmission is decreased to 1%. If a child were delivered

358
Q

359- A pool worker develops these painful erythematous nodules with ulceration on his fingers and then his arms. The antibiotic of choice to treat this infection is:

A. Penicillin
B. Ceftriaxone
C. Minocycline
D. Trimethoprim-sulfamethoxasole
E. Cephalexin

A

►C

Sporotrichoid spread may be seen in cutaneous leishmaniasis, actinomycosis, atypical mycobacterial infections, deep fungal infection, and melanoma. Mycobacterium marinum is associated with skin injury followed by exposure to contaminated water, usuall y from an aquarium, lake, or pool. The treatment of choice for M. marinum infections is minocycline.

359
Q

360- A 25-year old male from Egypt complains of slowly enlarging chronic growths on his external nares that are causing nasal obstruction. Biopsy reveals Mikulicz cells on histopathology. Which of the following medications would be most appropriate for treatment of these lesions?

A. Ciprofloxacin
B. Penicillin
C. Erythromycin
D. Trimethoprim-Sulfamethoxazole
E. Imipenem

A

►A

This patient has rhinoscleroma, a chronic granulomatous infection of the nose and upper respiratory tract by Klebsiella pneumoniae rhinoscleromatis. The disease is endemic in Africa, Southeast Asia, Mexico, Central and South America, and Central and Eastern Europe and is rarely reported in the

United States. Presentation may vary from rhinitis to diffuse airway obstruction. Mikulicz cells are foamy, vacuolated macrophages seen in rhinoscleroma. Antibacterial therapeutic choices include ciprofloxacin, tetracycline, and rifampin. Surgical treatment may be required for obstructive lesions.

360
Q

361 -A patient has a granulomatous plaque that is diagnosed as swimming pool granuloma. The treatment for this is:

A. Minocycline
B. Ampicillin
C. Thalidomide
D. Penicillin
E. Rifampin

A

►A

Swimming pool granuloma or fish tank granuloma is caused by M. marinum and begins as a small plaque at the site of inoculation and evolves to a granulomatous plaque. It occurs as a sporotrichoid spread. The treatment is minocycline.

361
Q

362 -Pediculus humanus corporis can transmit:

A. Murine typhus
B. Epidemic typhus
C. Scrub typhus
D. Endemic typhus
E. Oriental typhus

A

►B

The body louse or Pediculus humanus corporis transmits Rickettsia prowasekii the organism responsible for epidemic typhus. Endemic typhus or murine typhus is caused by R. typhi that is transmited by the rat flea/Xenopsylla cheopis. Scrub typhus is caused by R. tsutsugamushi and is transmitted by chiggers or trombiculid mite larvae.

362
Q

The causative organism in Whitmore disease is which of the following?

A. Streptobacillus moniliformis
B. Vibrio vulnificus
C. Klebsiella pneumoniae
D. Pseudomonas aeruginosa
E. Burkholderia pseudomallei

A

►E

Whitmore disease also known as Melioidosis is characterized by pulmonary disease, septicemia, and miliary abscesses. It is caused by Burkholderia pseudomallei.

363
Q

364 -All of the following are features of the Ramsay Hunt Syndrome EXCEPT:

A. Tinnitus
B. Facial paresis
C. Vesicles on the external ear
D. Herpes simplex infection
E. Infection of the geniculate ganglion

A

►D

Ramsay Hunt syndrome results from varicella zoster virus infection of the geniculate ganglion of the seventh cranial nerve (CN VII). It is characterized by vesicles on the external ear or ear canal, tinnitus and/or other auditory symptoms, and ipsilateral facial paresis.